POČETNÍ PRAKTIKUM Mgr. Ing. arch. Petr Kurfürst, Ph.D. MASARYKOVA UNIVERZITA ELPORTÁL Obsah Úvod Kapitola 1 Diferenciální a integrálni počet 1 1.1 Derivace funkcí jedné proměnné........................... 1 1.2 Neurčil ó integrály funkcí jedné proměnné ..................... 4 1.3 Určité integrály funkcí jedné proměnné....................... 8 1.4 Geometrické a fyzikální aplikace integrace funkce jedné proměnné........ 9 Kapitola 2 Základy vektorové a tenzorové algebry 13 2.1 Vektory a matice................................... 13 2.2 Báze a jejich transformace.............................. 18 2.3 Tenzorový počet ................................... 23 Kapitola 3 Obyčejné diferenciální rovnice 35 3.1 Obyčejné diferenciální rovnice 1. řádu....................... 35 3.1.1 Rovnice separovatelné a homogenní..................... 35 3.1.2 Nehomogenní rovnice ............................ 37 3.1.3 Bernoulliova rovnice............................. 38 3.1.4 Rovnice exaktní ............................... 39 3.2 Obyčejné diferenciální rovnice 2. řádu ....................... 40 3.2.1 Rovnice s konstantními koeficienty..................... 40 3.2.2 Rovnice s nekonstantními koeficienty.................... 44 3.3 Soustavy obyčejných diferenciálních rovnic..................... 45 3.3.1 Homogenní soustavy s konstantními koeficienty.............. 45 3.3.2 Nehomogenní soustavy s konstantními koeficienty............. 48 Kapitola 4 Křivkový integrál 51 4.1 Křivkový integrál 1. druhu.............................. 51 4.2 Křivkový integrál 2. druhu.............................. 55 Kapitola 5 Skalární a vektorové funkce více proměnných 59 5.1 Parciální a směrové derivace, úplný diferenciál................... 59 5.2 Kmenová funkce................................... 62 5.3 Diferenciální operátory................................ 65 Kapitola 6 Dvojný a trojný integrál 69 6.1 Plošný integrál 1. druhu............................... 70 6.2 Plošný integrál 2. druhu............................... 76 6.3 Objemový integrál.................................. 78 6.4 Geometrické a fyzikální charakteristiky útvarů .................. 79 Kapitola 7 Integrální věty 85 7.1 Greenova věta..................................... 85 7.2 Stokesova věta .................................... 87 7.3 Gaussova věta..................................... 90 Kapitola 8 Taylorův rozvoj 95 8.1 Rozvoj funkce jedné proměnné ........................... 95 8.2 Rozvoj funkce více proměnných........................... 98 Kapitola 9 Fourierovy řady 101 9.1 Diskrétní Fourierovy řady.............................. 103 9.2 Fourierova analýza.................................. 105 Kapitola 10 Úvod do komplexní analýzy 109 Kapitola 11 Kombinatorika 115 Kapitola 12 Počet pravděpodobnosti 121 Příloha A Křivočaré souřadnice 127 A.l Kartézská soustava.................................. 127 A.1.1 Diferenciální operátory............................ 128 A. 1.2 Plochy, objemy................................ 130 A. 1.3 Vektory polohy, rychlosti a zrychlení.................... 131 A.2 Válcová soustava................................... 131 A.2.1 Diferenciální operátory............................ 133 A.2.2 Plochy, objemy................................ 134 A.2.3 Vektory polohy, rychlosti a zrychlení.................... 135 A.3 Kulová soustava ................................... 136 A.3.1 Diferenciální operátory............................ 138 A.3.2 Plochy, objemy................................ 139 A.3.3 Vektory polohy, rychlosti a zrychlení.................... 140 A.4 Eliptická soustava .................................. 141 A.5 Parabolická soustava................................. 143 A.6 „Anuloidová" soustava............. ................... 145 A. 7 Příklad neortogonální soustavy........................... 146 A.7.1 Diferenciální operátory............................ 149 A.7.2 Plochy, objemy................................ 151 A. 7.3 Vektory polohy, rychlosti a zrychlení.................... 151 Příloha B Stručný úvod do parciálních diferenciálních rovnic 153 B. l Parciální diferenciální rovnice 1. řádu........................ 153 B. l.l Homogenní parciální diferenciální rovnice 1. řádu............. 153 B.l.2 Nehomogenní parciální diferenciální rovnice 1. řádu ........... 155 B.2 Parciální diferenciální rovnice 2. řádu........................ 158 B.2.1 Klasifikace parciálních diferenciálních rovnic 2. řádu........... 158 B.2.2 Metoda fundamentálního řešení (metoda Greenovy funkce) ....... 159 B.2.3 Řešení parabolických parciálních diferenciálních rovnic Fourierovou metodou (metodou separace proměnných) .................. 161 B.2.4 Jednoduché příklady prostorových úloh .................. 167 B.2.5 Řešení hyperbolických parciálních diferenciálních rovnic Fourierovou metodou ..................................... 169 B. 2.6 Ukázka možných způsobů řešení jednoduchých eliptických parciálních di- ferenciálních rovnic.............................. 171 Příloha C Praktické základy numerických výpočtů 175 Cl Numerické metody lineární algebry......................... 175 C.2 Interpolace a regrese................................. 177 C. 2.1 Kubický interpolační splajn......................... 178 C.2.2 Lineární regrese metodou nejmenších čtverců............... 181 C.2.3 Polynomiální regrese metodou nejmenších čtverců ............ 183 C.2.4 Robustní regrese............................... 185 C.3 Numerické metody výpočtů funkcí jedné proměnné................ 186 C.3.1 Hledání kořene funkce jedné proměnné - Newtonova metoda....... 186 C.3.2 Numerické derivování............................ 188 C.3.3 Numerické integrování............................ 189 C.3.4 Jednoduché numerické metody řešení obyčejných diferenciálních rovnic . 190 C.4 Numerické metody výpočtů funkcí více proměnných - řešení parciálních diferenciálních rovnic................................... 193 C.4.1 Hledání kořenů soustavy funkcí více proměnných Newtonova-Raphsonova metoda....................... 193 C.4.2 Principy konečných diferencí........................ 195 C.4.3 von Neumannova analýza stability..................... 198 C.4.4 Laxova metoda................................ 198 C.4.5 Metoda zpětného kroku (Upwind method)................. 199 C.4.6 Laxova-Wendroffova metoda......................... 200 C.4.7 Implicitní schéma............................... 200 C.4.8 Příklad pokročilejšího numerického schématu............... 201 C.4.9 Příklady modelování reálných fyzikálních procesů............. 204 C.5 Paralelizace výpočetních algoritmů......................... 207 Reference 209 Úvod Protože matematika je nejdůležitějším pracovním nástrojem a zároveň vyjadřovacím jazykem fyziky, patří znalost základních matematických postupů, uváděných v této sbírce, k neodmyslitelné výbavě každého, kdo se chce fyzikou hlouběji zabývat. Souhrnný předmět Početní praktikum 1, Početní praktikum 2 představuje praktický kurz, určený studentům prvních semestrů bakalářského studia, bezprostředně navazující na předměty Základní matematické metody ve fyzice 1, Základní matematické metody ve fyzice 2. Smysl početního praktika, jak už samotný název napovídá, spočívá především v praktickém počítání a v detailním procvičování znalostí, získaných ve výše uvedených přednáškách. Tato sbírka představuje souborný studijní materiál, usnadňující výběr příkladů, souvisejících s danými tématy. Je členěná do dvanácti základních kapitol, řazených zhruba podle časové posloupnosti přednášené problematiky, doplněných třemi přílohami, určenými zájemcům o širší znalost základů některých důležitých okruhů matematiky, potenciálně využitelných v dalším studiu i ve fyzikální praxi. Jednotlivé kapitoly jsou vždy uvedeny stručným teoretickým shrnutím daného tématu, které si neklade za cíl podávat matematicky přesný a vyčerpávající výklad (doplněný větami, důkazy, atd.), nýbrž pokud možno jednoduchým a přehledným způsobem zrekapitulovat hlavní zásady pro praktické počítání příslušného problému. Pokud je někde výklad zjednodušen do té míry, že například opomíjí některé předpoklady nebo některá řešení uvedené rovnice, je to v textu uvedeno. Jádro každé kapitoly tvoří potom soubor příkladů, které v dostatečném rozsahu pokrývají danou problematiku. U každého příkladu je také uvedený výsledek, který podle mého názoru studentům, kteří se s danou látkou teprve seznamují, umožní správnou orientaci při počítání. Pro jednodušší zacházení je celá sbírka vybavena modře zvýrazněnými hypertextovými odkazy, umožňujícími v elektronické verzi se kliknutím ihned přesunout na odkazované místo a stejně zvýrazněnými odkazy URL, které po kliknutí automaticky otevřou uvedenou webovou stránku. Ani sebelepší studijní materiál nenahradí vlastní píli a odhodlání těch, kteří o získání znalostí a dovedností usilují, může jim pouze více nebo méně práci usnadnit. Velmi proto uvítám, pokud ti, kteří budou s touto sbírkou pracovat, mně sdělí svoje případné názory, podněty nebo výhrady např. ke srozumitelnosti výkladu nebo obtížnosti příkladů a zároveň mě kdykoli upozorní na jakoukoli nepřesnost nebo nedostatek, který v příkladech nebo v textu odhalí. Petr Kurfürst Kapitola 1 Diferenciální a integrální počet 1.1 Derivace funkcí jedné proměnné • Derivace je jedním ze základních pojmů diferenciálního počtu a matematické analýzy vůbec. Pomocí předem definovaného pojmu limita je derivace funkce jedné proměnné definovaná jako df(x) f(x + h)-f(x) . r o - = 7 (x) = nm-, kde h = Ax je přírůstek nezávisle proměnné x. dx h^o h (1.1) • Následující výčet shrnuje derivace elementárních funkcí jedné proměnné: (Cxn (lnx)' (logaa;)' (sin a;)' (cos a;)' (tgx)' (cotg a;)' (arcsinx)' (arccosx)' (arctgx)' (arccotgx)' (sinhx)' Cnx11 1, kde C G M je konstanta, n G M je konstanta, ď, ínx In ay _ „x - a ln a, kde a > 0 je konstanta, 1 x -, x > 0, 1 a; lna cos x, , x > 0, kde a > 0, a 1 je konstanta, ■ srna: 1 1 sin2 x , x / (2fc + i)-, fceZ, , a; fc7r, fceZ, vr :, -1< x < 1, Vi^2 -1 < x < 1, 1 1+x2' 1 "ľT^2' .x „—x e + e coshx, (1.2 (1.3 (1.4 (1.5 (1.6 (1.7 (1.8 (1.9 (1.10' (1.11 (1.12' (1.13 (1.14 (1.15 Kapitola 1. Diferenciální a integrální počet 2 (coshx)' (tanhx)' (cothx)' sinhx, 1 cosh2 x 1 sinh x 1 — tanh x, = 1 — coth21, i / 0. (1-16) [1.17) (1-18) Z rovnice (1.15) lze odvodit následující identity pro hyberbolometrické funkce (funkce inverzní k hyperbolickým funkcím): ar£ gsinhx = ln (x + V'x2 + 1) a tedy (argsinhx)' 1 argcoshx = ln [x + \/x2 — 1J a tedy (argcoshx)' 1 + x argtanh x = — ln 6 2 argcoth x = — ln 6 2 1 — x x + 1 x — 1 a tedy (argtanh x)' a tedy (argcoth x)' Vx2 + 1 1 i 1 — x2 ' 1 1 — x2 ' (1-19) , x>l, (1.20) -Kx 1. (1.22) Pravidla pro derivování součtu, součinu a podílu funkcí jedné proměnné: (af + pgy U 9)' 7 ľ AM = ^ - / . \gj. af1 + /3 0 8\/x\/x + y/x\Jx + \/x~T^/x 2 sinlnx, x > 0 Vx2 + 1 , x/0 arccos x x G (-1,1), x / 0 \[xx 1 + xx ln x (ln x + 1)] , x > 0 x 1 arcsm x + 1 yfx (x + 1) , x > 0 2, sm x + 2 sin x cos x ln x ) , x > 0 x1 — 1 2x arcsm —-r + x2 + 1 x2 + 1 5xlogxln5 (iogx+ -) , x>0 In 10 1 ;, X > -1 (x + l)ln7! 2ex — (l +e2:E) arctgex e3x _|_ ex sinx\ e^tgVe^TI x cos x m x H--I--:—, —, x > 0 2Vex + 1 1.17 xln2(x + Vl +x2) - 2Vl + x2 ln(x + Vl + x2) + 2x ln2 (x + Vl + x2 Kapitola 1. Diferenciální a integrální počet 4 1.19 /'(27r) funkce f (x) = x2(cosx) /'(2tt) = (cosx)~ feeZ 2x + x sin2 a; cos a: cos x ln(cos x) Tľ 2tt 4tt, x G (4fc-l,4fc + l)-. 1.20 xln(sin:E) „ln(sin x) Vi + x2 ln(sin x) + x ln x cotg x 1 \/l + x2 + 1+x2)3/2 x > 0 A x G (2fc, 2fc + 1) 7T, G 1 21 a;sín(ln:E) Vl +x2 „sin (ln x) Jx2 + 1) [sin(lnx) + lnx cos(lnx)] — 1 x2Vl + x2 , x > 0 „_ _ a sin bx — b cos bx 1.22 e - :-, kde a, b jsou kladne konstanty. V ax2 + b2 a sin bx — b cos bx \J ax2 + b2 ax a cos x — ax srn x + ab cos bx + b sin bx ax2+b2) Vax2 + b2 ■ a sin v bx — b cos v bx 1.23 x -, -, kde a, b jsou kladne konstanty. V a2 + b2 y/b cos\/bx V a2 + b2 ' a sin x / r— \ a + b tg v bx 1 + ln x + x ln x cotg x) I a tg v bx — b I + y/b x > 0. Proč není nutná podmínka x 7^ (2fc + 1) — , fcgZ? 1.24 (axsinx) í , kde a, b jsou kladné konstanty. b (ax sin x) sin x + x cos x xz + ln(ax sin x) cos x srn x x xz x > 0 A x G (2k,2k + 1)tt x < 0 A x G (2k - l,2k)ir' k G 2Vx" 1.25 (axe1) b , ax e x kde a, b jsou kladné konstanty. [(1 + x) lnx + ln(axex) (1 — 2 lnx)] , x>0 1.26 (le31)1'"1, kde a je kladná konstanta. a(x + ln2 x) a(x+]nx) x2ln2 x e x\nx , x > 0, x 7^ 1. 1.2 Neurčité integrály funkcí jedné proměnné • Neurčitým integrálem nazýváme nekonečně velkou množinu funkcí, tvořenou součtem libovolné reálné konstanty C s tzv. primitivní funkcí F(x) k dané původní funkci f(x), Kapitola 1. Diferenciální a integrální počet 5 pro niž platí F'(x) = f(x). V případě funkce jedné proměnné lze psát //(*)<* = FW + C. ;i.27) Integrace je tedy inverzním procesem k derivování, neurčité integrály (anglicky také an-tiáerivatives) některých elementárních funkcí můžeme vyčíst přímo z rovnic (1.2)-(1.22). Následující výčet shrnuje neurčité integrály elementárních funkcí jedné proměnné1: ™n+l dxn dx ex dx &x dx — dx x sin x dx cos x dx dx cos2 x 1 sin2 x dx dx 1 1 +x2 sinh x dx cosh xdx 1 : dx cosh x 1 sinh2 x 1 Vx2 + 1 1 Vx2 ■ dx dx dx dx 1 -x2 1 dx Ci--h C2, Ci, C2 £ I, n É M\{-1}, jsou konstanty, n + 1 e^ + C, C G M je konstanta, In a + C, a > 0, a 7^ 1 je konstanta, ln|x| + Ci =ln(C2|x|), x / 0, C2 > 0, Ci =lnC2, — cos x + C, sinx + C, 7T tgx + C, x / (2k + 1)-, fceZ, — COtgX + C, X 7^ fc7T, fceZ, arcsinx + Ci = — arccosx + C2, —1 1, + C = argtanhx + C, — 1 1, 1 In 1 + x 2 1 — x 1 In x + 1 2 x — 1 1neurčité integrály jsou vyčerpávajícím způsobem tabelovány například v Bartsch (2008), Rektorys (2009). Kapitola 1. Diferenciální a integrální počet 6 L x - 1 - In - x1 - 1 2 x + 1 2 n dx = -ln —p-p +C. (1.45) Součin dvou funkcí u{x) a v'{x) nezávisle proměnné x můžeme integrovat metodou per partes, která je integrálem rovnice (1.24): uv = I (uv)' dx = / (v!v + uv,sj dx a tedy / uv' dx = uv — / u'vdx. (1.46) Substituční metoáou můžeme integrovat složenou funkci (viz rovnice (1.26)), kdy vnitřní funkci lze nahradit novou proměnnou, anebo můžeme integrovat jednoduchou funkci, kdy nezávisle proměnnou nahrazujeme novou vnitřní funkcí: — Substituční metodu 1. typu lze použít při integraci složené funkce f[(f>(x)] nezávisle proměnné x ve tvaru f[(x)]<(>'(x) dx = j f(z) dz = F{z) + C = F[<(>(x)] + C, (1.47) kde můžeme nahradit (substituovat) vnitřní funkci novou proměnnou: (f>(x) = z, (f>'(x) dx = dz. Substituční metodou 1. typu je i univerzální substituce tg (x/2) = z, pomocí které lze libovolnou goniometrickou funkci převést na funkci racionální. Substituční metodu 2. typu lze použít při integraci jednoduché funkce f{x) nezávisle proměnné x způsobem f{x) dx = f f[^(z)]4>'(z) dz = F{z) + C = + C, (1.48) kde můžeme nahradit původní proměnnou novou vnitřní funkcí nové proměnné: x = (f>(z), dx = (j)'(z)dz a kde výraz znamená inverzní funkci k 0. Typickým příkladem této metody je substituce x = sin z, pomocí níž lze iracionální funkce typu y/l — x2 dx anebo dx/ y/l — x2 v integrandu nahradit v prvním případě goniometrickou funkcí cos2 z dz, ve druhém případě pouze dz. Racionální funkci ve tvaru f(x) — ^>m^X) _ amxm + am-lXm 1 + • • • + &iX + ao , . ~ Qn{x) ~ hnXn + bn-iX"-1 + • • • + biX + b0 ' [' ' kde Pm{x) a Qn{x) jsou polynomy stupně man (kdy m > n), lze rozložit na součet polynomu a ryze racionální funkce (kdy m < n). Racionální funkci lze vyjádřit buď jako součet parciálních zlomků, nebo, v případě kdy např. f{x) = 1/Q2(x), kde Q2Íx) je polynom 2. stupně dále nerozložitelný v M, provedeme úpravu (tzv. doplnění na čtverec) b2X2 + bix + bo = [Vb^x + bi/(2v/b2)] + bo — b2/(4b2), vedoucí na integrál ve tvaru rovnice (1.37). Obdobným způsobem můžeme řešit integrály iracionálních funkcí typu f(x) = 1/\JQiip)-, kde Q2Íx) je polynom 2. stupně, jehož doplnění na čtverec vede na integrály ve tvaru rovnic (1.36), (1.42) nebo (1.43). Vyčerpávajícím způsobem jsou metody analytických výpočtů neurčitých integrálů funkcí všech typů tabelovány např. ve sbornících: Bartsch (2008), Rektorys (2009), atd. Kapitola 1. Diferenciální a integrální počet 7 • Příklady: i.27 f ^-tf dx -x? — 3x + 6 y/x — ln \x\ + C, x > O 3 1.28 1.29 x2 — 3 3x - 4 x2 - 4 dx dx --h 3x + ln 3 2 73 + C, x/ ±V3 ln (x + 2)iv/äTr2 +C, x>2 1.30 1.31 dx V2 + 3x - 2x2 dx x2 + 3x + 3 1 /4x-3\ ^ —= arcsm - + C, x G 72 V 5 / 2 ^2x + 3^ , ^ 2'" 1.32 / ŕ-x2 +x)e3:Edx 3 9 27, , 1 + x , L33 VÍ+? Vx2 + 1 + ln (x + Vx2 + 1) + C 1.34 72 1 1 x" 72 + x2 dx x x ,- arcsin —p= + argsinn —= + C, \x\ < \J2 72 72 1.35 / x sinxdx í2 — x2) cos x + 2x sin x + C 1.36 dx smx ln x 6 2 + C, x ku, k G 1.37 / tg3xdx 1 7T --5— +ln|cosx| +C, x/(2fc + l)-, fcG 2 cosz x 2 1.38 1 + cos x sin3 x dx x 1 + cos x ^ ln 4 / tg---^--h C, x G {2k, 2k + l)vr, k G 2 2 sin x 1.39 dx 1 + sin x + cos x ln x 1+tg- + C, x / (4fc + 2,4fc + 3)-, k G 1.40 y arctg \/2x — 1 dx .- V2x - 1 ^ 1 x arctg v2x — 1-----h C, x > — 1.41 dx sin2 x cos2 x Tľ -2 cotg (2x) + C, x/k-, fcG 1.42 dx 7(4-x 2^3 474 - x2 + C, |x| < 2 1.43 / ln (x2 + 1) dx x ln (x2 + l) + 2 arctg x — 2x + C Kapitola 1. Diferenciální a integrální počet 8 1.44 dx >x + 1)3 (4^x~- 3) + C, x > O 1.45 1.46 Vx^T x + 1 v^l x + 1 dx dx 2Vx - 1 - 2v/2arctg x — 1 + C, x > 1 ^2 ^2(x-l) + 2^2(x-l)+2 V V J 23/4 arctg [1 + ^2(x - 1)] + C, x > 1 1.3 Určité integrály funkcí jedné proměnné Určitý integrál funkce f (x) spojité na intervalu a < x < b, je definován předpisem > f(x)dx = F (b) - F (á), [1.50) kde F (a), F (b) jsou funkční hodnoty primitivní funkce F (x) v bodech x = a, x = b. Geometrický význam určitého integrálu funkce jedné proměnné je dán velikostí celkové plochy v rovině xy, kde y = f (x), která je ohraničená grafem funkce f (x), osou x a přímkami x = a a x = b. Velikosti dílčích ploch nad osou x, tj. kde f (x) > 0, přispívají k velikosti celkové plochy, velikosti dílčích ploch pod osou x, kde f (x) < 0, se od velikosti celkové plochy odečítají. • Příklady: .'š xdx 1.47 tl sin2 x cos2 x 6 [x (tg x — cotg x) + ln (sin x cos x)] f Tľ 7s 1.48 o x sin ( — j cos í — ) dx 1 tn ŕ 2x - 1 1.49 / —-dx J 2 x1 — 4x + 5 1.50 j v/4x2~+Idx ln(V4x2 + 1 + 2x) x\/4x2 + 1 sm [ln(x2 - 4x + 5) + 3 arctg (x - 2)]| = ln 2 + 3tt -i 2.96 1.51 / [(x tg x + a) tgx + 1] dx, a je konstanta. x tg x + (1 — a) ln(cos x)--— + x 7T í ~ 2 4 Kapitola 1. Diferenciální a integrální počet 9 ,4 / SIM 1.52 / (x + a cos x dx, a je konstanta. x tg x + ln(cos x)--— + a tg x — &x 2a 1 1.53 x dx o (cos2 x — sin2 x)2 - tg (2x) + ln ^cos(2x) f if*-ln2 o 4 VV3 1.54 / x arctg (x +l)dx x2 + 1 arctg (x2 + 1) - ln ^x4 + 2x2 + 2 7T 12 arctg 2---1— ln — 6 8 4 5 1.55 / x3ln(x2 + l)dx (x4 - 1) ln(x2 + 1) x4 x2 4 ~8~ + T 1.4 Jednoduché geometrické a fyzikální aplikace integrace funkce jedné proměnné2 V následujících příkladech je vždy třeba sestavit určitý integrál funkce jedné proměnné. Pokud je v příkladu zadána spojitě proměnná tzv. intenzivní fyzikální veličina (hustota, tlak, hustota náboje, atd.), pomocí níž se má na dané oblasti stanovit odpovídající extenzivní veličina (hmotnost, tlaková síla, velikost náboje, ...), výpočet provádíme jako integrál intenzivní veličiny přes tuto oblast. Například hmotnost m kruhové desky o poloměru R s plošnou hustotou o = cr(r), kde r je vzdálenost od středu desky, určíme pomocí integrálu m = a(r)dS= / a(r)2iTrdr. (1-51) S JO • Příklady: 1.56 Spočítejte velikost plochy, ohraničené „zespoda" parabolou y = x2 a „shora" křivkou 1 3 1.57 Spočítejte velikost plochy, ohraničené „zespoda" parabolou y = x2 a „shora" přímkou y = x/2 + 5. 243 Tě" 1.58 Rychlost hmotného bodu v jednorozměrném případě je dána vztahem (t + i) Ve výsledcích příkladů s geometrickými nebo fyzikálními veličinami nejsou uváděny příslušné jednotky. Kapitola 1. Diferenciální a integrální počet 10 Určete dráhu, kterou projde hmotný bod v časovém intervalu od t = 0 do zastavení. Bude hmotný bod v tomto časovém intervalu zrychlovat nebo brzdit (tj. bude se zvyšovat nebo snižovat velikost jeho rychlosti) ? s = 6 (1 — 3 ln 3), hmotný bod bude brzdit, vektor zrychlení má opačný směr než vektor rychlosti. 1.59 Přehradní hráz je tvořena svislou betonovou zdí tvaru obdélníku, jehož délka je L. Hloubka vodní nádrže je v celé délce hráze přesně H. Jaká je celková tlaková síla, kterou voda působí na hráz ? (pgH Fp = ^—---h Poj LH, kde po Je atmosférický tlak na hladině. 1.60 Válcová nádoba o poloměru R a výšce H je zcela naplněna plynem, jehož hustota směrem od osy válce klesá. Pokles hustoty je vyjádřen funkcí P = Po e 10 , kde po Je hustota plynu v ose válce, r je vzdálenost od osy válce. (a) Vypočítejte hmotnost plynu v nádobě. (b) Vypočítejte celkovou tlakovou sílu, kterou působí plyn na všechny stěny nádoby, pokud tlak p = a2p, kde a je konstantní (izotermická) rychlost zvuku. (c) Jaká bude celková hmotnost a celková tlaková síla, pokud by poloměr vzrostl „nade všechny meze" (R —> oo) ? (a) m = WttpqH I 1 — e io (b) Fp = 2na2pQ 10 + e-Tô {RH - 10) (c) m = IOtvpqH, Fp = 207ra2p0 1.61 Válcová nádoba o poloměru R a výšce H je zcela vyplněna plynem, jehož tlak směrem vzhůru klesá. Pokles tlaku je vyjádřen funkcí _h_ P = Po e 20 , kde po Je tlak plynu na dně válce, h je svislá vzdálenost ode dna válce. Vypočítejte celkovou sílu, kterou plyn působí na všechny stěny nádoby. Fp = AOpoirR [l-e--o) +p07rRz [1 + e~: 1.62 Válcová nádoba o poloměru R a výšce H je zcela vyplněna plynem, jehož tlak směrem od osy válce klesá. Pokles tlaku je vyjádřen funkcí _ Po P"i + (Ä)" kde po Je tlak plynu v ose válce, r je vzdálenost od osy válce. Vypočítejte celkovou sílu, kterou plyn působí na všechny stěny nádoby. Fp = 2007TPQ RH 1 / R2 + ln 1 + 100 + R2 V 100 Kapitola 1. Diferenciální a integrální počet 11 1.63 Nádoba ve tvaru kvádru o čtvercovém půdorysu s délkou strany A a výšce H je zcela vyplněna plynem, jehož vertikální pokles tlaku je vyjádřen funkcí V Po — + r 10 ^ 1 kde po Je tlak plynu na dně nádoby, h je svislá vzdálenost ode dna nádoby. Vypočítejte celkovou sílu, kterou plyn působí na (všechny) stěny nádoby. Fp = p0A 2A H+ 5 H + 10 + 401ní- + l 1.64 Kruhová deska o poloměru i? je elektricky nabitá s plošnou hustotou náboje o. Vypočítejte celkový elektrický náboj Q desky (velikost náboje by v případě o = konst. byla dána jejím součinem s velikostí příslušné plochy, Q = a S), pokud (a) a = AeBr\ (b) a = Aln(r2 + B), (c) a = Ae 3 + Br, (d) a = A ln (3r2 + B) + Ar, kde A, B jsou kladné konstanty a r je vzdálenost od středu desky. (a) Q tvA ~B~ 1 (b) Q = ttA{(R2 + B)[ln(R2 + B) - 1] - B{\nB - 1)} (c) Q = StvA (l - e" ttA 2 \ 9 + -tvBR3 3 (d) Q [(3R2 + B) ln(3i?2 + B) - B ln B + 2R3 - 3R2 1.65 Tenká přímá tyč (zanedbatelného průřezu) o délce L je kladně nabitá s homogenní délkovou nábojovou hustotou r. Koncové body tyče se nacházejí v bodech (0, 0, 0), (L, 0, 0). Určete elektrostatický potenciál 0 buzený nábojem tyče a vektor intenzity elektrického pole v bodě P = (-D, 0, 0), D > 0 (v případě bodového náboje Q je elektrostatický potenciál 0. Y ATieL \ D Kapitola 1. Diferenciální a integrální počet 12 1.67 Pro stejný případ tyče z příkladu 1.66 určete složky Ex a Ey vektoru intenzity elektrického pole v bodě P = (0, D, 0), D > 0. E = -9- f 1 -1\ E = Q X 47reL \^L2 + D2 D J ' y A-KeD^/Ľ2 + D2' 1.68 Pro stejný případ tyče z příkladu 1.66 určete elektrostatický potenciál (f) a složky Ex a Ey vektoru intenzity elektrického pole v bodě P = (L/2, D, 0), D > 0. 27reL ^ 2D y * > v 2TveDVL2 + 4D2' 1.69 Tenká oblouková tyč zanedbatelného průřezu s konstantním poloměrem i? je kladně nabitá s homogenní délkovou nábojovou hustotou r. Koncové body tyče se nacházejí (v polárních souřadnicích) v bodech (R, 4tv/3, 0), (R, 5tv/3, 0). Určete elektrostatický potenciál (f) a složky Ex a Ey vektoru intenzity elektrického pole buzené nábojem tyče v bodě P = (0,0,0). Výsledek vyjádřete pomocí délkové hustoty r i pomocí celkového náboje Q tyče. t Q t 3Q Ex — 0, Ev Y 12e 47reiť x ' y AneR An2eR2' 1.70 Předpokládejme hypotetické homogenní (p = konst.) kulové astronomické těleso o poloměru R. Gravitační potenciální energie libovolné vnitřní kulové slupky o poloměru r G (0, R) je mgr, kde m je hmotnost kulové slupky a g = GMr/r2 je velikost gravitačního zrychlení v místě slupky [G ~ 6, 67 x 10~n m3 kg-1 s~2 je gravitační konstanta). Veličina Mr značí hmotnost koule o poloměru r. Jak velká bude celková gravitační potenciální energie homogenní koule o poloměru R ? Výsledek vyjádřete pomocí hmotnosti celé koule M a jejího poloměru R. f - 3rM2 Kapitola 2 Základy vektorové a tenzorové algebry 2.1 Vektory a matice • Vektorový počet: Základní operace s vektory lze (v ortonormálních bázích - viz odstavec 2.2) stručně zapsat následujícím způsobem: • Norma (velikost) vektoru a je definovaná (v M3) jako ||a|| = (al + al+alÝ = (l>i) > ^ kde poslední forma zápisu předpokládá, že index i postupně „běží" přes všechny složky 1,2,3, respektive x,y,z vektoru a. Tato konvence pro tzv. volné indexy umožňuje podstatně stručnější zápis operací s vektory a maticemi (v této kapitole zatím pro jednoduchost nezavádíme tzv. kovariantní formu zápisu s horními a dolními indexy). • Vektorový součet dvou vektorů a a b, jehož explicitní forma zápisu je c = a + b = ai + bi, a2 + b2, a3 + b3 = ci, c2, c3, (2.2) lze pomocí uvedené konvence s použitím volného indexu i zapsat jako c = a + b = di^i + 6jě*j = Cjě*j, se složkami en + bi = C{ (vektor), (2-3) kde ě*j jsou vektory ortogonální (resp. ortonormální) báze (podrobněji viz odstavec 2.2). • Skalární součin dvou vektorů a a b má v ortogonální bázi (viz odstavec 2.2) tvar a ■ b = ciibjSíj = dibi = a (skalár), (2-4) kde indexy i, j opět „běží" přes všechny složky obou vektorů a kde symbol Sij (tzv. Kro-neckerovo delta - podrobněji viz odstavec 2.3) nabývá hodnot Sij = 1 pro i = j a Sij = 0 pro i 7^ j. Navíc je zde zavedena tzv. Einsteinova sčítací (sumační) konvence, která říká, že pokud se některý index v nějakém členu vektorové rovnice opakuje dvakrát (tzv. sčítací 13 Kapitola 2. Základy vektorové a tenzorové algebry 14 index), členy s tímto indexem sčítáme a sumační symbol ^2 Je tak možné vynechat. Geometrický význam skalárního součinu v Eukleidovském prostoru lze zapsat jako a ■ b = \\a\\\\b\\ cos tp, (2-5) kde tp je úhel mezi oběma vektory. • Vektorový součin dvou vektorů a a b je (obdobnou formou jako v rovnici (2.4)) v ortogonální bázi (viz odstavec 2.2) definován jako c = a x b = EijkCijbkei = ciei (vektor), (2-6) kde indexy i,j, k označují jednotlivé složky vektorů a, b, c. Symbol Eijk (tzv. Levi-Civitův symbol - podrobněji viz odstavec 2.3) nabývá hodnot e^k = +1 pro sudé permutace indexů (tj. 123, 231, 312), e^k = —1 pro liché permutace indexů (tj. 132, 213, 321) a £íjk = 0 pro nulové permutace indexů (tj. pokud se některý z indexů opakuje). Geometrický význam velikosti vektorového součinu v Eukleidovském prostoru lze zapsat jako a x b = \\a\\\\b\\sin Pro n = 3 bude det A = 011022^33 + a2ia32ai3 + a3iai2a23 - ana32a23 - a2iai2a33 - a3ia22ai3 (tzv. Sarussovo pravidlo). Determinant singulární matice det A = 0, determinant regulární matice det A^O. • Inverzní maticí k regulární čtvercové matici A bude matice B = A-1, pokud platí A-B = B-A = 1. (2.14) • Hermiteovsky sdružená matice (značená obvykle A v lineární algebře, A^ případně A+ v kvantové mechanice) je označení pro matici komplexně sdruženou a transponovanou, AH = (A*)T. (2.15) Pokud A^ = AT, jedná se o matici reálnou. Pokud A^ = A, mluvíme o tzv. Hermite-ovské matici. • Unitární matice U je regulární čtvercová matice, jejíž hermiteovsky sdružená matice je současně maticí inverzní, tj. = U-1 a tedy UHU = U\JH = 1. (2.16) Reálná unitární matice = UT je tzv. maticí ortogonální, kdy její řádky, respektive sloupce, tvoří ortonormální soustavu vektorů (viz kapitola 2.2). Kapitola 2. Základy vektorové a tenzorové algebry 16 • Číslo A nazýváme vlastní hodnotou (vlastním číslem) a nenulový vektor v nazýváme (pravým) vlastním vektorem čtvercové matice A typu n x n, pokud je splněna podmínka Av = \v, (2.17) matice A tedy působí na vlastní vektor jako skalár, tj. nemění jeho směr (v případě tzv. levých vlastních vektorů bude mít podmínka (2.17) podobu vA = Xv). Z rovnice (2.17) přímo vyplývá relace pro určení vlastních hodnot matice A, kdy soustava n lineárních rovnic (A - AE) v = 0 A v Ý 0, tedy ^(a«j _ ^íj)vj = 0 Pro * = 1> 2, ... , n (2.18) má nenulové řešení právě tehdy, pokud je matice této soustavy singulární, tj. pokud det(A-AE)=0. (2.19) Vlastní vektory příslušné jednotlivým vlastním hodnotám potom určíme z rovnice (2.18). Pravé vlastní vektory budou mít tvar cr(vir,V2r, • • • ,vnr)T, levé vlastní vektory budou cl(vu, v2£, • • • > vni), kde cr a C£ jsou libovolné konstanty. • Submatici matice A obdržíme vynecháním vybraných řádků a/nebo sloupců v matici A. Determinant regulární čtvercové submatice se nazývá subdeterminant nebo také minor. • Příklady: 2.1 Jsou dány vektory a = (0,2,4), b = (1,3,5) a c = (6,1,3). Vypočítejte \a\, \b\, |c|, a x (b x c), (a x b) x c, (a + b) ■ (c — a), (b + č) x (a — b), (a-b)2 + (cx a)2. V2Ô, VŠE, V46, (-142,16,-8), (14,-6,-26), -8, (4,-1,-3), 1400 2.2 Vypočítejte obsah rovnoběžníku, jehož vrcholy tvoří body A[0, 0, 0], B[1, 2, 3], C a D[3, 2,1]. Dopočítejte souřadnice bodu C. 4V6, [4,4,4] 2.3 Body A[2,1, 0], B[2, 2, 3], C[0,1 + ^40, 0] tvoří vrcholy trojúhelníku. Pomocí vektorového součinu najděte jeho obsah. 10 2.4 Body A[4,1,0], B[4, -2,-3], C[l,-5,-3] tvoří vrcholy trojúhelníku. Určete velikosti vnitřních úhlů trojúhelníku a pomocí vektorového součinu vypočítejte jeho obsah. 2tt tt 9V3 6' Y' 6' ~T~ 2.5 Body A[2, —4, 9], B[— 1, —4, 5], C[6, —4, 6] tvoří vrcholy trojúhelníku. Pomocí vektorového součinu vypočítejte jeho obsah a určete velikost úhlu a. 25 TT Kapitola 2. Základy vektorové a tenzorové algebry 17 2.6 Jsou dány matice A 3 -2 -5 7 9 4 B Vypočítejte A • B, B • A. -12 48 37 -33 2.7 Vypočítejte inverzní matici k matici 1 0 -2 -2 1 CO 0 1 — 2 2.8 Jsou dány matice A = a matici D = B • A-1. 3 11 -10 CO 1 2 0 2 1 -1 CO CO 1 12 2.9 Vypočítejte determinant matice 2.10 Vypočítejte determinant matice 2.11 Vypočítejte determinant matice B 1 3 -23 15 (2 3 0 1 2 1 3 4 1 \1 2 2 í2 1 1 / 2 0 1 1 -10 2 0 4 -4 3 1 1 -1/ -3 3\ 3 -1 8 0 -1 2/ 2 11 1\ 5 6 3 4 5 7 5 3 5 7 13 10 3 8 13 V 7 2 11 6/ Vypočítejte inverzní matici A -i 294 120 n 2 2.12 Vypočítejte hodnost matice 0 5 -1 -1 3 1 2.13 Vypočítejte hodnost matice 0 1 - V 2 — 1 A ■ -1 A 2.14 Jsou dány matice A = 1 0 5 2 , B = V - -4 0/ matice A — BT — 3C, (3AT + B) C ,c2 1 2 3^ 1 4 9 I . Vypočítejte .1 2 4; Kapitola 2. Základy vektorové a tenzorové algebry 18 -2 -5 -18\ / 9 20 38 \ -4 -16 -31 , 10 68 165 . -4 -13 -12/ \25 74 147/ 1, -174, 2 298 348 60 \ / 71 216 443 \ 678 788 136 , 187 556 1121 , 334 391 68 / \ 87 260 527 / 2.2 Báze a jejich transformace Bázi vektorového prostoru V můžeme definovat jako množinu lineárně nezávislých vektorů, které tzv. generují vektorový prostor V, tj. kdy každý vektor z vektorového prostoru V lze vyjádřit jako lineární kombinaci těchto (bázových) vektorů. Významnou roli při praktických výpočtech hrají báze ortogonální, resp. ortonormální. Ortogonální báze je speciálním případem obecné báze, kdy různé bázové vektory jsou na sebe kolmé. Pro bázové vektory x, tedy platí 11 -^j i i ŕ 3 Xi ■ Xj = XíXj = 0, případně v algebraické notaci, [Xí,Xj) Ortonormální báze je speciálním případem ortogonální báze, kdy všechny bázové vektory (značíme je v tomto případě ě*j, často se také používá Xj) mají navíc jednotkovou velikost, 0. (2.20) případně, (2.21) 1 3 — l3' pj-ipo-u-iic, — i>3' Na příkladu obrázku 2.1 zkonstruujeme matice přechodu mezi bázemi a ukážeme princip reprezentace vektoru v různých bázích v R2 (v případě vyšší dimenze vektorového prostoru bude postup zcela analogický). Jsou zavedeny dvě báze £ a T, černá a červená, s bázovými vektory ě*2 a fi, f2, kdy černá báze je ortonormální, červená báze je zcela obecná. Přechod z černé báze £ do červené báze T je zde dán vztahy (popisujícími vektorový součet) fi = 2,5 ěi + 0,5 e2, /2 = 0,3 ei + e2. Můžeme tedy ihned napsat matici přechodu T z báze £ do báze T: 2 2 (2.22) T {£ ^ F) A 1 10 (2.23) / Matici přechodu (2.23, i všechny další) lze zapsat také pomocí sloupcového formalismu, tj. vektory /i) f2 budou zapsány jako sloupcové. V takovém případě bude „sloupcově" zapsaná matice přechodu násobit libovolný, rovněž sloupcový vektor (viz rovnice (2.29) v dalším výkladu), zapsaný za maticí. Ostatní dále popsané principy zůstanou nezměněné. Z rovnic (2.22) snadno odvodíme rovnice pro e±, ě*2, tedy rovnice opačného přechodu z červené báze T do černé báze £: ei e2 20 r _ 10 r 47 47 6 - 50-:Ji + -rzh- (2.24) 47J 47- V maticovém zápisu půjde o zpětnou matici přechodu S, která je inverzní vůči matici T, tedy S = T"1: S {F ^ £) 20 47 _6_ "47 47 50 47/ (2.25) Kapitola 2. Základy vektorové a tenzorové algebry 19 Obrázek 2.1: Schéma reprezentace vektoru v, vyznačeného zelenou barvou, ve dvou různých bázích v M2, v černě vyznačené ortonormální bázi s bázovými vektory e±, ě*2 a v červeně vyznačené obecné bázi s bázovými vektory /i, f2- Přechod z černé do červené báze je dán vztahy: /1 = 2,5 ě*i + 0,5 ě*2, J2 = 0,3ei + ě*2. V černě vyznačené bázi má vektor v složky o velikosti (2,2), velikost složek je zde znázorněna průmětem vektoru v do směrů jednotlivých vektorů báze, zvýrazněným černými čárkovanými čarami a znamená poměr velikosti těchto průmětů ku velikosti příslušných bázových vektorů. Totéž platí v červeně vyznačené bázi, kde velikost složek je dána průmětem vektoru v do směrů jednotlivých bázových vektorů, zvýrazněným červenými čárkovanými čarami. Velikost složek vektoru v v červené / 2g gQ \ bázi bude Í^,^J «(0,6; 1,7). Daný (zelený) vektor v má vodorovnou a svislou složku v černé bázi £ znázorněnou průmětem vektoru do vodorovné a svislé osy, tj. do směrů, které odpovídají bázovým vektorům ě*i, e2. Velikosti těchto složek budou odpovídat poměru délek těchto průmětů (vyznačených čárkovaně černě) a příslušných bázových vektorů, můžeme tedy vektor v zapsat jako vektorový součet (v tomto případě předem zvolených) násobků vektorů báze £, nebo pouze pomocí složek: {ľ=2ei + 2e2, nebo v = (2,2), (2.26) kdy ve druhém případě implicitně předpokládáme, že se „pohybujeme" v bázi £. Určení složek vektoru v v červené bázi T bude zcela obdobné. Průměty vektoru v do směrů bázových vektorů /i) f2 jsou znázorněné čárkovaně červeně, velikosti složek budou opět odpovídat poměru délek těchto průmětů a příslušných bázových vektorů. Uvědomímedi si, že vektor v je jen jeden a tedy vektorový součet jeho složek musí být stejný bez ohledu na to ze které báze se na něj „díváme", můžeme pro stanovení jeho složek v bázi T například nejprve analogicky k rovnici (2.26) obecně napsat v = a/i + bf2 = 2ěi + 2e2. (2.27) Dosazením za vektory e±, e2 z rovnice (2.24) dopočítáme velikosti složek a, b: - 28 - 80 - , _ f 28 80 \ v = —ň-\--f2, nebo v=[—,— ), (2.28) 47J1 47,/z' 147 47 / kdy ve druhém případě opět implicitně předpokládáme, že se „pohybujeme" v bázi T. Stejný výsledek dostaneme, vynásobíme-li vektor v, zapsaný pomocí jeho složek v černé bázi £, maticí Kapitola 2. Základy vektorové a tenzorové algebry 20 S přechodu z červené báze T do černé báze £: (a, b) = (2,2) 20 10\ 47 47 \- 6 50 47 47/ /28 80 V 47'47 (2.29) Zpětnou transformaci můžeme ověřit vynásobením takto získaných složek (a, b) vektoru v v bázi T maticí T přechodu z černé báze £ do červené báze J7, výsledkem musí být původní složky vektoru v v bázi £: 28 80 \ 47' 47/ 11 i\ 2 2 (2,2). (2.30) V případě ortonormálních bází budou matice přechodu mezi nimi maticemi pouze rotačními, tedy ortogonálními. Musí tedy platit: T = T a zároveň: detT = detT"1 = ±1 (pokud detT = detT-1 = —1, jedná se o tzv. nepravou (nevlastní) rotaci, tedy rotaci spojenou se zrcadlením v rovině kolmé k ose rotace). Další podrobnosti, týkající se vektorového a maticového počtu včetně počítání s bázemi - viz příslušné kursy lineární algebry. • Příklady: 2.15 Nalezněte matice přechodu mezi standardní ortonormální bází (kartézské souřadnicové soustavy) a bází cylindrické souřadnicové soustavy. 'cos 0 — sin (f. sin cos 9 — sin 9 0 2.17 Nalezněte matici přechodu při otočení dvourozměrné kartézké souřadnicové soustavy o úhel a (tzv. matici rotace). cos a sin ot \ (x 1 ■( = \ y'I \— srna cos a I \y 2.18 Nalezněte matici G Galileovy transformace časoprostoru. Galileova časoprostorová transformace je daná přiřazením (t,x,y, z)T \—> (ť, x', y', z')T , kde ť = t, x' = x — vxt, y' = y — vyt a z' = z — vzt, přičemž vektor v = (vx,vy,vz) interpretujeme jako rychlost. Vynásobením matic dokažte vztah Gu x Gv = Gu+V a vysvětlete proč se tento vztah nazývá klasickým pravidlem skládání rychlostí. Kapitola 2. Základy vektorové a tenzorové algebry 21 / 1 O O 0\ -vx 1 O O -Vy 0 10 \-vz 0 0 1/ \-vz 0 O 0\ 1 O O O 1 o 0 0 1/ 2.19 Vektor a má v ortonormální bázi B v je dán vztahy l2 složky (11/2, —1). Přechod mezi bázemi B a B' e{ = 4ě*i + e2, e2 = ~ 2e2- Určete matici T přechodu z báze B do báze B', matici S přechodu z báze B' do báze B a složky vektoru a v bázi £>'. Je báze B' ortonormální (uveďte důvod) ? Nakreslete obrázek, znázorňující velikost a směr všech uvedených vektorů, tj. vektorů obou bází i vektoru a. !\ 19 S í± 19 _3_ 19 ^1,1), báze B' není ortonormální. 19 / 2.20 Vektor a má v ortonormální bázi £> složky (1, 0, —2). Přechod mezi bázemi B a £>' je dán vztahy el = ~e{ + e2 ~~ e3'' e2 = e/ +2 eg, e3 = e/ + e2' + 2 e3'. Určete matici T přechodu z báze do báze B', matici S přechodu z báze B' do báze B a složky vektoru a v bázi B'. Je báze B' ortonormální (uveďte důvod) ? -3 -1 2 ■1 1 1 0 1 1 -3,-1,-5), báze B' není orto- normální. 2.21 Vektor a má v ortonormální bázi fí' složky (1, 2, —1). Přechod mezi bázemi B a £>' je dán vztahy ei e2 ě*3 ;3> e/ +2e3', e/ + e{ + 2 e^. Určete matici T přechodu z báze do báze B', matici S přechodu z báze B' do báze B a složky vektoru a v bázi fí. Je báze B ortonormální (uveďte důvod) ? 2 TO "2\ 1 -1\ 0 -1 1 , S= 1 0 2 1 -1 ll 1 2J (3, 2, —1), báze není ortonormální. 2.22 Vektor a má v ortonormální bázi fí' složky (1,1,1). Přechod mezi bázemi B a B' je dán vztahy 2 c*/ 2e2' ei e2 ě*3 = e/ + e2' + c3. Určete matici T přechodu z báze do báze B', matici S přechodu z báze B' do báze B a složky vektoru a v bázi fí. Je báze B ortonormální (uveďte důvod) ? Kapitola 2. Základy vektorové a tenzorové algebry 22 S (O, 2, 3), báze B není ortonormální. 2.23 Vektor a má ve standardní kartézské bázi £ složky (1, —y/Š, 1). Dále jsou zadány dvě báze B a B', přičemž matice R přechodu z báze £ do báze B' má tvar Přechod mezi bázemi B a £>' je dán vztahy /i/l ' 2 0\ 0 V o 01/ 1 2 2 ei ě*3 2e2' 2 e l - 2 e 2' + -e{ + 2e2' + 3e3'. Určete matici T přechodu z báze B do báze £>', matici S přechodu z báze £>' do báze B a složky vektoru a v bázích B & B'. Jsou báze B & B' ortonormální (uveďte důvod) ? (0,-2,1) 15,-2,11) 2.24 Vektor a má ve standardní kartézské bázi £ složky (1,1,1). Dále jsou zadány dvě báze B a B', přičemž matice R přechodu z báze £ do báze B' má tvar i o r 0 1 o 0 1 -1 0 1 Přechod z báze B' do báze B je dán vztahy R ei ě*3 e,' + e J 2 e/ + e + e9 Určete matici R-1 přechodu z báze fí' do báze £, matici T přechodu z báze £ do báze B, matici S přechodu z báze B do báze £ a složky vektoru a v bázích B a £>'. Jsou báze B a B' ortonormální (uveďte důvod) ? \B') = (2,1, 0), a(B) = (-3, 3, 4), 2.25 Vektor a má ve standardní kartézské bázi £ složky (1,1,1). Dále jsou zadány dvě báze B a B', přičemž matice R přechodu z báze £ do báze B' má tvar \ -2 3 2 , s = -1 1 2 / 1 2 -1 0 0 Kapitola 2. Základy vektorové a tenzorové algebry 23 Přechod z báze B' do báze B je dán vztahy ei = e2' + e3, e 2 = 2 e / + e 3' 3' e 3 = -e/ + e2' Určete matici T přechodu z báze £ do báze £>, matici S přechodu z báze £> do báze £ a složky vektoru a v bázích £> a B'. Jsou báze B a B' ortonormální (uveďte důvod) ? 0 -1\ í-l 1 2\ -2 -1 ,S = 1 -1 -1 , am = (-1,1,-1), a(B)= (-2,1,3), 1 0/ \-2 1 2/ £>' ano, B ne. 2.26 Vektor a má ve standardní kartézské bázi £ složky (1,1, 2). Dále jsou zadány dvě báze B a B', přičemž matice R přechodu z báze £ do báze B' má tvar R(£ i—>• #) Přechod z báze £>' do báze £> je dán vztahy e\ = — e 2' — 2 e3, e2 = 2 el ~ 2 62 + 63' e3 = e2' + 3e3'. Určete matici T přechodu z báze £ do báze £>, matici S přechodu z báze B do báze £ a složky vektoru a v bázích B & B'. Jsou báze B & B' ortonormální (uveďte důvod)? Je báze B' ortogonální (prokažte) ? S= -7 1 -5 , a(B) = (-9,2,-6), a(&) = (1,0,2), 2.3 Tenzorový počet Kromě skalárů a vektorů (tj. tenzorů nultého a prvního řádu) existují složitější algebraické struktury, tedy tenzory vyšších řádů. Z nich nejběžnější a nejjednodušší jsou tzv. tenzory druhého řádu, které obvykle popisují fyzikální pole s tzv. smykovými účinky (reprezentovanými nediagonálními prvky v příslušném tenzoru) v mechanice kontinua, například tenzor deformace, tenzor napětí, atd. Tak jako každý obecný vektor v je tvořen třemi skalárními složkami («1,^2,^3), je obecný tenzor 2. řádu T tvořen třemi „vektorovými" složkami (Ti,T2,T3), které lze obecně zapsat, (2.31) Kapitola 2. Základy vektorové a tenzorové algebry 24 kde každý ze tří vektorů T je zapsán pomocí tří složek. Zapíšeme-li tenzor T formou explicitního maticového zápisu, 'Tu T12 Tis\ T2i T22 T23 , (2.32) je každý ze tří vektorů T reprezentován jedním sloupcem matice (2.32). Analogicky ke způsobu zápisu vektoru pomocí složky a jednotkového bázového vektoru (v Einsteinově notaci) v = v^í můžeme tenzor zapsat jako T = Tretej nebo T = TjEi ® ej. (2.33) Symbol (g> značí tzv. tenzorový (dyadický) součin, tedy součin dvou vektorů stejné dimenze, kdy první z nich je sloupcový a druhý řádkový (jedná se tedy o součin matic typu 3 x 1 a 1 x 3 s výslednou maticí typu 3 x 3, na rozdíl od skalárního součinu, který můžeme obdobně vyjádřit jako součin řádkové a sloupcové matice typu Ix3a3xls výslednou maticí typu lxl, tedy skalárem). Dyadický součin je speciálním případem tzv. vnějšího součinu dvou vektorů, které nemusí mít stejnou dimenzi. Z rovnice (2.33) zároveň vyplývá, že prvek T j tenzoru T můžeme určit (tak jako složku ví vektoru v lze určit skalárním součinem, ví = v ■ ě*j) pomocí dvojitého skalárního součinu Tij = ei-T-ej. (2.34) Tenzorovým (dyadickým) součinem dvou vektorů v a w je tedy tenzor T, pro jehož prvek Tj platí Tj = víWj. (2.35) Tenzorový součin má následující vlastnosti: • v0w^w0v (2.36) • u (g> (av + (3w) = au (g> v + f3ů 0 w, (au + f3v) (g> w = au (g> w + f3v (g> w (2.37) • {u (8> v) ■ w = (v ■ w) u, u ■ (v (2> w) = {u ■ v) w (2.38) Jednotlivé dyády (dvojice), tedy tzv. b azove tenzory e%ej v rovnici (2.33) můžeme explicitně vyjádřit maticovým zápisem (v kartézské bázi), dei = U U U , eie2 = U U U , eie3 e2ei = 1 0 0 , e2e2 = 0 1 0 , e2e3 = 0 0 1 , (2.39) e3ei = u U U , e3e2 = u U U , e3e3 Ne každý tenzor 2. řádu můžeme obecně vyjádřit jako tenzorový součin dvou vektorů, každý tenzor 2. řádu můžeme ovšem napsat jako lineární kombinaci tenzorového součinu vektorů (obdobně jako jsme jej napsali pomocí tenzorového součinu jednotkových bázových vektorů v rovnici (2.33)). Kapitola 2. Základy vektorové a tenzorové algebry 25 • Kroneckerovo delta: Tzv. Kroneckerovo delta je matematická funkce, značená symbolem Síj, určená následujícím způsobem: 6ii = \1> ^ %=J- (2-40) 3 \0, pokud i ť^j. y ' Kroneckerovo delta lze zavést i jako tzv. Kroneckerův tenzor 2. řádu S'^ pro i, j = 1,2,3 (význam horních a spodních indexů je popsán níže v odstavci „Kovariantní a kontravariantní transformace, duální tenzor"), definovaný jako /l 0 0 4=0 1 o \0 0 1 Některé důležité vlastnosti funkce Kroneckerovo delta: • Ortonormalitu vektorů eí,ej můžeme vyjádřit jako e^ejSíj. • Su = 3, stopa Kroneckerova tenzoru S\ = 3. • Kroneckerovo delta Síj zaměňuje indexy složek vektorů nebo prvků tenzorů, například VíSíj=Vj nebo obecně Tjj ...k...zhl = Tj ...i...z. (2.42) • Redukce (úžení) dvou funkcí SijSjk s jedním společným indexem j na výslednou funkci Sik, úžení dvou funkcí se dvěma společnými indexy i, j na výslednou funkci Síj Síj = S a = 3. • Kroneckerovo delta Síj redukuje (úží) sumaci (tj. odstraňuje jednu sumu), kdy například y~l y~l Aijdij = ^ A-a-, ^ ^ AjkSjkSíj = ^ Ajjôij = Aíj. (2.43) i j i j k j m Antisymetrický (permutační nebo také Levi-Civitúv) symbol: Tzv. antisymetrický (také Levi-Civitúv - viz oddíl 2.1) symbol, značený e^-fc, je definován způsobem {+1, pokud ijk jsou sudé permutace, tedy ijk = 123, 231, 312, — 1, pokud ijk jsou liché permutace, tedy ijk = 132, 213, 321, (2.44) 0, pokud ijk jsou nulové permutace, tedy pokud i=jVj = kVk = i. Některé důležité vlastnosti Levi-Civitova symbolu eíjk- • Umožňuje stanovit výraz pro determinant obecné čtvercové regulární matice A libovolného řádu (je popsán v rovnici (2.13)). Například pro matici A řádu 3x3 potom dostáváme det A = ^2 £íjk Au Aj2 Ak3. i, j, k (2.45) Kapitola 2. Základy vektorové a tenzorové algebry 26 • Velmi užitečná při výpočtech (například vektorových identit nebo působení diferenciálních operátorů) je také souvislost mezi Levi-Civitovým symbolem Eijk a Kroneckerovou funkcí Síj. Z definice Levi-Civitova symbolu (2.44) jasně vyplývá, že společným působením dvou symbolů e^k a £/mn dostáváme identitu ^■ijk^lmn — ^il^jm^kn ^ím^jn^kl ^ín^jl^km ^il^jn^km ^ím^jl^kn ^ín^jm^kh (2.46) kterou můžeme kompaktním způsobem zapsat pomocí maticového formalismu ve tvaru ($il Sím $in \ fijl 3jm Ůjn I • (2.47) Ski Skm Skn) • Z rovnice (2.47) a ze zúžení Kroneckerových funkcí delta se společnými indexy dále vyplývá, že působení dvou Levi-Civitových symbolů e-ijt^Hm s jedním společným indexem k zjednoduší rovnici (2.46) do podoby ^■ijk^klm — fiilfijm $im$jli (2.48) • V případě dvou, případně všech tří společných indexů dostaneme EíjkEjkl = 2<5jZ, £ijk^ijk = 6. (2.49) • Levi-Civitův symbol můžeme definovat i pro n-rozměrný prostor, v tom případě bude obsahovat n různých indexů, přičemž sudé permutace budou vytvářeny sudým počtem číselných záměn, liché permutace lichým počtem číselných záměn (celkový počet nenulových permutací je n\). Například sudé permutace symbolu ve čtyřrozměrném prostoročase budou £0123) £0231) ^0312) £1032) £1320) ^1203) ^2130) ^2301) ^2013) ^3210) ^3102) £3021-Ostatních 12 permutací (bez opakování) bude tedy lichých. • Příklad působení diferenciálních operátorů na tenzorové pole: • Gradient (viz odstavec 5.3) tenzoru zvyšuje tzv. řád tenzoru, tj. například z vektoru (tenzoru 1. řádu) vytvoří tenzor 2. řádu, z tenzoru 2. řádu tenzor 3. řádu, atd. Gradient tenzoru T druhého řádu můžeme v kartézské souřadné soustavě s konstantními vektory báze (viz příloha A) obecně zapsat, - d(Tjkej ®ek) _ dTjk _ VT = J--®ei = -^-ej®ek®ei, (2.50 OXi OXi kdy dvojitý tenzorový součin vytvoří tenzor 3. řádu (s odpovídající reprezentací pomocí 3D matice typu 3x3x3, respektive „obyčejné" matice typu 9 x 3) s prvky Tjkí- • Divergence (viz odstavec 5.3) tenzoru snižuje řád tenzoru, tj. například z tenzoru 2. řádu vytvoří vektor, z vektoru skalár, atd. Divergenci tenzoru T druhého řádu můžeme v kartézské souřadné soustavě s konstantními vektory báze (viz příloha A) obecně zapsat, (2.51) Kapitola 2. Základy vektorové a tenzorové algebry 27 kdy explicitní tvar rovnice (2.51) bude v kartézském systému vypadat (viz také rovnice (A.17) v příloze A), V • T d d d dx' dy' d z _ 9AXX ^ ÔAXy ^ ÔAXZ ÔAyX ^ ÔAyy ^ ÔAyZ d Azx ^ ÔAZy ^ ô A z z ^ dx dy dz ' dx dy dz ' dx dy dz • Kovariantní a kontravariantní transformace, duální tenzor: Působením metrického tenzoru dané soustavy (viz například rovnice (A.4), (A.3), (A.36) a (A.63) v příloze A) transformujeme složky vektorových a tenzorových veličin mezi tzv. kovariantní a kontravariantní bází, které rozlišují kvantitativní chování dané geometrické nebo fyzikální entity při změně báze. Abychom zachovali velikost vektoru jako takovou, musí být složky vektorů (například polohy nebo rychlosti), jejichž rozměr je přímo úměrný měřítku báze, kontra-variantní vůči bázovým vektorům, zapisujeme je V = Vě*j. Naopak, složky tzv. duálních vektorů, nazývaných také kovektory (například vektor gradientu, který má rozměr prostorové derivace, respektive vzdálenosti-1), musí být ko-variantní vůči změně báze, zapisujeme je V = Ví e\ V zápisu se tedy formálně odlišují spodní nebo horní polohou indexů. V ortogonálních souřadných soustavách pro tzv. kovariantní metrický tenzor platí Qíj = híhjSíj (viz tzv. Laméovy koeficienty, rovnice (A.11)). Pro tzv. kontravariantní metrický tenzor glJ vždy platí gijg^ = 1, tedy g%3 = g^1. Obecné a explicitní vyjádření transformace vektoru Ví z kovariantní do kontravariantní báze lze tedy zapsat způsobem (viz Einsteinova sumační konvence): Vj = gľlV = 93lVx + r -r T° ré 1 r 1 r6 — L ér ~ L r -r sin2 9, TÉ sin 9 cos 0, T* = cotg#, (c) viz relace (A.45) v příloze A, (d) viz relace (A.71) v příloze A. 2.38 Jsou zadány kovariantní tenzor Aíj a kovariantní metrický tenzor Qíj dané souřadné soustavy, v pořadí souřadnicových směrů r, 9, 0, ve tvaru Ai au a\2 ai3 «21 «22 «23 y, «31 «32 «33y 9íj 0 0 r2sm29, Určete: (a) smíšený metrický tenzor g1- a smíšený tenzor Ap (b) kontravariantní tenzor A%3. íl 0 0\ / au laj g- 0 1 0 V0 0 1/ Q21 «31 «12 Q22 «32 «13 \ Q23 «33 (b) Aij au «12 «13 Q21 Q22 r4 «23 \r2gjn2 0 r2 sin2 r2sm26' «31 \ r2 sin2 0 Q32 r4 sin2 9 «33 \r2gjn2^ r4sin2ř? r4sin4#' 2.39 Ve čtyřrozměrném prostoru (prostoročase) jsou zadány kovariantní tenzor A^v a kovariantní metrický tenzor gap dané souřadné soustavy, v pořadí souřadnicových směrů A («00 «01 «02 «03 \ «10 «11 «12 «13 «20 «21 «22 «23 \«30 «31 «32 «33/ ga/3 (-1 o o 0\ 0 1 0 w 0 0 u2 0 V 0 w 0 u2J Určete: Kapitola 2. Základy vektorové a tenzorové algebry 31 (a) kontravariantní metrický tenzor gal3 a smíšený metrický tenzor g^, (b) smíšený tenzor A„, (c) kontravariantní tenzor A^v. laj g a/3 (b) At (c) A^ /"I 0 0 V° / o o o w o \ w 0 1 ur — uz -aoo 9p w (1 0 0 o\ 0 1 0 0 0 0 1 0 1° 0 0 v 013- -doi -ao2 "a03 U a±Q — W 030 u all ~ w a31 u a12 — w a32 u a13 ~ w a33 «20 «21 «22 «23 w aio - a30 W a±i — ci31 1« ai2 - ri32 w ai3 - a33 u a±o — w «20 w a10 - a30 v?aoi — wao3 u4an + i«2a33 — u2wS u2d2i — w023 u2a3i + ^2(íi3 — wT w2 — u2 (w2 — u2)2 u2(u2 — w2) (u2 — w2)2 ao2 U2CLi2 — w a32 «22 a32 - w a\2 u2 u2(u2 — w2) u2(u2 — w2) w a01 - a03 U2al3 + w<2a3l — wT a23 - w a2i a33 + w2an — w S u2 — w2 (u2 — w2)2 u2(u2 — w2) (w2 — u2)2 V kde ľ = u2an + 033 a S = 013 + 031. 2.40 Kovariantní tenzor elektromagnetického pole F^v je definován, dAv dAu F dx^ dxv ' kde tzv. čtyřpotenciál (čtyřvektor elektromagnetického potenciálu) A^ = y — , Složka Aq = — vyjadřuje škálovaný skalární potenciál elektrického pole a složky A±,A2, A% c tvoří tzv. vektorový (magnetický) potenciál. Kovariantní čtyřvektor souřadnic události zapíšeme jako x^ = (cí, —r). Metrický tenzor (Minkowského) plochého čtyřprostoru (prostoročasu) má tvar /l 0 0 0\ 0-100 0 0-10 \0 0 0 -1/ Vektory elektrické intenzity a magnetické indukce jsou definovány jako 8Ä 9p.v = 9 E -V0 dt B = V x A. Kapitola 2. Základy vektorové a tenzorové algebry 32 Napište: (a) explicitní podobu tenzoru F^v a tenzoru F^u, (b) duální tenzor *F^V a duální tenzor *F^V, (c) tzv. invarianty elektromagnetického pole F^VF^V a F^v *F^h (d) pomocí „čtyřrozměrné" divergence BF dxv odvoďte 1. pár Maxwellových rovnic, (e) pomocí „čtyřrozměrné" divergence dF^ dxv Q*pph dxv 0 kde j v je kontravariantní čtyřvektor proudové hustoty = (cp,j), odvoďte 2. pár Maxwellových rovnic. fa) F, [IV (b) *Ft c Ex c Ey c E, Bx By c 0 Bz —B,, B (c) F^F^ -Bx 0 Ez c _Ey c E2 -B. y _EZ c 0 Ex c B' Bz B 0 y Bx 0 / -Bz\ E. JL c Ex pav í o Ex c Er, k p ah c Ez c í 0 -Bx -Br,, o B -B y Bx 0 E* B? p *pt^ (d) í^A- = V • B = 0, dx° gppO dx'1 -V x E E-B dB dt 0, i = 1,2,3, Ey Ez\ c c -Bz By 0 -Bx Bx 0 / By Bz _E± Ey c c 0 Ex c Ex c 0 p dF^ dE , N_1/9 dx0 - ~^-E = --, -g— = p0e0 — -VxB = -fi0j, i = l,2,3, c= (fi0e0) x/2. 2.41 Kontravariantní tenzor energie-hybnosti Tal3 pro makroskopickou ideálni tekutinu je definován T a/3 P + p i u*/ — pg kde p je hustota, p je skalární tlak a je tzv. čtyřrychlost (čtyřvektor rychlosti), definovaná jako tečna k tzv. světočáře s, tedy = dx^/ds, kde s = ct. Tzv. vlastní Kapitola 2. Základy vektorové a tenzorové algebry 33 čas r v soustavě spojené s pohybujícím se tělesem je pomocí tzv. souřadnicového času t (tj. „normálního" času pozorovatele) definován jako t = 7T, kde tzv. Lorentzuv faktor 7 = (1 — v2/c2)-1/2 (viz také příklad 8.5). Čtyřvektor události x^ a metrický tenzor Minkowského prostoročasu g^v lze odvodit pomocí jejich definice v příkladu 2.40. Napište: (a) explicitní podobu tenzoru Tal3 a tenzoru Ta/3, (b) explicitní podobu těchto tenzorů v soustavě (0), spojené s pohybující se tekutinou. „2 „ „ „,2 7 Pomocí výrazů: W = (p + pv ), S = 7 ( —z + p) v, a T f P P aaß = -j ( -r + p) vaVß + p öaß, můžeme zapsat, ^2 {^+p) vav?+pöaP, '-paf} (w Sx Sy ( W Sx Sy Sz\ c c c c c c Sx Sx - ®xx ®xy ®xz — - ®xx ®xy ®xz c > Taß — c Sy c ®yx uyy ayz c &yx uyy ayz sz Sz 0zx azy c 0zx azy Czz 1 9 (b) T^(0) = Taß(0) 0 0 Vo p o o o o p o o\ o o p) Kapitola 3 Obyčejné diferenciální rovnice1 3.1 Obyčejné diferenciální rovnice 1. řádu Obyčejnými lineárními diferenciálními rovnicemi nazýváme rovnice, obsahující derivace funkce jedné nezávislé proměnné (zpravidla značíme x, závisle proměnnou obvykle značíme y(x)) v obecném tvaru + an^(x)y^ + ... + ai(x)y' + a0(x) = f(x), (3.1) kde značí fc-tou derivaci funkce y(x). Členy ak{x) jsou koeficienty, které mohou být funkcemi proměnné x, pokud jsou konstantní, mluvíme o diferenciální rovnici s konstantními koeficienty, a funkce f{x) představuje pravou stranu diferenciální rovnice. Pokud f{x) = 0, potom se jedná o homogenní diferenciální rovnici (rovnici bez pravé strany). Diferenciální rovnice s derivacemi funkcí více nezávislých proměnných nazýváme parciální diferenciální rovnice (viz příloha B). Rád diferenciální rovnice je dán nejvyšším řádem derivace závisle proměnné y{x), který se v rovnici vyskytuje, v případě rovnic 1. řádu půjde o 1. derivaci y' = dy/dx. 3.1.1 Rovnice separovatelné a homogenní Pokud lze diferenciální rovnici 1. řádu vyjádřit v jednoduchém tvaru y' = f{x), řešíme ji přímou integrací, tj. y = j f (x) dx. (3.2) Pokud lze diferenciální rovnici 1. řádu vyjádřit ve tvaru y' = f{x)g{y), kde g{y) 7^ 0, řešíme ji rozdělením funkcí s nezávisle proměnnou x a se závisle proměnnou y na různé strany rovnice {separacíproměnných), tedy fwrfnx)ix- M Funkci f{x, y) nazýváme homogenní, n-tého stupně, pokud pro všechna x, y a pro všechna z > 0, kde z je libovolný parametr, platí f{zx, zy) = znf{x,y). Obyčejná diferenciální rovnice 1. řádu M(x,y)dx + N(x,y)dy = 0 (3.4) 1Návody k řešení různých speciálních typů diferenciálních rovnic je možné nalézt v literatuře, např. v publikacích: Plch (2002), Bartsch (2008), Rektorys (2009). 35 Kapitola 3. Obyčejné diferenciální rovnice 36 je homogenní, pokud M(x, y) a N{x, y) jsou homogenní funkce stejného stupně. Pokud je homogenní rovnice zapsána v obecném tvaru (3.4), resp. ve tvaru y' = f(l), (3-5) řešíme ji vhodnou substitucí, například y = zx a převedeme na separovatelnou rovnici. Obdobně rovnici ve tvaru y' = f {ax + by + c) (3.6) převedeme na rovnici se separovatelnými proměnnými pomocí substituce z = ax + by + c. Rovnice ve tvaru racionální funkce , ax + by + c V = Ax + By + C (3-7) pokud výrazy ax+by, Ax + By nejsou lineárně závislé, řešíme eliminací absolutních členů c, C pomocí substituce u = x — xq, v = y — yo, kde xq, yo jsou kořeny soustavy rovnic ax+by + c = 0, Ax + By + C = 0 a následným převedením na tvar rovnice (3.5). Pokud výrazy ax + by, Ax + By jsou lineárně závislé (soustava nemá řešení), soustavu řešíme pomocí substituce ax + by = z (nebo Ax + By = z), která umožní její následné převedení na tvar rovnice (3.5). Řešení diferenciálních rovnic 1. řádu obsahují vždy jednu nezávislou (integrační) konstantu. Její hodnotu získáme z tzv. počáteční (Cauchyho) podmínky, kdy y(xp) = yp. • Příklady: 3.i -¥L= + ^== = o yr^2 = c_yrr y/l+y2 Vl + 2 1 3.2 (x2 - l)y3 - exy' = 0 y = 0 V —^ - (x + l)2 e~x = C, y / 0 2y 3.3 y' = 3^y, y(0) = 1 y = - 63 U ^ j, x < i log3 \ 3.4 x2(y3 + 5) dx + (x3 + 5)y2 dy = 0, y(0) = 1 (y3 + 5)(x3 + 5) = 30 7T 3.5 y' = cos(x — y), y(0) = — y = x + 2 arccotg (x + 1) + 2kir, k G Z 3.6 x2 fy'--) = xy y = x ln(C llnxl), x>0, C > 0, x / 1 \ mxy 3.7 y' = V2x + y - 3, y(0) = 3 2(V2x + y-3 + 2) - 4 ln(V2x + y - 3 + 2) = x - (4ln 2 - 4), 2x + y - 3 > 0 3.8 (x + y)2y' = 4, y(0) =2 y = 2 arctg - | + 2 3.9 y'= - fl+ln-) y = xeC:E, ->0 X V X/ X Kapitola 3. Obyčejné diferenciální rovnice 37 3.10 y' = - + tg^ x x y = 0 V sin - = Cx, x / 0, y / (2k + 1)- x, k G Z x 2 9 , 2 ln x + 1 ^ 3.11 xzy = xy + lnx y =--—--h Cx, x > 0 3.12 xy' = y + - y = Cxe"* V y = 0, x / 0 x 3.13 xV = ^2[y + ln(x2)] y = -2 (ln |x| + 1) + Cx, x / 0 3.14 y' = 1 + (x - y)2 y = x--j—, x / -C x + u 3.15 y' = (y + x) ln(y + x) — 1 y = eCeX — x, x G M e^+í/)2 / / 1 \ CN 3.16 y' = —— - 1 j, = ±^ln ^^-^J —x, x G (-oo, -). 3.17 y' = ey+x2 - 2x y = ln [(C - x)-1] — x2, x G (-oo, C). 3.18 y' = X + 2V-7 x + y_5 = C(x- 3)2, x / 3 x — 3 , 1 + 9x — 3y , , „ ^ 3.19 y'= —-- 3x-y 2 + 2x = C, y / 3x 3x — y 3.20 y' = 2X ~ V + f x2 + y2-xy + 3x-3y = C, 2y / x + 3 x — 2y + 3 3.21 y' = —— y = ±VC + 2x2 - x, 2x2 + C > 0 x + y 3.1.2 Nehomogenní rovnice Nehomogennílineární diferenciální rovnici 1. řádu (nazývanou také „rovnice s pravou stranou"), zapsanou v obecném tvaru y' + P(x)y = Q(x), (3.8) kde Q (x) 0, řešíme metodou variace konstanty. Nejprve řešíme homogenní rovnici ve tvaru y'+P{x)y = 0, kdy její integrační konstantou C bude obecná funkce nezávisle proměnné C(x), obecné řešení tedy bude y = C(x)e-fpWdx. (3.9) Funkci C{x) nalezneme dosazením rovnice (3.9) do rovnice (3.8)), její tvar bude CW = /«,)^^ + It (3.0) Kapitola 3. Obyčejné diferenciální rovnice 38 kde K je konstanta. Takto získaný výraz pro funkci C(x) dosadíme do rovnice (3.9), výsledné partikulární řešení bude y (3.11) • Příklady: 3.22 y' + 2xy = xe~x2 sinx, y(0) = 1 3.23 y' + y cos x = cos x 3.24 (1 + x2)y' - 2xy = (1 + x2)2 3.25 y' - 6xy = 4xe3x\ y(0) = 1 3.26 y' + 3x e3x = -y + 7, y(0) = 7 3.27 xy' + y = x sin x, y ( — ) =0 3.28 y' = 2x + 3y + 2, y(0) = 0 3.29 y' = 4x2 + 3y - 1, y(0) = 0 3.30 y' = 2x2 - y + 1, y(0) = 0 3.31 y' = 2x3 - y + 1, y'(0) = 0 3.32 y' 4x 1 , , y = (sinx — x cos x + 1) e x y = Ce-smx + 1 y = (x + C)(l + x2) y = (2x2 + 1) e3x2 . 3 3 ti = 7 +---x | e 1 16 4 x2 + 1 x2 + 1 3x 16 sin x — 1 9 (e3x - 1) cos x, x 0 2x i l-e 3x\ -x--X 3 9 27 y = 2x2 - 4x + 5 (1 - e~x) y = 2x3 - 6x2 - 11 + 12(x + e" 1 / q „ „n / o - \ — 2 xd + 3x + 3 [x1 + 1 3.1.3 Bernoulliova rovnice Bernoulliovou rovnicí nazýváme diferenciální rovnici 1. řádu s n-tou mocninou závisle proměnné y(x) ve tvaru y' + p(x) y + q(x) yn = 0, kde íiéI, (3.12) která má i přes svoji nelinearitu analytické řešení. Pokud n = 0, přejde Bernoulliova rovnice na nehomogenní lineární rovnici (3.8), pro n = 1 přejde na jednoduše separovatelnou homogenní lineární rovnici (3.3). Pomocí substituce z = y1~n pro n/0,1 (3.13) dostaneme nehomogenní lineární diferenciální rovnici 1. řádu typu (3.8) ve tvaru z' + (1 - n) p(x)z + (1 - n) q(x) = 0. (3.14) Kapitola 3. Obyčejné diferenciální rovnice 39 • Příklady: 3.33 y' = 6x2y3 3.34 xy' — y = —xy2 4 3.35 y' -\--y = x3y2 x 1 C-4x3 2x , y = O, 4x3 < C x2 + C , y = O, x2^ "C 1 x4(C — lnx -, y = O, x > O 3.36 y' + — — y/y = O x 1 f+ 7^= ) 3.37 y' + xy = xy3, y2{0) y2 = [e* +1 , y = 0 3.38 xy' + y = y2 lnx, y(l) = —1 3.39 2xyy' - y2 = x2, y(l) = O 3.40 x2y2y' + xy3 = 1, y(-2) = -1 1 1 + ln x — 2x , y = o 2 2 y — x = x , 2 3 3 2x 3.1.4 Rovnice exaktní Rovnici ve tvaru (3.4) nazýváme exaktní, pokud výraz na její levé straně je totálním diferenciálem nějaké skalární (tzv. kmenové) funkce F(x,y), tedy (podrobněji v kapitole 5.2) dF(x,y) = dFty) dx+9Fty) dy. dx dy (3.15) Funkce M(x,y), N(x,y) z rovnice (3.4) odpovídají jednotlivým parciálním derivacím (tj. složkám gradientu - viz kapitola 5.1) skalární funkce funkce F(x,y) v pořadí podle rovnice (3.15). Pokud jsou obě funkce M(x,y), N(x,y) spojitě diferencovatelné, musí podle Schwarzovy věty o rovnosti smíšených derivací platit, dM(x,y) dN(x,y) dy dx (3.16) K vyřešení rovnice (3.4) je třeba najít kmenovou funkci F(x,y), jejíž obecné řešení zpravidla zapíšeme ve tvaru F(x, y) = C. Pokud rovnice (3.16) neplatí, rovnice (3.4) není rovnicí exaktní. Pokud však nalezneme funkci R(x,y) (tzv. integrační faktor) takovou, že platí R{x, y) [M(x, y) dx + N{x, y) dy] = 0, (3.17) rovnice (3.17) bude rovnicí exaktní. Pro spojitě diferencovatelné funkce M(x, y) ^ 0, N{x, y) ^ 0 takový integrační faktor R{x, y) existuje. Kapitola 3. Obyčejné diferenciální rovnice 40 • Příklady: 3.41 2xy - 9x2 + (2y + x2 + l) y' = 0 y2 + (x2 + l) y - 3x3 = C 3.42 (2xy + 6x) dx + (x2 + 4y3) dy = 0 x2y + 3x2 + y4 = C 3.43 (8y - x2y) y' + x - xy2 = 0 X-x2 (1 - y2) + 4y2 = c 3.44 (e4x + 2xy2) dx + (cos y + 2x2y) dy = 0 1 Ar , 2 2 i — e + x y + srn y = = C 3.45 (3x2 + y cos x) dx + (sin x — 4y3) dy = 0 x3 + y sin x — y4 = C 3.46 x arctg y dx H----?r- dy = 0 &y 2(1 +y2) y x2 — arctg y = C 3.47 (2x + x2y3) dx + (x3y2 + 4y3) dy = 0 x2+ l +ž/4 = C 3.48 (2x3 - 3x2y + y3) y' = 2x3 - 6x2y + 3xy2 x4 3 —--2x3y + -x2y2 - ŕ _ 4 3.49 (y2 cos x — sin x) dx + (2y sin x + 2) dy = 0 y2 sin x + cos x + 2y = = C 3.50 2xy2 dx + (3x2y + 4) dy = 0 x2y3 + 2y2 = C 3.51 (2y + 4x2y2) dx + (x + 2yx3) dy = 0 x2y + x4y2 = C 3.52 2xydx + (y2 - 3x2) dy = 0 r2 1 = c yá y 3.2 Obyčejné diferenciální rovnice 2. řádu 3.2.1 Rovnice s konstantními koeficienty Obyčejnou diferenciální rovnici 2. řádu (tj. obsahující 2. derivaci závisle proměnné y (x)) s konstantními koeficienty řešíme v prvním kroku jako rovnici homogenní, kdy rovnici ve tvaru y" + py' + qy = 0, (3.18) řešíme pomocí tzv. charakteristické rovnice A2 + pA + q = 0. Pro kořeny charakteristické rovnice Ai, A2 £ M bude mít rovnice (3.18) řešení ve tvaru y = C1eXlX + C2eX2X proAi/A2, (3.19) y = CľeXlX + C2xeÁ2X proAi = A2. (3.20) Pro kořeny charakteristické rovnice Ai, A2 = a±/3i £ C bude mít rovnice (3.18) řešení ve tvaru y = Ci e^-^ x + C2 e(a+^ x = A eax cos /3x + B eax sin /3x. (3.21) Kapitola 3. Obyčejné diferenciální rovnice 41 Uvedené řešení lze zobecnit i pro diferenciální rovnice vyšších řádů: pro každý kořen charakteristické rovnice n-tého řádu Ai, A2, • • • , An G M s násobností II bude mít rovnice (3.18) n řešení ve tvaru y = d eXlX + C2x eX2X + ... + Cn x11'1 eXnX (3.22) a pro každou dvojici kořenů charakteristické rovnice n-tého řádu Ai, A2 = a±/3 i G C s násobností II bude mít rovnice (3.18) n řešení ve tvaru y = eax cos /3x (Ai + A2x + • • • + Aux11'1) + eax sin /3x (Bi + B2x + • • • + Bux11'1) . (3.23) Posloupnost lineárně nezávislých členů yi{x),..., yn{x) v řešení homogenní rovnice představuje tzv. fundamentální systém. Analogicky ke způsobu, popsanému v odstavci (3.1.2), můžeme hledat partikulární řešení nehomogenní rovnice (rovnice s pravou stranou) ve tvaru y" + py' + qy = R{x) (3.24) metodou variace konstant, kdy rovnice (3.19), (3.20), resp. (3.21) napíšeme jako obecné řešení diferenciální rovnice, tedy y = Ci(x) eAl* + C2{x) ex*x = Cxux + C2u2. (3.25) Funkce C±(x), C2{x), eXlX, eX2X, které pro jednoduchost budeme dále psát jako C±, C2, u±, u2, opět nalezneme dosazením rovnice (3.25) do rovnice (3.24). Dostaneme tak jednu rovnici pro dvě neznámé funkce C±, C2, R(x) = (ClUl + C2u2)" + p (ClUl + C2u2)' + q (ClUl + C2u2) (3.26) = Ci (u'l + pu[ + qUl) + C2 (v% + pu'2 + qu2) + (C"m + 2C[u[ + C2u2 + 2C2u'2) + p {C[Ul + C'2u2) , kde první dva závorkované členy (násobené nederivovanými funkcemi C±, C2) představují homogenní rovnice (3.18). Druhou rovnici vytvoříme tak, že stanovíme např. tuto podmínku: Cíui + C'2u2 = 0. (3.27) Protože derivace rovnice (3.27) musí být také nulová, dosazením do rovnice (3.26) dostáváme výsledný systém dvou rovnic pro dvě neznámé funkce C±, C2, C[Ul + C'2u2 = 0, C[u[ + C2u'2 = R(x). (3.28) Zapíšeme-li systém rovnic (3.28) pomocí tzv. Wronského matice, tj. ve tvaru (u\ u2\ fC[\ ( 0 \ / r,,i I \ rv C'J \R(x) (3.29) jejíž determinant uiu'2 — (tzv. wronskián) značíme W, snadno nalezneme řešení systému rovnic (3.28), zapsané například jako f u2R(x) f UlR(x) Ci = - / dx, C2 = / dx. (3.30) Kapitola 3. Obyčejné diferenciální rovnice 42 Dosazením rovnice (3.30) do obecného řešení (3.25) dostaneme partikulární řešení obyčejné diferenciální rovnice 2. řádu. V případě obyčejné diferenciální rovnice obecného (n-tého) řádu přejde rovnice (3.29) do podoby: Ui U2 \ (n-1) (rc-1) ,(n-l) íC'l\ C \cj í 0 \ o \R{x)J (3.31) V případě, že pravá strana R{x) nehomogenní rovnice bude mít formu (tzv. speciální pravá strana) obecně zapsanou jako R{x) = [Pn{x) cos f3x + Qn{x) sin f3x] eax, (3.32) kde Pn a Qn jsou polynomy nejvýše n-tého stupně (n je rovno vyššímu stupni obou polynomů P, Q), bývá často jednodušší nalézt řešení diferenciální rovnice tzv. metodou neurčitých koeficientů. Při hledání partikulárního řešení vyjdeme (bez ohledu na hodnoty koeficientů a a /3, které mohou být i nulové, případně bez ohledu na to, jestli jeden z polynomů Pn, Qn je nulový) z rovnice [{Apxn + Bpx12-1 + ... + Cp) cos f3x + {Aqxn + B,/"1 + ... + Cq) sin f3x] x11 eax, (3.33) kde II je násobnost kořene A = a + /3i charakteristické rovnice (kde opět a, (3 mohou být nulové). Rovnici (3.32) dosadíme do rovnice (3.24) a obecné koeficienty Ap,..., Cp, Aq,..., Cq porovnáme s koeficienty funkce R(x), danými rovnicí (3.32). Partikulární řešení diferenciálních rovnic 2. řádu obsahují vždy dvě nezávislé konstanty. Jejich hodnoty získáme řešením tzv. okrajové úlohy, zadané formou okrajových podmínek, kdy pro dvě různá x±,X2 platí y{xi) = y±, y{x2) = í/2 {Dirichletovy okrajové podmínky) nebo y'{xi) = yi, y'{x2) = 2/2 {Neumannovy okrajové podmínky), případně y{x\) = y±, y'{x2) = y2, v tomto případě x±,X2 mohou být různá nebo stejná (tzv. Newtonovy okrajové podmínky). • Příklady: 3.53 y"-2y' + y = -, y(l) = 0, y'(l) = 0 3.54 y" - 7y' + 12y = 5 ,3x 3.55 y" - 3y' + 2y 1 +e2x 1 y = ex + xex(lnx — 1), x > 0 5 y = Ci e-ix + C2 e4x + 12 y = Ci ex + C2 e2:E - - ex ln(l + 2x) + e2x arctg (e2) 3.56 y" +y= —!—, y /?ľ sin x V 2 7T ( — — x ) cos x + sin x (1 + ln sin x), x G (2k, 2k + 1)tv, k G 3.57 y" - 2y' = x2 - x y = Ci + C2e 2x Kapitola 3. Obyčejné diferenciální rovnice 43 3.58 y" + y' l+ex y(0) = 1, y'(0) = 0 y = l+x + (\ +e-x)\n 1 +ex 3.59 y" + y = cosx, y(0) = 1, y (|) = 1 + ^ 4 10 3.60 y" - 2y' + 5y = e^cosi, y(0) = -, y'(0) = — y cos x + sin x 1 H— 2 cos 2x + sin 2x + — cos x j ex 3.61 y" — 6y' + 9y = Axe3x cosx, y(0) = 1, y'(0) =0 y = (1 — 7x + 8 sin x — 4xcosx) e 3x 3.62 y" + y' - 6y = 12x2 + 2x + 1 y 3.63 y" + y' - 6y = 12x2 - 2x + 1, y(0) = 1, y'(0) = 0 y 3.64 y" + 4y' + 4y = e-2x ln x y = de-2x + C2xe-2x + ^e-2x(2lnx - 3), x > 0 3.65 y" + 4y' + 4y = e-2x ln2 x x2 / 7 y = Ci e~2:E + C2x e_2:E + ^-e_2:E í ln2 x - 3 ln x + - Ci e-3x + C2 e2x - 2x2 - x - 1 31 -3x , 6 2x o 2 X 8 — e dx + - ezx - 2xz---- 45 5 3 9 x > 0 3.66 y" - Ay' + 5y = 0, y(0) = 1, y'(0) = 0 y = e (cos x — 2 sin x) 3.67 y" - 2y' + 5y = 0, y(0) = 1, y'(0) = 0 ex cos 2x--sin 2x 2 3.68 y" - 8y' + 32y = 0, y (|) = 1, y' = 0 y = e4x 27r (cos 4x — sin 4x) 3.69 y" - 3y' + 2y = (x4 + 1) ex ^.5 y=[c x4 - 4x3 - 12x2 - 25x ex + C2 e 2x 3.70 y" - Ay' + 5y = (x2 + 2x) e2x cos x y = e 2x Ci + — + - ) cos x + ( C2 H---h 42/ V 6 2 4 S 2 X° X X smx 3.71 y" - 3y' + 2y = (1 - 2x)ex, y(0) = 1, y'(0) = 0 y = 3ex - 2e2x + (x2 + x) e 3.72 y" - 2y' + y = (x + 1) e*, y(0) = 1, y'(0) = 0 y = e S 2 X X + — -x + 1 3.73 y" + 4y' + 4y = (6x + 2) e"2*, y(0) = 1, y'(0) = 0 y 3.74 y" + Ay' + Ay = e-2x sin x, y(0) = 1, y'(0) = 0 y 6 2 e-2x ^x3 + x2 +2x + l e~2x(3x — sinx + 1) 3.75 y" - 2y' + 2y = ex sinx, y(0) = 1, y'(0) = 0 y [(2 — x) cos x — sin x] Kapitola 3. Obyčejné diferenciální rovnice 44 3.76 y" - 2y' + 2y = x2 + x + ex sinx, y(0) = 2, y'(0) = 3 y = e 1--cos x + sin x 2 x2 3x + T + T + 1 3.2.2 Rovnice s nekonstantními koeficienty Obyčejné diferenciální rovnice 2. řádu typu rovnice (3.24), kde koeficient p = p (x) a kde koeficient q = q (x) = 0, řešíme jejich převedením na rovnice 1. řádu závisle proměnné z = y'. Rovnice typu y" + p{x)y' + q(x)y = R(x), (3.34) řešíme tak, že hledáme nějakou funkci I{x) (integrační faktor) takovou, že pro z = I{x)y rovnice (3.34) přejde do podoby rovnice s konstantními koeficienty z +pz + qz = R(x). Obyčejné diferenciální rovnice 2. řádu typu y" + p(y')myn + ^yr = RW (3.35) (3.36) kde m, n, r, s jsou konstanty, lze řešit nalezením takového z = f(y), pro které opět platí rovnice (3.35). • Příklady: 11 3-77 y" - ^ = x2 + 1, y(l) = y'(l) = 1 y 3.78 xy" + (x + 2)y' + y = 0 3.79 xy" - (3x - 2)y' + (2x - 3)y = 0 A 3 2 tie tlC tlC —i-----1, x^O 4 3 2 ' ^ y = -(C1 + C2e-x), y = 0, x/O ž/=-(C1e- + C2e2-), y = 0, x/0 3.80 x2y" - 2x(x + 2)y' + (x2 + 4x + 6)y = 0 y = e* (Cix2 + C2x3) , y = 0 3.81 x2y" + x(x + A)y' + (x2 + 2x + 2)y = 0 y e_f / a/Š \/3 \ —2~ Ci cos —x + C2 sin —x , y = 0 3.82 x V - 4xy' + Ay = x + 1 3.83 x2y" - xy' + y = 2x - A 3.84 y" -(j^-±-)y' + -y = 2^ \ a/x 2x / x ^ „a xlnx 1 y = CiX + C2X4--— + -, X > 0 y = C±x + C2x ln x + x ln2 x — 4, x > 0 ^(9/7 A K 2 9^ 21a/x 45 Ci e2^ + C2 e4^ +12 H---1-----1--, x > 0 32 Kapitola 3. Obyčejné diferenciální rovnice 45 3.85 y" + -^^y' + —íV = O y = Ci e"2/* + C2 e"4/*, y = O, x / O xz x4 3-86 lě^"" (lěb + \) ^ + AV = + 1 y = Cie4x2 +C2^2e4^ + x4+x2 + jj, x/O 3.87 y" - y' = 2x(2x - l)2 y = Ci + C2(x2-x) + -^x5 - Jx4 + ^, x / ± 2x — 1 15 o 3 2 3.88 2yy" + 2y'(y' - 4y) + 4y2 = x y = ±i VCi e2* + C2x e2* + x + 1 = D > 0 3.89 y~~ (f^" + 1) =2_e:EV/y V = e~2:E(Cicosx + C2sinx + ir2, y>0 3.3 Soustavy obyčejných diferenciálních rovnic Podobně jako „běžné" rovnice, mohou i obyčejné (lineární) diferenciální rovnice tvořit soustavu. Uvažujme systém lineárních diferenciálních rovnic pouze 1. řádu (rovnice vyššího řádu lze na takový systém vždy jednoduše převést, například rovnici 2. řádu y" + a±y' + a^y = f zapíšeme jako dvě rovnice 1. řádu: y' = z, z' = —a±z — aoy + /) y'2 aii(x)yi + ai2(x)y2 + a2i(x)y1 + a22(x)y2 + + ain(x)yn + /i(x), + a2n(x)yn + /2(x), (3.37) y'n = ani(x)y1 + an2(x)y2 H-----h ann(x)yn + /„(x). Systém rovnic (3.37) zapíšeme vektorově jako y' = A y + / (nebo, pokud f{x) = 0, jako homogenní systém y' = Ay), kde matice A(x) an{x) a12{x) a2i(x) a22(x) ani{x) an2{x) aln(x) a2n(x) (3.38) (3.39) a kde y', y a / jsou sloupcové vektory. Řešení soustav rovnic s nekonstantními koeficienty A(x) může být v praxi značně komplikované a představuje samostatnou disciplínu, vymykající se rozsahu těchto skript, v následujících odstavcích se proto zaměříme pouze na systémy rovnic s konstantními koeficienty A(x) = A. 3.3.1 Homogenní soustavy s konstantními koeficienty V případě homogenního systému dle rovnice (3.38) s konstantními koeficienty a^-, kdy matice A (typu n x n) má n různých reálných vlastních hodnot Aj, i = 1.. .n (viz rovnice (2.17)), můžeme zapsat řešení v obecném vektorovém tvaru y(x) = CieAl^i + C2eA2^2 + • • • + Cnex-Xvn, (3.40) Kapitola 3. Obyčejné diferenciální rovnice 46 kde ví jsou jednotlivé vlastní vektory dle rovnic (2.17) a (2.18), příslušející vlastním hodnotám Xí (k rovnici (3.40) bychom dospěli i například postupným dosazováním, tedy náhradou n rovnic 1. řádu jednou rovnicí n-tého řádu, zejména v případě vyššího n je to ovšem způsob značně obtížný a pracný). Jako jednoduchý příklad uvedeme systém dvou homogenních rovnic y'A = A 6\ fyi y'J \1 2 \y2 (3.41) Vlastní hodnoty matice A budou Ai, A2 = —1,4, příslušné vlastní vektory budou v\ = (—3,1)T a v2 = (2,1)T. Z kapitoly 2.1 je zřejmé, že vlastními vektory jsou i všechny vektory v\, v2, násobené libovolnou konstantou (v následujícím textu budeme uvádět pouze jeho základní tvar). Výsledné řešení systému rovnic, v případě že nejsou zadány další podmínky, můžeme zapsat jako y{x) = Ci (~f) e~* + C2 (*) e4*. (3.42) ij V1, Pokud jsou vlastní hodnoty matice A reprezentovány také dvojicemi (komplexně sdružených) komplexních čísel, budeme řešení hledat obdobným způsobem jako v případě reálných vlastních hodnot. Jako jednoduchý příklad uvedeme systém dvou homogenních rovnic y'A = í2 ~5\ y'J \l -2) \y2 (3.43) Vlastní hodnoty matice A v tomto případě budou Ai, A2 = ±i, příslušné vlastní vektory budou vi = (2 — i, 1)T a V2 = (2 + i, 1)T. Výsledné řešení systému rovnic bude m = C1(j )e- + C2(j )e-=A(0 5c°SX. )+B(0.5s'mX V (3.44) yy ' \2-iJ \2+iJ \2 cos x + srn xj \2sim - cosxj ' v ; kde vztah mezi exponenciální a goniometrickou formou rovnice (3.44) je dán Eulerovou identitou e±ĽE = cosx ± isinx a kde koeficienty A = Ci + C2, B = i (C2 — Ci). Pokud je některá reálná vlastní hodnota matice A vícenásobná, způsob řešení bude dále záležet na počtu jí odpovídajících vlastních vektorů, kdy existují v zásadě 2 možnosti: (a) Vícenásobné (fc-násobné) vlastní hodnotě p odpovídá k lineárně nezávislých vlastních vektorů, potom část obecného řešení, týkající se této vlastní hodnoty, bude mít tvar yp(x) = Cie^ŤTi + C2e"xv2 + ••• + Gkďxvk. (3.45) Jednoduchým příkladem může být například následující systém, y'A = í3 0>i íyi vU V° V \yz (3.46) s dvojnásobnou vlastní hodnotou p = 3 a se dvěma lineárně nezávislými vlastními vektory vi = (1, 0)T a v2 = (0,1)T. Výsledné řešení systému ve smyslu rovnice (3.45) bude yp(x) = C1Qe^ + C2(j)e^. (3.47) Kapitola 3. Obyčejné diferenciální rovnice 47 (b) Vícenásobné (fc-násobné) vlastní hodnotě p odpovídá j lineárně nezávislých vlastních vektorů, kdy 1 < j < k, tedy s = k — j vlastním hodnotám p odpovídá jediný lineárně nezávislý vlastní vektor u. Taková matice se nazývá defektní a není diagonalizovatelná, tj. převoditelná na diagonálni matici po vynásobení zleva maticí řádkových levých vlastních vektorů a zprava maticí sloupcových pravých vlastních vektorů. Potom část obecného řešení, týkající se tohoto vlastního vektoru u, bude mít tvar yp{x) = duepx + C2 {wi + xu) epx + • • • í ^ ----h Cs í Ws-i + XWs-2 + "27^-3 H-----1" / _ 2n, „s-1 Wi + kde vektor Wi odpovídá libovolnému řešení algebraických rovnic (A — pE) Wi = Wi-i, ... (A — pE) wi = Ú. Následující příklad ilustruje popsaný princip řešení: uvažujme systém u e- px (3.48) (3.49) (3.50) (3.51) s dvojnásobnou vlastní hodnotou p = 2, které ovšem odpovídá pouze jeden lineárně nezávislý vlastní vektor u = (1,1)T. Vektor w určíme z rovnice (3.50), Wi w2 tedy Wi w2 Výsledné řešení systému ve smyslu rovnice (3.48) bude Ci (;) e2x + C2 o) +x(i J2x (3.52) (3.53) Řešení systémů s více než dvěma lineárními rovnicemi 1. řádu je analogické k uvedeným jednoduchým příkladům se dvěma rovnicemi, některé principy více ozřejmí následující příklady, zahrnující i systémy tří rovnic. • Příklady: 3.90 y'=(\ l^)y y = Ci (")efe + C2 3.91 y' 7 13 -1 1 13cos2x \ / 13sin2x 3 cos 2x — 2 sin 2x1 \2 cos 2x — 3 sin 2x -Ax 3.92 y' 3.93 y' 1 -1 0 5 4 -4 1 -4 f-2 1 + c3 0 0/ ^ v 1 -3x Kapitola 3. Obyčejné diferenciální rovnice 48 3.3.2 Nehomogenní soustavy s konstantními koeficienty Řešení lineárních soustav s pravou stranou bude v principu analogické metodám řešení obyčejných diferenciálních rovnic 2. řádu (viz odstavec 3.2.1), tj. metodám variace konstant a neurčitých koeficientů. Metodu variace konstant můžeme aplikovat následujícím způsobem: předpokládejme partikulární řešení nehomogenní rovnice (3.38), ve tvaru yp = Y(x)t(x), (3.54) kde Y(x) je matice, jejíž sloupce tvoří jednotlivá lineárně nezávislá řešení příslušné homogenní rovnice (3.38), přepsané nyní do tvaru Y' = AY, t(x) je hledaný, obecně zapsaný sloupcový vektor. Protože rovnice (3.38) musí platit také pro partikulární řešení, tedy y' = Ayp + /, první derivace partikulárního řešení v takovém případě bude yp' = Y't + Yt' = AYt + / = Y't + f, tedy Yt' = /. (3.55) Hledaný vektor t získáme integrací rovnice (3.55), t = j Y^fdx, yp = Y J Y_1/dx. (3.56) Uvedenou metodu ilustruje následující řešený příklad: použijeme homogenní systém z řešeného příkladu (viz rovnice (3.41)) s přidanou pravou stranou, (3.57) Kapitola 3. Obyčejné diferenciální rovnice 49 Z rovnice (3.42) ihned vidíme, že matice Y(x) bude -3e" 2é Ax -Ax I ) z toho Y -i 5 Ve 2ex .-Ax 3e -Ax (3.58) Podle rovnice (3.56) tak dostáváme partikulární řešení pomocí integrace (kdy každou složku vektoru yp integrujeme zvlášť) 1 -3e~ 2é Ax -Ax 2ex -Ax 3e -Ax 3x dx -3/4 (1 -4x)/8 (3.59) Úplné řešení tedy bude součtem rovnic (3.42) a (3.59). Ve výsledku rovnice (3.59) neuvádíme integrační konstantu, předpokládáme, že je již „skrytá" v konstantách homogenního řešení v rovnici (3.42). Metoda neurčitých koeficientů pro systém rovnic je zcela analogická již uvedenému řešení pro rovnice 2. řádu, jediný rozdíl spočívá v tom, že koeficienty nyní budou vektory. Pokud například v odstavci 3.2.1 byla pravá strana rovnice polynomem 1. stupně, obený zápis partikulárního řešení měl tvar yp = Ax + B, nyní to bude yp = Ax + B. Metodu ukážeme na stejném řešeném příkladě: předpokládejme uvedenou obecnou formu partikulárního řešení, tedy A2 +\B2 což dává y' = A Ax A2 (3.60) Rovnice (3.38) musí opět platit i pro partikulární řešení, y' = Ayp + f, tedy A = A(Ax+B) + f. Přepíšeme-li (vektorový) polynom 1. stupně do následujícího explicitního tvaru + x + A A2 0, (3.61) dostáváme pro lineární i absolutní člen (oba musí být nulové) následující rovnice: 1 6 1 2 Ai A2 1 6 1 2 B2 Ai A2 Nyní již snadno dopočítáme jednotlivé neurčité koeficienty, dostáváme A\ = 0, A2 = B\ = —3/4, B2 = 1/8, což po dosazení a úpravě dává shodný výsledek s rovnicí (3.59). (3.62) -1/2, • Příklady: 3.97 y' \ -ij^+ll'6 -5x ž/ = Ci l")e2* + C -2x U/21) 3-98 y'= u "1)^(1 y = ci (; )e2- + C2 o)+x[i J2x 1/2 1/2 Kapitola 3. Obyčejné diferenciální rovnice 50 3.99 y' 3.100 y' J2x Kapitola 4 Křivkový integrál1 4.1 Křivkový integrál 1. druhu Křivkovým integrálem 1. druhu nazýváme integrál f ds obecné skalární funkce f (x, y, z) Jc podél křivky C, kde ds je délkový element křivky: ds2 = dx2 + dy2 + dz2 (Pythagorova věta v diferenciálním tvaru). Stanovíme-li např. souřadnici x jako nezávisle proměnnou a y (x), z (x) jako závisle proměnné, můžeme psát fcf=£n™'Ý+{£) +(£) «*■ «-i Nalezneme-li vhodný parametr t, potom bude parametrizovaná rovnice (4.1), kde f(t) = f[x(t),y(t),z(t)], s(t) = s[x(t),y(t),z(t)], mít tvar ^■rw(s),+(i),+(sv Pomocí křivkového integrálu 1. druhu lze určit geometrické a fyzikální charakteristiky dané křivky. Položíme-li / = 1, výsledkem bude délka křivky C. Položíme-li / = r (délková hustota křivky), dostáváme rds = dm, tedy element hmotnosti křivky, výsledkem integrace bude hmotnost křivky C, m = I dm = / rds. (4.3) JC Jc Pokud položíme například / = zt, dostáváme tzv. statický moment Sz křivky vzhledem k ose z, jeho vydělením hmotností dostáváme z-ovou souřadnici středu hmotnosti zt křivky C (obdobně pro ostatní souřadnicové směry), tedy xx = — / xdm = — / xrds, Vt = — / y dm, z? = — / z dm. (4.4) m J c m J c m Jc m Jc Položíme-li / = r2r, kde r je vzdálenost obecného bodu křivky od zvolené přímky v prostoru (osy o), dostáváme moment setrvačnosti JQ křivky C vzhledem k této ose. Momenty setrvačnosti křivky C např. vzhledem k jednotlivým kartézským souřadnicovým osám potom budou Jx= (V2 + z2) dm= (y2 + z2) r ds, Jy = / (z2 + x2) dm, Jz = (x2 + y2) dm. J C J C J C J c (4.5) 1Ve výsledcích příkladů s geometrickými nebo fyzikálními veličinami nejsou uváděny příslušné jednotky. 51 Kapitola 4- Křivkový integrál 52 • Příklady: 4.1 Vypočítejte délku křivky s = / (x2 + y2) ds, kde C je křivka s parametrizací x = a(cos t+ Jc ísiní), y = a(siní — t cosi), i £ (0, 27r). s = a3 (27T2 + 4tt4) 4.2 Vypočítejte délku úseku paraboly y = x2, ohraničeného body (—2,4) a (2,4). s = 2VY7 + ln yjA + Vvf « 9, 2936 4.3 Kolikrát delší bude skutečná trajektorie s šikmého vrhu (s počátečním a koncovým bodem ve stejné výšce) (a) s maximálním možným doletem D, (b) pokud počáteční (elevační) úhel a = 30°, (c) pokud elevační úhel a = 60°, (d) pokud elevační úhel bude takový aby maximální výška trajektorie se rovnala doletu, ty s s než příslušný dolet? Dokažte, že pro a = —, — —> oo, a rovněž že pro a = 0, — —> 1. 1 a s (b) s c s (d) s 1 + ln COS Q 1 + sin q COS Q COtg Q 1= + ln Vl + V2 72 _L_ 73 ln 73 73+ 2 1 + ln 72 + 73 ^3^ D « 1,15L», D « 1,05L>, D « 1,38L>, 7l7 ln 74 + 7l7 D « 2,327J 4.4 Přímým výpočtem v kartézských souřadnicích a také pomocí vhodné parametrizace vypočítejte hmotnost asteroidy x2!3 + y2/3 = a2/3 s délkovou hustotou r = x4/3 + y4/3 (viz obrázek 4.1). Parametrizace asteroidy může být například: x = a cos31, y = a sin3 í, kde t neznačí čas, nýbrž úhlový parametr. m = 4a7/3 4.5 Pomocí parametrizace x = a cos t, y = a sin í, z = bt, vypočítejte hmotnost jednoho z2 „závitu" válcové šroubovice s délkovou hustotou r = —^-x- x2 + y2 8tt3 (b\2 /->- m =- — Vfl + o 3 \a/ Kapitola 4- Křivkový integrál 53 Obrázek 4.1: Asteroida. Délka poloosy a je vyznačena zelenou barvou. 4.6 Pomocí vhodné parametrizace a s použitím obecných koeficientů vypočítejte hmotnost jednoho „závitu" válcové šroubovíce s délkovou hustotou r = —^-^-~. xl + yz + Va2 + 62 2tt6 m =---arctg- ab a 4.7 Pomocí parametrizace x = at cos t, y = aísiní, z = bt, vypočítejte hmotnost jednoho „závitu" kuželové šroubovice s délkovou hustotou r = 2\J x2 + y2 — z. m 2a-b i2+ b2+An2a2f - J(a2+ b2) 4.8 Dokažte vztah s 4>2 f + / d0, kde s je délka hladké křivky, vyjádřené v polárních souřadnicích r = /(), a kde / = df /d(f>. z transformačních vztahů pro polární souřadnice a z definice křivkového integrálu 4.9 Dokažte vztah s = J ^ f2 + 92 (V2 sin2 6 + f2^j dO, kde s je délka hladké křivky, vyjádřené v kulových souřadnicích r = /(), 0 = = —x2y + C 5.35 Intenzita fyzikálního pole je určena vektorem A = ln(x — y) H--,--,0 . Lze L x — y x — y pro toto pole stanovit příslušný potenciál ? Pokud ano, nalezněte jej. Bude tento potenciál určen jednoznačně ? (j) = —x \n{x — y) + C 5.36 Dokažte, že dané centrální silové pole F = —kfr je konzervativní a určete odpovídající potenciální energii V v bodě x,y,z = {Xq,Yq, Zq), pokud její hodnota v bodě x,y,z = (0, 0, 0) je rovna Vq. Veličina k je konstanta, r je polohový vektor, r je jeho velikost. k V(X0,Y0,Z0) = - y- (X2 + Y2 + Z2)3/2 + V0 5.37 Dokažte, že dané centrální silové pole F =--, definované pro r > 1, je konzervativní a r určete odpovídající potenciální energii V v bodě x,y, z = (Xq, Yq, Zq), pokud její hodnota v minimální definované vzdálenosti od bodu x,y,z = (0,0,0) je rovna V$. Veličina k je konstanta, r je polohový vektor, r je jeho velikost. V(X0, Y0, Zq) = k Í^/X2 + Y2 + Z2 -Í)+V0 5.38 Dokažte, že dané silové pole F = —k—, definované pro r > 1, je konzervativní a určete odpovídající potenciální energii v v bodě Xq,Yq,Zq = (2,2,1), pokud hodnotu potenciální energie ve vzdálenosti r = 1 od bodu x,y, z = (0, 0, 0) stanovíme jako Eq = 0. Veličina k = 1,5 je obecná konstanta, r je velikost polohového vektoru r = (x,y, z). 7(2,2,1) k 1 VX2 + Y2 + Zl 1 5.39 Dokažte, že centrální silové pole F, definované pro r > 1, je konzervativní a určete potenciální energii V pole v bodě x,y, z = (Xq, Yq, Zq), pokud stanovíme její hodnotu v minimální definované vzdálenosti od bodu x,y,z = (0, 0, 0) je jako nulovou: (a) F (b) F (c) F -k f In r, -krlnr2, - k f In r3. Veličina k je konstanta, r je polohový vektor, r je jeho velikost. (a) V(Xq,Yq,Zq) = - x2q+y2 + Z2)(\n^X2 + y2 + Z2 1 + 2 (b) V(X0, Yq,Zq) = -{ (X2 + Y2 + Z2) [ln (X2 + Y2 + Z2) - l] + 1} (c) V(X0,Yq,Zq) = - \(XŠ + Y* + Zl ln(X2 + Y2 + Z2 + Kapitola 5. Skalární a vektorové funkce více proměnných 65 5.40 Dokažte, že dané silové pole f = — k (x, y, z) lnr~2, definované pro r > 1, je konzervativní a určete odpovídající potenciální energii V v bodě x, y, z = (Xq,Yq, Zq), pokud potenciální energie ve vzdálenosti r = 1 od bodu x, y, z = (0,0,0) je rovna Eq. Veličina k je konstanta, r je velikost polohového vektoru r = (x, y, z). V(X0, y0, z,) = -k-{{X2 + y02 + Z2) [ln (Xq + Y2 + Z2) - l] + 1} + E0 5.41 Dokažte, že dané centrálni silové pole f = —kfer je konzervativní a určete odpovídající potenciální energii V v bodě x, y, z = (Xq,Yq, Zq) pokud hodnota potenciálni energie v bodě x, y, z = (0, 0, 0) je rovna — Vq = —k. Veličina k je konstanta, r je polohový vektor, r je jeho velikost. V(X0, Y0, Z0) = Vo eVWf Ux* + Yg + Z* - 1 5.3 Diferenciální operátory Diferenciální operátory určují působení operátoru nabla (viz oddíl 5.1) na skalární nebo vektorové (případně tenzorové) pole některým z následujících způsobů: • gradient skalární funkce/: grad/ = V/, výsledkem je vektor (5.10) • divergence vektorového pole A: div A = V • A, výsledkem je skalár (5-11) • rotace vektorového pole A: rot A = V x A, výsledkem je vektor (5.12) • Laplaceův operátor (Laplacián): A = V • V, nemění původní pole (5.13) Laplaceův operátor tzv. nemění řád tenzoru, tj. pokud působí na skalár, výsledkem je skalár, pokud působí na vektor, výsledkem zůstává vektor, atd. (viz příloha A). Variantní forma zápisu diferenciálních operátorů pomocí volných indexů (v Einsteinově konvenci) může v kartézském souřadnicovém systému vypadat následovně (význam funkce Síj a tenzoru Eijk je vysvětlen v odstavci 2.3): ^ df • gradient skalární funkce/: grad / = ě*j——, (5-14) OXi d d • divergence vektorového pole A: div A = ——Aj ôij = ——Ai, (5.15) (Jju'i (Jdu'i • rotace vektorového pole A: rot A = sak^i——Ak, (5.16) OXj d d -> _ d d • Laplaceův operátor (Laplacián): A/ = ————/, AA = ě*j————Ai. (5-17) OXi OXi OXj OXj Gradient skalární funkce reprezentuje vektorové pole, udávající velikost a směr největšího nárůstu dané skalární funkce. Divergenci vektoru můžeme interpretovat například jako „míru expanze" dané vektorové veličiny (respektive jejího toku) v obecném bodě v prostoru, případně jako její „zřídlovost", tj. míru toho, jak mnoho se tok daného vektorového pole chová jako „zdroj" příslušné vektorové veličiny. Například, pokud vektorové pole zároveň „vzniká" (zdroj) Kapitola 5. Skalární a vektorové funkce více proměnných 66 i „zaniká" (propad), je jeho divergence nulová (případ magnetické indukce), rovněž homogenní vektorové pole (konstantní vektor) musí mít z definice nulovou divergenci, atd. Rotace vektorového pole (jak vyplývá z názvu) popisuje infinitesimální rotaci daného pole v obecném bodě v prostoru; pokud je rotace nulová, mluvíme o „nevírovém" toku dané vektorové veličiny. Podrobný popis odvození jednotlivých diferenciálních operátorů v hlavních souřadnicových soustavách včetně související matematiky je uveden v příloze A. Zde je uveden pouze základní přehled operátorů (ve válcové soustavě nyní zavádíme pro odlišení p namísto r): Kartézská souřadná soustava (x±, x2, x% = x,y,z) grad / = 2i df 21 dx' dy' d z div A rot A A + dAx , dAu dx dA dy + dz dAy dAx dAz dAv dA, dy dz ' dz dx ' dx dy _cP_ _cP_ _cP_ dx2 dy2 dz2 Válcová souřadná soustava (x±, x2, x% grad / = div A rot A A 2i i2i 21 d p' p d dz ' dz 1 d d P- dp p d2 d_ dp dA, p2 d r sin 6 d d dr dr rAa dAr | 1 d , r2 sin 9 dO \ d9 • Příklady: Pro skalární funkce f,g& vektory A, B dokažte: 5.42 V(/fl) = (V/)5 + fVg 5.43 VxV/ = 0 + d2 r2 sin2 6 dep2 Kapitola 5. Skalární a vektorové funkce více proměnných 67 5.44 V^Vxi 5.45 V • f Ä = Ä ■ V/ + f'V ■ Á 5.46 V • (Äx É) = B ■ (v x A) - A - (V x B 5.47 V x f A = V f x A + fV x A 5.48 Dokažte platnost operátorové identity V x [V x A) = V ( V • A) — A A. 5.49 Dokažte platnost operátorové identity I x |v x ij = ^\'7A2 — ■ Ä. Příklady 5.42 - 5.49 - pomocí vektorových operací v kartézských souřadnicích. 5.50 Spočítejte div ^rot F J, F = [xyz, y (x2 — z2) , xy + zx + yz\. 0 5.51 Spočítejte div (rot FJ, F = (x2y, y2, z2x). 0 5.52 Spočítejte grad /, f (x, y, z) = 2xyz + x2y + y2 z + z2x. 2yz + 2xy + z2, 2xz + x2 + 2yz, 2xy + y2 + 2x2 — r 5.53 Středově symetrické (izotropní) fyzikální pole je určeno vektorem A = -, kde r je polo- r - - 2 hový vektor, r je jeho velikost. Dokažte, že divergence tohoto pole, tedy V • A = -. r r r (x, y. z) ^/x2 + y2 + z2 -\/x2 + y2 + ,2 r 5.54 Hypotetické centrální fyzikální pole je určeno potenciálem 1 1 E = — = —v ;—, V • E = — =- r2 x2 + y2 + z2' r2 x2 + y2 + z2 5.55 Hypotetické centrální fyzikální pole je určeno potenciálem 1, je určeno potenciálem (f) = —Ar2lnr2 + i3, kde konstanta A škáluje velikost r polohového vektoru r, konstanta B nastavuje hodnotu potenciálu (f) v minimálni definované vzdálenosti od bodu Kapitola 5. Skalární a vektorové funkce více proměnných 68 x, y, z = (0,0,0). Určete vektor intenzity E tohoto pole a dokažte, že divergence tohoto pole, tedy V • Ě = A (6 ln r2 + 10). Ě = 2Ar (lnr2 + l) = 2A(x,y,z) [ln(x2 + y2 + z2) + l] , V • Ě = A (61nr2 + 10) Ax 5.57 Hypotetické fyzikální pole je určeno nesymetrickým potenciálem 0 = -, kde A je kladná r konstanta a r je velikost polohového vektoru r. Určete vektor intenzity E tohoto pole a - - 20 dokažte, že divergence tohoto pole, tedy V • E = —. -A(y2 + z2), Axy, Axz 2Ax 2Ax E =-——'---—-, V • E =-=- r3 (^,2 _|_ y2 _|_ ^2)3/2 r3 5.58 Hypotetické centrální fyzikální pole je určeno potenciálem 0 = Ae~r, kde A je kladná konstanta, r je velikost polohového vektoru r. Určete vektor intenzity E tohoto pole a - - (2 \ dokažte, že divergence tohoto pole, tedy X7 ■ E = Ae r(--11. \J'x2 + y2 + z2 \ \Jx2 + y2 + z2 5.59 Hypotetické centrální fyzikální pole je určeno potenciálem 0 = A~r, kde A je kladná konstanta, r je velikost polohového vektoru r. Určete vektor intenzity E tohoto pole a - - /2 \ dokažte, že divergence tohoto pole, tedy V • E = A rlnA(--InAJ. Ě = A-rlnA-, V ■ Ě = A-^x2+y2+z2lnA[ / ^=-lnA\ r \ \fx2 + y2 + z2 J 5.60 Pomocí Kroneckerova delta a Levi-Civitova symbolu ověřte vektorové identity z příkladů (5.42)-(5.49) v Einsteinově notaci. Pomocí rovnic (2.48) a (5.14)-(5.17). Kapitola 6 Dvojný a trojný integrál1 Na integrál funkce f (x, y) dvou proměnných x, y (dvojný integrál), která je spojitá na dvourozměrné oblasti S = (a, b) x (c, d), kde ai(x), 2(x) proměnné x na celém intervalu a, b platí (f>i(x) < y < (f>2(x), je integrál spojité funkce f (x,y) dvou proměnných definován jako f (x, y) dx dy 02 O) 01 (x) f(x,y)dy dx b ľi(x), 2(x) proměnné x na celém intervalu a, b platí i{x) < y < ^{x) a kde pro spojité funkce ip\{x,y), ip2{x,y) dvou proměnných x,y na celé oblasti S = {a, b) x [0i(x), 2(x)] platí ip\ (x, y) < z < ?p2(x, y), integrál spojité funkce f (x, y, z) tří proměnných (trojný integrál) definován jako f(x,y,z)dx dy dz 4>2(x) í ľip2(x,y) \ / f{x,y,z) dz dy dx b ľ4>2Íx) rip2(x,y) dx dy f(x,y,z)dz. J4>i{x) Jipi(x,y) (6.3) Transformaci souřadnic dvojného integrálu lze definovat (viz obrázek 6.1) pomocí prostého regulárního zobrazení <í> : íl —> M2(x,y), zadaného transformačními rovnicemi x = £(u,v), y = i](u, v), kde íl C M2(u, v) je otevřená oblast. Pokud A C M2(x, y) a B C M2(u, v) jsou uzavřené oblasti, kdy A C íl, B C íl, a pokud funkce f (x, y) dvou proměnných x a y je spojitá v oblasti A = $(B), potom platí f(x,y)dxdy = // / [£(u,v),r)(u,v)] J(u,v) dudv. a JJb (6.4) 1Ve výsledcích příkladů s geometrickými nebo fyzikálními veličinami nejsou uváděny příslušné jednotky. 69 Kapitola 6. Dvojný a trojný integrál 70 Výraz J(u,v) I deti I dx dx\ du dv dy dy \du dv/ (6.5) v rovnici (6.4) je Jakobián dvourozměrné souřadnicové transformace. V případě trojného integrálu funkce f(x,y,z) tří proměnných x,y,z, spojité v uzavřené oblasti A C M3(x,y, z), s transformačními rovnicemi x = £(u,v,w), y = r](u,v,w), z = ((u,v,w), v oblasti B C M3(u, v,w) analogicky k rovnici (6.4) platí, f(x,y,z)dx dy dz f [Š(u, v, w),r}(u, v, w), v, w)] J(u, v, w) dudv dw. (6.6) Výraz J(u, v, w) Idetl / dx dx dx \ du dv dw dy_ dy_ dy du dv dw dz dz dz \du dv dw 1 (6.7) v rovnici (6.6) je Jakobián trojrozměrné souřadnicové transformace. Významné a často používané jsou Jakobiány souřadnicových transformací (podrobný popis je v příloze A) z kartézské do válcové souřadné soustavy, kdy (u,v,w) = (p,(f>,z), s transformačními rovnicemi x = pcoscf), y = psin (f), z = z, kde J = p, a z kartézské do kulové souřadné soustavy, kdy (u, v, w) = (r, 6, 0), s transformačními rovnicemi x = r sin 9 cos 0, y = r sin 9 sin 0, z = r cos 9, kde J = r2 sin 9. 6.1 Plošný integrál 1. druhu Plošným integrálem 1. druhu nazýváme (analogicky ke křivkovému integrálu 1. druhu v odstavci 4.1) integrál j j f dS skalární funkce f(x, y, z), spojitě diferencovatelné na úseku (hladké, tj. spojitě diferencovatelné) plochy S, kde dS je element plochy S. Stanovíme-li například souřadnice x a y jako nezávisle proměnné a funkci z = z(x, y) jako závisle proměnnou, můžeme tečné vektory tx,ty k ploše S ve směrech souřadnicových os x,y určit jako parciální derivace funkce dané plochy (viz odstavec 5.1) podle příslušných směrů, tedy f dx dy dz\ I dz\ -, f dx dy dz\ / dz y^dx' dx' dx) ^^'dx)' y \dy' dy' dy) \ ' dy) ^) Vektor v normály plochy S (tj. vektor kolmý k ploše S) určíme jako vektorový součin tečných vektorů tx x ty (jejichž pořadí ve vektorovém součinu závisí na požadované orientaci normály), jeho velikost (viz rovnice (2.1)) bude ||i7|| = \\tx x ty\\, tedy - - í dz dz \ - / /dz\2 (dz\2 .-=±fexř„) = ±^-,--,lj, H = ||řlXg| = ^+(_j +^_) . (8.9) Samotný plošný element je vektorem, orientovaným ve směru normálového vektoru, kde velikost plošného elementu je určena velikostí normálového vektoru. Ve zvolené kartézské parametrizaci Kapitola 6. Dvojný a trojný integrál 71 v Obrázek 6.1: Schéma transformace obecných souřadnic, popsané rovnicí (6.4), kde symbolizuje vzájemně jednoznačné zobrazení. C\, C2 jsou souřadnicové křivky, pro které platí C\ : x = x(u,vo),y = y(u,vo), C2 : x = x(uo,v),y = y(uo,v), čárkované čáry, ohraničující zvýrazněnou plochu, můžeme označit jako: C^(u,vq + Av), C-f(it,i>o — Av), C^(ito + Au, v), C^Íuq — Au, v). Tečné vektory tu, tv k souřadnicovým křivkám v bodě u0,v0 stanovíme podle rovnice (6.12). tedy můžeme psát dS* = v dx dy = ± dz dz dx' dy' 1 ) dxdy, dS = dxdy \dx) \dy dx dy. (6.10) Zjevně tedy platí dS* ■ dS* = ndS, kde n je jednotkový normálový vektor. Explicitní zápis plošného integrálu 1. druhu v kartézské bázi, kde a < x < b, (f>i(x) < y < (f>2(x) (srovnej rovnice (6.1), (6.2)) a z = z{x,y), tedy bude: f{x,y,z) dS 02 O) a J(f>i(x) f{x,y,z) I dx + dz\' dy) dx dy. (6.11) Nalezneme-li vhodné parametry u, v (např. 9, , dS = R2 sin 9 d9 d, (6.16) jejím zintegrováním v mezích 9 G (0,7r), 0 G (0, 2tt) dostáváme S = 4ttR2. Vzhledem k tomu, že Jakobián, odpovídající rovnici (6.5) bude v tomto případě R2 sin 9 cos 9, snadno se z rovnic (6.15), (6.16) přesvědčíme o platnosti vztahu ||ž?|| dx dy = \\u'\\J(9, (f>) d9 d(f>. Pomocí plošného integrálu 1. druhu lze určit geometrické a fyzikální charakteristiky dané plochy: Položíme-li / = 1, výsledkem bude celková velikost plochy S. Položíme-li / = o (plošná hustota), dostáváme a dS = dm, tedy element hmotnosti plochy S, výsledkem integrace bude celková hmotnost m dané plochy, m = jj dm = jj adS. (6.17) Pokud položíme například / = za, dostáváme tzv. statický moment Sz dané plochy vzhledem k ose z, jeho vydělením hmotností dostáváme z-ovou souřadnici středu hmotnosti zt plochy (obdobně pro ostatní souřadnicové směry), tedy xt = — íí xdm = — [i xadS, yr = — [f y dm, zt = — ff z dm. (6.18) m JJS m JJS m JJ s m JJ s Položíme-li / = r2a, kde r je vzdálenost obecného bodu plochy od zvolené přímky v prostoru (osy o), dostáváme moment setrvačnosti JQ dané plochy vzhledem k této ose. Momenty setrvačnosti plochy S např. vzhledem k jednotlivým kartézským souřadnicovým osám potom budou Jx = // (žT + z1) dm = U (yz + zz)udS, Jy = I {z1 + xz) dm, Jz = I (xz + yz) dm. JJs JJ s JJs JJ s (6.19) • Příklady: V kartézských souřadnicích i pomocí vhodné parametrizace vypočítejte obsah plochy 5éR2, ohraničené následujícími křivkami: 1 4 y = x",y = ex-, y = -,y = - 6.1 - - ~2----~2 Kapitola 6. Dvojný a trojný integrál 73 y = X2^y= IQ^^ y = _J_ y 6.3 i, = x3 2^2 _ 16 g tie tlC x3 1 e (e-l)ln3 á x x 4 6.4 y = 1, y = e, y = e°>2x, y = e°>4x 2,5 Pomocí vhodné parametrizace, případně v kartézských souřadnicích, vypočítejte obsah plochy S G M3: 6.5 S={x2+y2 + z2=R2, x2+y2 < a2, z > 0, a < R}(kulový vrchlík) 2tvR R - v'R2 - a 6.6 S = {x2 + z2 = a2, z > 0} (a) |x| < |y| < a 2a2(7r - 2) (b) |x| > |y| 4a2 6.7 S = {x2 + z2 = a2, x2 + y2 < a2, z > 0} 4a2 6.8 Prostorová křivka, daná průnikem kulové plochy S = {x2 + y2 + z2 = i?2} a válcové plochy 5' = {(x - a)2 + y2 = a2}, kde a = R/2, vymezuje na povrchu kulové plochy uzavřenou plochu (tzv. Vivianiho okno). Vypočítejte obsah této plochy. 2i?2 (tt - 2) 6.9 Vypočítejte obsah části zemského povrchu, ohraničeného v jednom směru dvěma sousedními poledníky (například 15. a 16.) a ve druhém směru dvěma sousedními rovnoběžkami: (a) 0. (rovníkem) a 1., (b) 49. a 50., (c) 89. a 90. (pólem). Obsahy jednotlivých ploch udejte v km2. Poloměr Země, R = 6371 km, považujte za konstantu. (a) cca 12 364 km2 (b) cca 8 030 km2 (c) cca 108 km2 V kartézských souřadnicích i pomocí vhodné parametrizace vypočítejte plošné integrály 1. druhu: ttR5 6.10 JJ x2zdS, kde S = {x2 +y2 + z2 = Ŕ2, z > 0}. 6.11 JJ V2x2zdS, kde S = {x2+y2 - z2 = 0, ze (0,H)}. ^-,R = H Kapitola 6. Dvojný a trojný integrál 74 6.12 // (x2+y2)dS, kde S = {x2 + y2 + z2 = R2, z > 0}. -ttí?4 3 6.13 JJ x2y2dS, kde S = {x2 + y2 + z2 = R2, z > 0}. 6.14 x2z2dS, kde S = {x2 + y2 + z2 = Ŕ2, z > 0}. — TlRb 15 2 « —ttí?6 15 6.15 f] V2&S, kde -5 = j * = H - ^x2 +y2, z G ((),#) j. ^~VR2 + ii2 6.16 Ij zdS, kde S = {x + y + z = 1, x e (0,1), y e (O, 1 - x)}. 6.17 1 5 x2 + y2 + z dS, kde S* = {x + y + z = ii, z > 0}. 2V3 7T 6fľ 1 +4ii)2 - 1 i?2 6.18 Vypočítejte hmotnost kulového vrchlíku S = <^x +y + z = R , x + y < —, z > 0 s plošnou hustotou a(x,y,z) = \x\ + |y| + |z|. i?3 (II. + VŠ 6.19 Vypočítejte polohu středu hmotnosti plochy S = {x2 + y2 + z2 = Ŕ2, z > O}, jejíž plošná hustota a je dána funkcí cr = x2 + z2. 9i? xt = 0, yT = 0, zT = — 16 6.20 Vypočítejte polohu středu hmotnosti plochy S = {x2 + y2 — z2 = 0, z G (0,ii}}, jejíž plošná hustota a je dána funkcí cr = x2 + z2. AH xt = 0, yT = 0, zT = — 6.21 Spirálová plocha s konstantní plošnou hustotou a je zadaná parametricky ve tvaru x ucost), y = usvav, z = v, u G (0, a), u G (0,2tt). Vypočítejte: (a) hmotnost této plochy, (b) polohu jejího těžiště, (c) moment setrvačnosti vzhledem k její geometrické ose. a tto" iVl + a2 + ln (a + vTT (b) (0,0, tt) 7TO" Í2a3 + a) Vi + a2 - ln ( a + Vi + a2 6.22 Vypočítejte následující parametry plochy z příkladu 6.15, pokud její plošná hustota bude Kapitola 6. Dvojný a trojný integrál 75 (a) hmotnost této plochy, (b) polohu jejího těžiště, (c) moment setrvačnosti vzhledem k její geometrické ose. tt [a) —HR3V R2 + H2 (b) zT = j íc) —HR5VR2 + H2 w 42 6.23 Vypočítejte následující parametry plochy z příkladu 6.16, pokud její plošná hustota bude a = x2 + y2 : (a) hmotnost této plochy, (b) polohu jejího těžiště. 2VŠ 6.24 Vypočítejte hmotnost plochy z příkladu 6.17, pokud její plošná hustota bude a = x2 16 [AR2 + lý (Ji?2 2 \ 2 15 / + 15 6.25 Vypočítejte celkovou tlakovou sílu, kterou působí kapalina o konstantní hustotě p na všechny stěny uzavřené nádoby, tvořené plochou z příkladu 6.15 a odpovídající podstavou (atmosférický tlak zanedbejte). npgH ( -RVR2 + H2 + R2 3 6.26 Vypočítejte celkovou tlakovou sílu, kterou působí kapalina o konstantní hustotě p na všechny stěny uzavřené nádoby, tvořené plochou z příkladu 6.17 a odpovídající podstavou (atmosférický tlak zanedbejte). ňP9 i 9 \^ /4 9 2\ 2 <4H+1)a 5^-15 +15 + irpgR4 6.27 Plášť vodojemu ve tvaru kužele, stojícího „špičkou" dolů, o poloměru horní vodorovné plochy R = 3 m a výšce H = 4 m je dimenzován tak, aby odolal celkové tlakové síle 106 N. Je dimenzován dostatečně, nedostatečně, nebo je přibližně na hranici konstrukční odolnosti ? Uvažujte hodnoty konstant p = 1000 kg m-3, g = 9, 81ms~2. Vliv atmosférického tlaku zanedbejte. F„ ~ 6, 3 x 105 N. Plášť vodojemu je dimenzován dostatečně. Kapitola 6. Dvojný a trojný integrál 76 6.28 Nádoba ve tvaru kužele stojícího „špičkou" dolů je naplněna speciální kapalinou, v níž tlak roste s hloubkou jako p = p^gh2, kde po Je hustota kapaliny na hladině a h je hloubka daného místa v nádobě. Poloměr horní vodorovné plochy nádoby R = 0,5 m a výška nádoby H = 1 m. Určete tlakovou sílu, které musí nádoba odolat. Uvažujte hodnoty konstant po = 1000 kg m-3, g = 9,81ms~2. Vliv atmosférického tlaku zanedbejte. Fp « 3000 N. 6.29 Plášť kulového vodojemu o poloměru R = 2 m je dimenzován tak, aby odolal celkové tlakové síle 106 N. Je dimenzován dostatečně, nedostatečně, nebo je zhruba na hranici konstrukční odolnosti ? Uvažujte hodnoty konstant p = 1000kgm~3, g = 9,81ms~2. Vliv atmosférického tlaku zanedbejte. Fp ~ 106 N. Plášť vodojemu je dimenzován zhruba na hranici konstrukční odolnosti. 6.30 Mísa ve tvaru polokoule o poloměru R = 1 m je naplněna speciální kapalinou, v níž tlak roste s hloubkou jako p = p^ghÄ, kde po je hustota kapaliny na hladině a h je hloubka daného místa v nádobě. Určete tlakovou sílu, které musí nádoba odolat. Uvažujte hodnoty konstant po = 1000 kg m-3, g = 9,81ms~2. Vliv atmosférického tlaku zanedbejte. F„ « 25 000 N. 6.2 Plošný integrál 2. druhu Plošným integrálem 2. druhu nazýváme integrál J J F ■ dS = J j F ■ ndS obecného vektorového pole F (x, y, z), definovaného na orientované, po částech diferencovatelné ploše S, kde n = (nx,ny,nz) Je jednotkový vektor normály orientované plochy S. Explicitní zápis plošného integrálu 2. druhu v kartézské souřadné soustavě bude, v případě že všechny tři souřadnice x, y, z jsou na dané ploše (soustavě ploch) vzájemně nezávislé, mít tvar Fx dy dz y J z x=konst. + Fy dz dx + I I Fzdxdy j/=konst. Jx Jy z=konst. (6.20) kde Fx, Fy, Fz jsou jednotlivé složky vektoru F. V případě, že můžeme na dané ploše stanovit některou ze souřadnic jako závislou na druhých dvou, bude mít (analogicky k rovnici (6.14)) parametrizovaný plošný integrál 2. druhu tvar Fx(£,ri,0 du' du di] d( dv' dv d( d£ du du dv' dv + + Fz(Z,ri,0 <9£ di] du du <9£ di] dv' dv dudv . (6.21) Pořadí parametrů u a v ve vektorových součinech je dáno požadovanou orientací normály plochy. Typickou fyzikální aplikací plošného integrálu 2. druhu je výpočet toku <í> vektorového pole F orientovanou plochou S. Kapitola 6. Dvojný a trojný integrál 77 • Příklady: 6.31 Pomocí plošného integrálu 2. druhu určete tok vektorového pole F = (x2, y2, z2) uzavřenou plochou S = {(x,y,z)\x G (A, 2A), y G (B, 2B), z G (C, 2C)}. $F = 3ABC(A + B + C) 6.32 Pomoci plošného integrálu 2. druhu určete tok vektorového pole F = (x2, y2, z2) uzavřenou plochou S = {(x, y, z) \ x2 + y2 = R2, z G (0, H)}. $f = ttR2H2 6.33 Pomoci plošného integrálu 2. druhu určete tok vektorového pole F = (x2, y2, z2) uzavřenou plochou, tvořenou povrchem tělesa z příkladu 6.17. ttR2H2 &F = - 3 6.34 Pomocí plošného integrálu 2. druhu určete tok vektorového pole F = (x3 — y3, x3 + y3, z) í R2 povrchem tělesa V = < (x, y, z) \ z G (0, H), x2 + y2 < j^(H — z)2 $F = (9i?2 + 10) 6.35 Pomoci plošného integrálu 2. druhu určete tok vektorového pole F = (x3, y3, z3) plochou, danou předpisem S = {(x, y, z) | x2 + y2 + z2 = i?2 }. $f = ^7tí?5 5 6.36 Je dáno silové pole F = (x3 — x2, y3 — y2, z3 — z2). Pomoci plošného integrálu 2. druhu vypočítejte jeho tok povrchem tělesa V = {(x, y, z) | x, y, z G (0,i?), x2 + y2 +z2 < i?2}. •,-?£<«-«> 6.37 Pomoci plošného integrálu 2. druhu vypočítejte tok vektorového pole F = (x, y, z) povrchem tělesa z příkladu 7.35. Proč je výsledná hodnota trojnásobkem výsledné hodnoty z uvedeného příkladu ? 37r2a3 &F =- 4 6.38 Pomoci plošného integrálu 2. druhu vypočítejte tok vektorového pole F = (x3, y3, z3) uzavřenou plochou, tvořenou povrchem tělesa z příkladu 7.35. ä,F = ^!7r2a5= (!]3AS 64 V4/ 6.39 Pomoci plošného integrálu 2. druhu vypočítejte tok vektorového pole F = (z2, x2,y2) kruhovou plochou o poloměru R se středem v bodě x, y, z = A, B, C, ležící v rovině z = C. Kapitola 6. Dvojný a trojný integrál 78 TtR4 „o „o -+ tvB2R2 4 6.40 Pomoci plošného integrálu 2. druhu vypočítejte tok vektorového pole F = (y, z, x) rovinnou plochou ve tvaru obdélníka s vrcholy v bodech (1, 0, 0), (3, 0,1), (3, 2,1), (1, 2, 0), ve směru normály u této plochy jejíž složka ux je kladně orientovaná. -6 6.41 Pomocí plošného integrálu 2. druhu vypočítejte tok vektorového pole F = (3, z, y) rovinnou plochou ve tvaru obdélníka s vrcholy v bodech (0, 0,1), (0,1, 3), (2,1, 3), (2, 0,1), ve směru normály u této plochy jejíž složka uy je kladně orientovaná. 7 6.42 Pomocí plošného integrálu 2. druhu vypočítejte tok vektorového pole F = (3, z, y), rovinnou plochou ve tvaru obdélníka s vrcholy v bodech (0, 0,1), (0, 2, 2), (5, 2, 2), (5, 0,1), ve směru normály u této plochy jejíž složka uy je kladně orientovaná. 5 ~2 6.43 Pomoci plošného integrálu 2. druhu vypočítejte tok vektorového pole F = (y, z, x) rovinnou plochou ve tvaru lichoběžníka s vrcholy v bodech (1,1,1), (1, 3, 3), (2,4, 5), (2,1, 2), ve směru normály u této plochy jejíž složka uy je kladně orientovaná. 53 ~6~ 6.44 Pomoci plošného integrálu 2. druhu vypočítejte tok vektorového pole F = (x, z, y) rovinnou plochou ve tvaru trojúhelníka s vrcholy v bodech (3, 0, 2), (1, 2, 0), (0, 0, 7), ve směru normály u této plochy jejíž složka uy je kladně orientovaná. 98 ~3 6.3 Objemový integrál Objemovým integrálem označujeme trojný integrál skalární funkce f (x, y, z) přes oblast (těleso) T G M3 s objemem V: jjj f (x, y, z) dV = Jjj f (x, y, z) dx dy dz. (6.22) Pomoci objemového integrálu lze určit geometrické a fyzikálni charakteristiky těles: Položíme-li / = 1, výsledkem bude velikost objemu V tělesa T. Položíme-li f = p (objemová hustota hmoty), dostáváme pdV = dm, tedy element hmotnosti tělesa T, výsledkem integrace bude celková hmotnost M tělesa, M = illrám = l!lvPAV- (6.23) Kapitola 6. Dvojný a trojný integrál 79 Pokud položíme například / = zp, dostáváme tzv. statický moment Sz tělesa vzhledem k ose z, jeho vydělením hmotností dostáváme z-ovou souřadnici středu hmotnosti zt tělesa (obdobně pro ostatní souřadnicové směry), tedy xT=^!S!Txám=^!S!vxpáV' vt=^\\\tv^ zT=^IÍfTzdm' (6-24) Položíme-li / = r2p, kde r je vzdálenost obecného bodu tělesa od zvolené přímky v prostoru (osy o), dostáváme moment setrvačnosti JQ tělesa T vzhledem k této ose. Momenty setrvačnosti tělesa T např. vzhledem k jednotlivým kartézským souřadnicovým osám potom budou Jx = IIJ (y2+z^dm = jff (y2+ňpdV,Jy = jff {z2+x2)ám, Jz = fff {x2+y2)ám. (6.25) Příklady k problematice objemového integrálu jsou součástí následujícího odstavce 6.4. 6.4 Geometrické a fyzikální charakteristiky útvarů • Vypočítejte objem: 4 6.45 elipsoidu o poloosách a, b, c, V = —nabc 6.46 kužele o poloměru podstavy R a výšce H, V 6.47 tělesa A = {(x, y, z) | z G (0, H - x2 - y2)}, kde H = R2, V 3 ti R2 H 3 ■k R2 H í R2 6.48 tělesa „4 = {(x,y,z)\z G (0, h-x2-y2)}, x2 + y2 < R2, h > R2, V = ttR2 (h--— 6.49 anuloidu (toroidu) o poloměru osy toru R a poloměru trubice a,1 V = 2n2Ra2 6.50 tělesa „4= j (x, y, z) | z G (J^ ^ ■ yjR2-x2-y^y V = 6.51 tělesa A = j (x, y, z) | z G ^R2 - x2 - y2^, V = ^ttR 6.52 tělesa A, jehož povrch vznikne rotací asteroidy z příkladu 4.4 okolo osy y, v 32 3 105 6.53 tělesa A, jehož povrch vznikne rotací kardioidy z příkladu 4.13 okolo osy y. V = -Tra3 3 1 Podrobný popis anuloidu - viz odstavec A.6 v příloze A. Kapitola 6. Dvojný a trojný integrál 80 • Vypočítejte velikost plochy: 6.54 kulové slupky o poloměru R, S = AttR2 6.55 pláště kužele o poloměru podstavy R a výšce H, S = ttRVR2 + H2 6.56 pláště tělesa z příkladu 6.47, S = — 6 6.57 celého povrchu tělesa z příkladu 6.48, [l+4R2)i -1 S 6 [1 + 4R2)2 - 1 + 2ttR(H - R2) + ttR2 6.58 pláště tělesa z příkladu 6.50, S = I ——h 1 I ttR2 7 6.59 celého povrchu tělesa z příkladu 6.51, S = 6.60 pláště anuloidu (toroidu) o poloměru osy toru R a poloměru trubice a,1 S = An2Ra 3 6.61 ohraničené asteroidou z příkladu 4.4, S = g7™2 12 6.62 která vznikne rotací asteroidy z příkladu 4.4 okolo osy y, S = —na 5 3 6.63 ohraničené kardioidou z příkladu 4.13, S = i^0^ 16 6.64 která vznikne rotací kardioidy z příkladu 4.13 okolo osy y, S = —na2 3 6.65 hyperbolického paraboloidu, daného předpisem z = x — y , x + y < 4, S = ^ (l7Í - l) « 36,18 6.66 hyperbolického paraboloidu, daného předpisem z = xy, x2 + y2 < 4. Jaký by musel být poloměr p válce, jehož pláštěm je hyperbolický paraboloid ohraničen, aby jeho plocha byla stejná jako v příkladu 6.65 ? i S = ^ (5§ - 1 2 17§ +3V _ x i ps 2,44 • Ve vhodně zvolené soustavě souřadnic vypočítejte polohu středu hmotnosti: 3 6.67 homogenní polokoule o poloměru R, zt = -R Kapitola 6. Dvojný a trojný integrál 81 6.68 homogenního kužele o poloměru podstavy R a výšce H, zt = — 6.69 homogenního symetrického jehlanu o hraně podstavy A a výšce H, H zr = — 4 6.70 homogenního tělesa z příkladu 6.47, zj* = — 3 R2(3H_2i?2) 6.71 homogenního tělesa z příkladu 6.48, zt =-—-^— GH — 3i? 9 6.72 homogenního tělesa z příkladu 6.50, zt = —R 16 6.73 homogenní plochy z příkladu 6.64 a homogenního tělesa z příkladu 6.53, 25 4 Vr =--o,, Vr =--a 6.74 homogenního tělesa, ohraničeného „seshora" plochou x2 + y2 + z2 = Ŕ2 a „zespoda" plochou z = \Jx2 + y 2 3i? zT = —-=- « 0,64 i? 8 (2 - y/2) 27 6.75 tělesa z příkladu 6.51, zt = 6.76 poloviny homogenního elipsoidu o poloosách a, 6, c, s rovinou podstavy, vymezenou poloosami a, b. 3 = —c • Vypočítejte moment setrvačnosti vzhledem k ose symetrie: 2 9 6.77 homogenní koule o hmotnosti M a poloměru i?, J = —MR 5 ilii?2 6.78 homogenního válce o hmotnosti M a poloměru i?, J = —-— 6.79 homogenního kužele o hmotnosti M, poloměru podstavy i? a výšce H, j = Amí?2 10 MR2 6.80 homogenního tělesa z příkladu 6.47, J =- 3 _2i?2 6.81 homogenního tělesa z příkladu 6.48, J = ——-—^MR2 GH — 3i? Kapitola 6. Dvojný a trojný integrál 82 MR2 6.82 homogenního tělesa z příkladu 6.50. J = —-— 53 6.83 tělesa z příkladu 6.51, J = -MR2 F ' 200 6.84 homogenního elipsoidu o hmotnosti M a poloosách a, b, c, rotujícího okolo poloosy c, J = —(a2 + b2) 5 6.85 homogenního tělesa, jehož povrch vznikne rotací asteroidy z příkladu 4.4 okolo osy y, j = ^-Ma2 143 6.86 homogenního tělesa, jehož povrch vznikne rotací kardioidy z příkladu 4.13 okolo osy y, 94 J = -Ma2 35 6.87 homogenního tělesa, ohraničeného seshora plochou z = H — 2 (x2 + y2) a zespoda plochou z = 0. Výsledek vyjádřete jako funkci hmotnosti daného tělesa a délky i? = ^Jx2 +y2 = y/H/2 v rovině z = 0, _ Mi?2 6.88 prázdné uzavřené válcové nádoby, tj. sestávající z pláště a obou podstav, vytvořené z materiálu zanedbatelné tloušťky s konstantní plošnou hustotou a, s poloměrem R a výškou H = R. Výsledek vyjádřete v jednotkách celkové hmotnosti nádoby M a poloměru R, J = -MR2 4 6.89 prázdné uzavřené kuželové nádoby, tj. sestávající z pláště a podstavy, vytvořené z materiálu zanedbatelné tloušťky s konstantní plošnou hustotou a, s poloměrem R a výškou H. Výsledek vyjádřete v jednotkách celkové hmotnosti nádoby M a poloměru R, _ MR2 6.90 prázdné uzavřené nádoby, tvořené celým pláštěm tělesa (tj. sestávající z vlastního pláště i podstavy) z příkladu 6.47, vytvořené z materiálu zanedbatelné tloušťky s konstantní plošnou hustotou o. Výsledek vyjádřete v jednotkách celkové hmotnosti nádoby M a poloměru podstavy R, 4i?2)3/2í?i?2-ÍVÍ + 3i?4 J = M- 5 10/10 (l+4i?2)3/2-l + 6i?2 6.91 prázdné uzavřené nádoby, tvořené celým pláštěm tělesa (tj. sestávající z vlastního pláště i podstavy) z příkladu 6.50, vytvořené z materiálu zanedbatelné tloušťky s konstantní plošnou hustotou o. Výsledek vyjádřete v jednotkách celkové hmotnosti nádoby M a poloměru podstavy R. Kapitola 6. Dvojný a trojný integrál 83 _ (9V3 + 20) MR2 3 + 2) 24 6.92 Odvoďte moment setrvačnosti homogenní polokruhové desky zanedbatelné tloušťky s poloměrem R, rotující (a) okolo osy, procházející jejím středem, kolmé k rovině desky, (b) okolo osy, ležící v rovině desky, procházející její základnou (průměrem), (c) okolo osy, ležící v rovině desky, procházející jejím středem hmotnosti rovnoběžně s její základnou. Výsledek vyjádřete v jednotkách hmotnosti desky M a poloměru R. MR2 (a) J (b) J 4 „ T_MR2 (4VUD2^ 7 „n2 2 MR2 (c) pomocí Steinerovy věty: zt = —R, J =--( — MR ~ -MR 3ir 4 \3vr/ 100 6.93 Odvoďte moment setrvačnosti homogenní desky zanedbatelné tloušťky, jejíž okraj má tvar asteroidy z příkladu 4.4, rotující okolo osy procházející jejím středem kolmo k její rovině. Výsledek vyjádřete v jednotkách hmotnosti desky M a délky poloosy a. J = —Ma2 32 6.94 Odvoďte moment setrvačnosti duté koule o poloměru R s kulovou koncentrickou dutinou o poloměru H, s konstantní hustotou p. Výsledek vyjádřete v jednotkách hmotnosti M duté koule, jejího poloměru R a poloměru dutiny H. Pomocí limitního přechodu (případně jiným způsobem) následně odvoďte moment setrvačnosti Js homogenní kulové slupky o poloměru R. k 5 R3-H31 3 6.95 Odvoďte moment setrvačnosti homogenní krychle o hraně A, rotující (a) okolo osy, procházející jejím středem a středy dvou protilehlých stran, (b) okolo osy, procházející jejím středem a středy dvou protilehlých hran, (c) okolo osy, procházející hranou krychle (vypočítejte přímou integrací a ověřte pomocí Steinerovy věty). Výsledek vyjádřete v jednotkách hmotnosti M krychle a délky její hrany A. MA2 (a) J (b) J 6 MA2 6 Kapitola 6. Dvojný a trojný integrál 84 J '-MA2 Kapitola 7 Integrální věty1 7.1 Greenova věta Věta, nazvaná podle matematika a fyzika George Greena (1793 - 1841), dává do souvislosti integrál přes oblast D £ M2 a integrál po uzavřené křivce C, ohraničující oblast D. Pro vektorové pole F = [F±(x, y), F2{x, y)], spojitě diferencovatelné v D{x,y), platí následující formulace Greenovy věty: dF2 dF\ dx dy D \ vx uy / jdD áxáy=(L (Fldx + F2dy), (7.1) JdD kde d D značí matematicky kladně orientovanou uzavřenou hranici oblasti D (křivku C). Sto-kesova věta (viz odstavec 7.2) je zobecněním Greenovy věty pro W1. • Příklady: 7.1 Pomocí Greenovy věty vypočítejte křivkový integrál (j) ex [(1 — cos y) dx — (y — sin y) dy], kde C je kladně orientovaná uzavřená křivka ohraničující oblast Ľ:0 0, z G (0, íř)}. 4 ttR2H2 tR3H + _ 27ľR2H 3 2 7.45 Pomoci Gaussovy věty určete tok vektorového pole F = (x2, y2, z2) uzavřenou plochou, tvořenou povrchem tělesa z příkladu 6.47. tvR2H2 &F =- 3 7.46 Pomocí Gaussovy věty určete tok vektorového pole F = (x3 — y3, x3 + y3, z) uzavřenou f R2 plochou, tvořenou povrchem tělesa V = < (x, y, z) | z G (0, H), x2 + y2 < -^(H — z)2 $F = (9i?2 + 10) 7.47 Pomoci Gaussovy věty určete tok vektorového pole F = (x3, y3, z3) plochou, určenou předpisem S = {(x, y, z) | x2 + y2 + z2 = i?2 }. $f = — 7tí?5 5 7.48 Je dáno silové pole F = (x3 — x2, y3 — y2, z3 — z2). Pomoci Gaussovy věty určete jeho tok povrchem tělesa, určeného předpisem V = {(x, y, z) \ x, y, z G (0,i?), x2 + y2 + z2 < R2}. Kapitola 7. Integrálni věty 93 $F = _(4Ä-5) 7.49 Pomocí Gaussovy věty vypočítejte tok vektorového pole F = (x, y, z) uzavřenou plochou, tvořenou povrchem tělesa z příkladu 7.35. Proč je výsledná hodnota trojnásobkem výsledné hodnoty z uvedeného příkladu ? 37rV &F = - 4 7.50 Pomocí Gaussovy věty vypočítejte tok vektorového pole F = (x3, y3, z3) uzavřenou plochou, tvořenou povrchem tělesa z příkladu 7.35. 3 27 „ K /3 )p = 64" ~ V 4 $p = Zl ^5 = ^ 7r2a5 7.51 Pomocí Gaussovy věty vypočítejte tok <í> vektorového pole F(x, y, z) = (0, 0, z2) uzavřenou plochou, tvořící povrch tělesa: V = {(x, y, z) | x2 + y2 < 4, x < 0, y > 0, z £ (0, |x|}}. 7.52 Pomocí Gaussovy věty vypočítejte tok <3? vektorového pole -F(x, y, z) = (0, 0, z2) uzavřenou plochou, tvořící povrch tělesa: V = {(x, y, z) | x2 + y2 + z2 < 4, x > 0, y < 0, z > 0, z2 1 x2 + y2 + z2 2 3>F = 7T 7.53 Pomocí plošného integrálu 2. druhu i pomocí Gaussovy věty vypočítejte tok <í> vektorového pole F(x,y,z) = (0,0, z2) uzavřenou plochou, tvořící povrch tělesa: V = {(x, y, z) | x2 + y2 + z < 9 A z - 3x2 - 3y2 > 0}. 9X 2 $F = 21tt í- 7.54 Pomocí plošného integrálu 2. druhu i pomocí Gaussovy věty vypočítejte tok <í> vektorového pole F(x,y,z) = (x2,0,0) uzavřenou plochou, tvořící povrch tělesa: V = {(x,y,z) | x2 + y2 + z < 5, x > 0, y < 0, z > l}. 56 ä>F = — 5 7.55 Určete kapacitu válcového kondenzátoru, který tvoří dvě souosé vodivé válcové slupky (elektrody) s poloměry R\ a i?2 a délkou ií, kde i?i < i?2- Na vnitřní elektrodu je přiveden náboj +Q, na vnější elektrodu náboj —Q. Zanedbejte nepravidelnosti elektrického pole na obou koncích elektrod. 27re0H C \n{R2/Ri 7.56 Určete kapacitu kulového kondenzátoru, který tvoří dvě soustředné vodivé kulové slupky (elektrody) s poloměry R\ a i?2, kde R\ < i?2. Na vnitřní elektrodu je přiveden náboj +Q, na vnější elektrodu náboj —Q. Kapitola 7. Integrální věty 94 C = 47re0 R1R2 R2 — R\ Kapitola 8 Taylorův rozvoj Možnost nahrazení libovolné matematické funkce polynomem byla formulována počátkem 18. století matematiky Jamesem Gregorym a Brookem Taylorem. V případě nekonečněkrát diferencovatelné funkce půjde o nekonečnou mocninnou řadu. Rozvoj funkce do řady je jedním z nejpoužívanějších nástrojů pro vyjádření přibližné hodnoty funkcí, který tvoří základ mnoha principů numerické matematiky, atd. 8.1 Rozvoj funkce jedné proměnné Obecný zápis Taylorova rozvoje nekonečněkrát diferencovatelné funkce jedné proměnné v obecném bodě xq lze vyjádřit pomocí nekonečné řady x0 * 1 d2f Xo) + 2! x0 1 d4/ 4! <9x4 x0 ^2 1 93f {X~X0) +3! W n\ dx (x - XqY x0 (x-x0) + ... =^__ n=0 (x-x0)n, 1.1) x0 kde řád derivace charakterizuje řád Taylorova rozvoje, stupeň mocniny určuje stupeň členu Taylorova polynomu (v případě funkce jedné proměnné se obojí shoduje). Položíme-li xq = 0, dostáváme tzv. Maclaurinovu řadu (rozvoj) jako speciální případ Taylorova rozvoje. • Příklady: 8.1 Rozviňte následující neurčité integrály do Taylorovy řady (b) (d) ~x dx, do x sin x ■ dx, x 1" + £1Í!2Í + C do ™2fc+l toK ' (2fe + l)!(2fe + l) + C cosx dx, sm x cos x do J2k + C dx. fc=0 -D i + c 2k + 1 „^0 ml (2k-m + 1)1 1.2 Napište rozvoj následujících funkcí do 4. stupně v bodě xo = 0 (Maclaurinův rozvoj): 95 Kapitola 8. Taylorův rozvoj 96 (a) f (x) = e3x, (b) /(x) = x2~X + 1, JV 7 2x + l ' (c) /(x) = ln (l — sin2 x) 9 27 1 + 3x + - (x2 + x3) H--x4 + 0(x5) 2 8 1 - 3x + 7x2 - 14x3 + 28x4 + O (x5 (d) f (x) = ex sin x, sinh (x2 + 2 sin4 x) x + x2 H---h C(x5) 3 /(*) 1 +x10 (f) /(x) = \Jcos (3x + x3). x2 + 2x4 + C(x6 9 „ 75 1 - -xz x4 + C(x6 4 32 .3 Pomoci Taylorova rozvoje určete hodnoty uvedených limit následujících funkcí: (a) lim ex — 1 — x x^o xz (b) lim e — srn x — cos x x—>o e — e 1 - ex (c) lim . / . . , x^o V ln (x + 1) (d) hm x^O (ej hm (f) lim x^O Vi — 5x2 + x4 — 1 + x2 x4 1 sin 3/ o" 3/ ô" _ a / _ ln cos x x sin x ln( 1 + x arctgx) + 1 - ex2 y/l + 2x4 - 1 cos x — 1 H—x sin x (g) Hm 2 (h) lim x—>0 *->o [ln (1 + x)] 1 1 4 ' 9 "Š 1 3 4 ~3 1 24 1 2 ln (1 + x) tgx .4 Vypočítejte přibližnou hodnotu následujících integrálů s chybou nepřevyšující 10~3 2 fx 2 (a) tzv. chybové funkce erfx = —= \ e~* dt pro horní mez x = 1, 0,842 714 v71" Jo 222 ^ sin t (b) tzv. integrálního sinu Si x = / ^ dt pro horní mez x = 1, 0,946 082 766 o t (c) tzv. integrálního kosinu Cix cosi dt (x > 0) pro spodní mez x = 1. Tento r'x cos t - 1 integrál lze přepsat do tvaru Cix = 7 + lnx + / -dt, kde tzv. Eulerova Kapitola 8. Taylorův rozvoj 97 / n l \ (Eulerova-Mascheroniho) konstanta 7 = lim ( Yl--lnn ~ 0,577215 665, \k=i « / 0,337400 849 r00 e-t rx et (d) tzv. exponenciálního integrálu Eix = — / -dt = — dt pro x = — 1. Tento ./ — x t J — oo i r—x e-t _i integrál lze přepsat do tvaru Eix = 7 + ln\x\ + / -dt, kde 7 je stejná Jo ŕ Eulerova konstanta jako v příkladu 8.4c, -0,219 386 753 (e) exponenciálního integrálu Eix popsaného v příkladě 8.4d, kde hodnota meze x = 1 (tento případ má konečné řešení, protože integrování funkce se singularitou lze za určitých podmínek provést přiřazením tzv. hlavní hodnoty určitého integrálu), 1,894 854 554 fX2 dt (f) tzv. integrálního logaritmu li x = / -pro x\ = 2 a X2 = 10 (integrovanou funkci JXl lni lze rozvinout do vhodné řady pomocí substituce t = éa). 5,073 622 569 .5 Pomocí Taylorova rozvoje dokažte Eulerovu identitu pro y{x): Ci elx + C2 e~lx = A cos x + Bsinx. Jaký je vztah mezi jednotlivými koeficienty a čemu se budou rovnat, pokud y(0) = 1, y'(0) = 1 ? A = Ci + C2, B = i(Ci - C2), Ci = C2 = ±±±, A = 1, B = 1 .6 Ověřte platnost klasického vztahu pro kinetickou energii T = ^mv2 pro malé rychlosti, v 2/ [x — xq) - dx dy (x - x0f + (x - x0)(y - y0) + d2f %o,yo dy2 (y - yo) %o,yo dx2 dy (x - x0)2(y - y0) %o,yo + 3d3f dx dy2 (x - x0)(y - y0)2 + d3f %o,yo dy3 (y - yo) %o,yo + ... , (8.2) kde řád derivace opět charakterizuje řád Taylorova rozvoje, stupeň mocniny určuje stupeň členu Taylorova polynomu. Obecně lze tedy Taylorův rozvoj funkce více proměnných zapsat: oo oo f(xu...,xk) = H H ni=0n2=0 nfe=0 £piH-----hnfe ý dxl1 ■ ■ ■ dx\k {xi - xQ1)ni ■■■{xk- xok) (xoi,...,xok) n\\■■■nk\ (8.3) • Příklady: .9 Spočítejte všechny nenulové členy Taylorova rozvoje funkce f(x, y) = x2y a pro jednotlivé řády rozvoje vyčíslete vždy hodnotu / (2,1; 2,9). Výsledky porovnejte s hodnotou, udanou kalkulačkou. TQ = 12, Ti = 12,8, T2 = 12,79, T3 = 12,789 (kalkulačkou 12,789) ^.10 Pomocí Taylorova rozvoje funkce f(x,y) = \/l + Ax2 + y2 do prvního, druhého a třetího řádu vyčíslete vždy přibližnou hodnotu /(1,1; 2,05). Výsledky porovnejte s hodnotou, udanou kalkulačkou. Ti = 3,166, T2 = 3,169120, T3 = 3,168 984 (kalkulačkou 3,168 990...) Kapitola 8. Taylorův rozvoj 99 .11 Spočítejte všechny nenulové členy Taylorova rozvoje funkce f(x,y,z) = x3 + y3 + z3 — 3xyz a pro jednotlivé řády rozvoje vyčíslete vždy hodnotu / (0,95; 1,05; 1,1). Výsledky porovnejte s hodnotou, udanou kalkulačkou. T0 = 0, Ti = 0, T2 = 0,0525, T3 = 0,054 25 (kalkulačkou 0,054 25) .12 Pomocí Taylorova rozvoje funkce fix, y, z) =-do prvního, druhého, třetího a čtvrtého xyz řádu vyčíslete vždy přibližnou hodnotu / (0,9; 2,1; 3,1). Výsledky porovnejte s hodnotou, udanou kalkulačkou. Ti = 0,169444, T2 = 0,170 602..., T3 = 0,170 634..., T4 = 0,170 648..., (kalkulačkou 0,170678...) .13 Vypočítejte Taylorův polynom druhého stupně funkce f(x,y) = e~(x2+y2) v bodech (a) Pi = (0,0), (b) P2 = (l,2). (a) Ti(0,0) = l-x2-y2 (b) T2(l, 2) = e"5 [x(x + 8y- 20) + y(7y - 40) + 56] .14 Vypočítejte Taylorův polynom funkce z příkladu 8.10 (a) třetího stupně v bodě (0,0), (b) druhého stupně v bodě (1, 2). ř_ 2 (a) T3(0,0) = l + 2x2 + y (b) T2(l, 2) = i [4x + y - 8(x - l)(y - 2) - 5] + A 2(x-l)2 + -(y-2)2 .15 Vypočítejte Taylorův polynom třetího stupně funkce z příkladu 8.12 v bodě (1,1,1). T3(l, 1,1) = -x3 — y3 — z3 — x2y — x2z — xy2 — xz2 — y2z — yz2 — xyz + 6(x2 + y2 + z2 + xy + xz + yz) — 15(x + y + z) + 20 .16 Napište Taylorův polynom 2. stupně funkce f(x,y) = \/x2 — y2 — 2 v bodě (2,1). 2x - y - 2 + i [-3(x - 2)2 + 4(x - 2)(y - 1) - 2(y - l)2] .17 Napište Taylorův polynom 2. stupně funkce f(x,y) = \Je2x — y2 + 1 v bodě (0,1). 2 + x - y + I [x2 + 2x(y - 1) - 2(y - l)2] .18 Napište Taylorův polynom 2. stupně funkce f(x,y) 1 1 v bodě (2,1) x 1 H---y + - 2 y 2 (x-2)2 + (y-l) Kapitola 8. Taylorův rozvoj 100 8.19 Napište Taylorův polynom 2. stupně funkce f(x,y) = ■. v bodě (0,1). \J x2 + 1 l + 2(y-l) + i[-x2+2(y-l)2] 8.20 Napište Taylorův polynom třetího stupně funkce f(x,y,z) = —^— v bodě (1,1,1). xzyz T3(l, 1,1) = -4x3 -y3 - z3 - 3x2y - 3x2z - 2xy2 - 2xz2 - 2y2z - yz2 - 2xyz + 7(3x2 + z2 + 2xy + 2xz) + 8y2 + 9yz - 2{2\x + liz) - 23y + 36 Kapitola 9 Fourierovy řady Metoda rozkladu obecných periodických funkcí na součet nekonečného počtu sinových a kosinových vln byla pojmenována po francouzském matematiku Jean-Baptiste Josephu Fourierovi (1768-1830). Fourierovy řady jsou v různé míře aplikovány ve většině fyzikálních oborů, např. v akustice, optice, kvantové fyzice, atd. Princip formulovaný nejprve pro Fourierovy řady byl později zobecněn v tzv. Fourierově analýze. Libovolná periodická funkce f(x) s periodou T, integrovatelná v intervalu (xq,xq + T), může být vyjádřena jako následující nekonečná suma (Fourierova řada): oo f(x) =--h &k cos(kujx) + bfc sm.(kujx), (9.1) k=l kde uj = 2tv/T. Fourierovy koeficienty a&, b& lze stanovit následujícím způsobem: a0 = -/ f(x)dx, (9.2) 2 rxo+T ak = — f(x) cos(kujx) dx, (9.3) 2 rxo+T bfc = — / f(x)sm(kujx)dx. (9.4) Fourierova transformace je zobecněním komplexních Fourierových řad. Nahradíme-li diskrétní Fourierovy koeficienty afc,6fc spojitou funkcí F(£) d£, potom za předpokladu 1/T —> £ (frekvence) přejde (záměnou sumy za integrál) diskrétní Fourierova řada do spojité podoby /*oo poo F(Z)= f(x)e-2^xdx, f(x)= F(0e2^dí. (9.5) Ve fyzice a v technických aplikacích se Fourierova transformace zapisuje častěji pomocí úhlové frekvence u = 2tv£. Fourierova transformace J~(f) = f (kde / je tzv. Fourierův obraz funkce /, tj. vzoru) a zpětná Fourierova transformace Jr_1(/) = / jsou potom (při jisté ztrátě symetrie) definovány jako: /oo -i poo f(x) eT™ dx, /(*) = — / f(u) du. (9.6) oo ^ J—oo Zavedeme dále pojem konvoluce dvou funkcí f(x),g(x) (viz obrázek 9.1), která je definována jako: {f*9){x)= f(y)g(x-y)dy, (9.7) 101 Kapitola 9. Fourierovy řady 102 kde ovšem x a y neznamenají dva různé souřadnicové směry, ale pouze dvě různé proměnné. Fourierův obraz konvoluce funkcí f(x),g(x) potom bude: Pomocí transformace oo roo (f*g)(x)e-i^dx x — y = z y = y jejíž Jakobián det roo f(y)g (x J — oo dz dz dx dy dy_ 9y_ dx dy det 1 -1 0 1 1, dostáváme — oo J —oo f(y)g(z)e-i^y+^dydz= /(y) dy / g(z) e^z dz = f(u)g(u). (9.9) ) J —oo j—oo Výsledný vztah můžeme tedy zapsat jednoduchým způsobem, íf^g) = fg, (9-10) Fourierův obraz konvoluce dvou funkcí f(x), g{x) se rovná součinu jejich Fourierových obrazů. Příklad konvoluce dvou funkcí f * g, které jsou původně zadány například ve tvaru /(*) g\x) 0 x G (-oo, 0) 3e~x x G (0, oo), 0 x G (-oo, 0) 1 x G (0, 2) 0 x G (2, oo), (9.11) (9.12) znázorňuje obrázek 9.1. Funkce /, g transformujeme podle rovnice (9.7) následujícím způsobem: f{x)=Se-x g(x) f(y)=3e-y, g (x-y) = g (z) (9.13) (9.14) kde potom pro funkci g (z) (konvoluční jádro), platí g (z) = 1, pro z G (0,2) (a tedy y G (x, x — 2)) a g (z) = 0 pro z ^ (0,2) (a tedy y ^ (x, x — 2)), zároveň také platí dz = —dy. Protože f (y) = 0 pro y < 0, dostáváme tak tři oblasti integrace rovnice (9.7): x-2<0Ax<0 i-2<0Ai>0 i-2>0Ax>0 (obrázek 9.1a), (obrázek 9.1b), (obrázek 9.1c). (9.15) (9.16) (9.17) Integrace rovnice (9.7) bude mít pro tyto tři oblasti podobu: 0 pro x G (—oo, 0) (f*9) (x) e~y dy = 3 (l - e" pro x G (0,2), (9.18) ľX 3/ e~ydy = 3e~x (e2 - l) pro x G (2, oo). Jx-2 Výsledkem konvoluce dvou funkcí f{x) a g{x) bude tedy funkce (/ * g) (x), jejíž hodnota se pro každé x G (—oo, oo) bude rovnat velikosti zvýrazněné plochy na obrázku 9.1. Kapitola 9. Fourierovy řady 103 Obrázek 9.1ě Obrázek 9.1b Obrázek 9.1c Obrázek 9.1: Schéma konvoluce / * g funkcí f(x), g(x), původně popsaných rovnicemi (9.11) a (9.12). V obrázku 9.1a platí, že v rovnicích (9.13), (9.14) je x e (—oo, 0), v obrázku 9.1b je x E (0, 2), v obrázku 9.1c je x G (2,oo). Tvar výsledné funkce (f * g) (x) je zakreslen modrou barvou, funkce je popsána v rovnici (9.18), její hodnota se pro každé x bude rovnat velikosti zvýrazněné plochy. 9.1 Diskrétní Fourierovy řady • Příklady: Napište Fourierovu řadu pro následující periodické funkce s periodou T: 9.1 f(x) = —, x G (-7r,7r), T = 2ir 7T 2 9.2 = —, x G (0,2tt), T = 2tt 9.3 f(x) = \x\, x G (-vr, vr), T = 2tt 9.4 f(x) = x|x|, x G (-L, L), T = 2L 9.5 /(x) = ä,xG<-l,l),T = 2 7T 4 00 (~l)fc ô + ~ S ^2— COs(fcx) 1 4?r ~ á fc=i cos(fcx) — — sin(fcx) 7T 2 °° ("l)fc-l /; , ô + ~ L -Ta-COs(fcx 2 7T fc=1 F 2L2 °° (2-7T2fc2)(-l)fc-2 /fcTT —s- 2^ -—-sin I -^x fc=l k3 L 2 ~ (2-7r2fc2)(-l)fc-2 . —r V--sin (K7rxj 7Td k=1 kó 9.6 /(x) 0 x G (0,1) 1 °° x- 1 x G (1,2) ' 4 ^ fc=i l-(-l)fe 1 - - COs(fc7Tx) —-- SÍn(fc7Tx) 9.7 /(x) = ea:E, x G (-7T, 7r), konstanta a / 0, T = 2tv — sinh(a7r) ^~ + Yl —-Ta (acos(^x) ~ fcsin(fcx)) 2 A 9SL (-1) 9.8 f (x) = (x - l)(x — 3), x G (1, 3), T = 2 -- + — £ 1—i- cos(fc7rx 9.9 /(x) = —, xG (0,2L),T = 2L 3 vr2 fc=1 fc2 1 1^1 / fcTT ---> — sin —x fc V L Kapitola 9. Fourierovy řady 104 9.10 f (x) = x4, x G (-1,1), T = 2 9.11 f (x) = sgn cos (kux) sm VTX T 4 oo 1 - E —-r srn VT fcf 1 2fc + 1 9.12 /(x) = , x G (0, 2L), T = 2L (2fc + l)vT L x x G (-1, 0) x x G (0, 1) 1 x G (O, 1 4 oo x T = 2 ---77 Y)---T7 cos \(2k + l)vrxl 9.13 /(x) = { o oo 1 1 ie(~0l, T = 2vr - E T 9.14 /(x) = < 2 0 |X|G(|, 7T 0 x G ( -1, , 1 1 COS 3 7TX x G ( — —, — , , 0 x G ( i 1 fc=i /fcvr\ 1 — cos — v 2; sin(fcx) T = 2 1 6 cos(27rx) cos(37rx) 6 1 --1----- H----- H— V - 3tt 5tt 2 vr fce4 fc2 - 9 V 2 y COS ( - J COs(fcvrx) 9.15 /(x) 9.16 /(x) 9.17 f(x) 1 x G (-1,0) - x = 0 2 x x G (0,1) 4 / 7T —x x G ( 0, — 7T \ 2 4 / 7T --x x G (--, 0 VT \ 2 T = 2 T+ E -l)fc - 1 i fc2VT2 COs(fcvrx) —-- SÍn(fc7TX kir cos [2(2k - l)x] vr2^ (2fc-l)2 x (1 - x) x G (0,1) 0 x G (-1,0) T = 2 12 VT2 1 fc2 9.18 /(x) = |sinx|, x G (-vr,vr), T = 2vr 9.19 /(x) = |cosx|, x G (—vr,vr), T = 2vr 9.20 /(x) = sinxcosx, x G ( — 1,1) 2 fc3vr (1- cos fcvr) sin(fc 2 4 oo E fc=i 1 vr vr (4fc2 - 1) 2 4 oo E fc=l {-lf vr vr (4k2 - 1) cos(2fcx) cos(2fcx) oo k(—l)k vrsin(2) Y--—r-^ sin(fcvrx) k=i 4 - k k Kapitola 9. Fourierovy řady 105 9.21 f (x) = \x\ + a, x G (—L, L), kde a je konstanta. L 2L °° (-l)fc - 1 /fcvr . . í x — a x G (—L, 0) 9.22 f (x) = < , kde a íe konstanta. JK ' | x + a 16(0,1) 2 ~ a- (a + L)(-l)fc . /kir - E -;-sin —: 7T k=1 k \L 9.2 Fourierova analýza • Příklady: Určete konvoluci funkcí (/ * g)(x): 9.23 f (x) 9.24 /(x) g(x) ■ 9.25 f (x) g(x) 9.26 /(x) g(x) 9.27 /(x) 0 x G (-oo, 0) 2 x G (0,oo) 0 x G (-oo, 0) 1 x G (0, oo) 0 2.ľ 0 J- x G (—oo, 0) x G (0, oo) x G (—oo, 0) x G (0, oo) 0 x G (-oo,0) e~x x G (0, oo) 0 x G (-oo, 0) sin x x G (0, oo) 0 x G (-oo, 0) sin x x G (0, oo) 0 x G (-oo, 0) sin x x G (0, oo) 0 x G (-oo, 0) 1 16 0, 0 7T 7T f*9 f*9 f*9 f*9 f*9 0 x G (-oo, 0) 2x x G (0, oo) (1 0 x G (-oo, 0) i~x) xG(0,oo) 0 x G (-oo,0) - (e~x + sin x — cos x) x G (0, oo) 0 x G (-oo, 0) 1 — cos x x G (0, oo) x G x G -oo,0) Tľ "'2 '7T I 2'°° Kapitola 9. Fourierovy řady 106 9.28 f (x í 0 x G (-oo,0) 1 sin x G (0, oo) g{x) = sin(ax), a > 0 f*9 a2 - 1 9.29 f (x) 0 x G (-oo, 0) sin x x G (O, oo) O x G (-oo, 0) 2 1--x 7T x G (O, 7T /*5 O x G (-oo, 0) [a sin x — sin(ax)] x G (O, oo) O x G (-oo, 0) 1--x H--sin x — cos x x G ( 0, — (smx + cos x) — cos x x G 0 x G | -, oo 7T Tľ oc 9.30 Určete Fourierův obraz funkce: 1 1 1 1 (a) f (x) = 1 pro x G -y, /(x) = 0 pro x ^ 2' 2 ;' (b) /(x) = e~b:E pro x G (0, oo), f (x) = 0 pro x G (—oo, 0), b > 0 = konst., (c) f (x) = e~h\x\ pro x G (—oo, oo), (d) f (x) = e~hx2 pro x G (—oo, oo), (e) f (x) = 1 + x pro x G ( — 1, 0), f (x) = 1 — x pro x G (0,1), f (x) = 0 pro |x| > 1, (f) f (x) = x2 pro x G ( — 1,1), f (x) = 0 pro |x| > 1. (a) f (u) (b) m (c) m (d) m (e) /H (f) m 2 — sm — u V2 1 b + ílú 2b b2+W2 e 4b u' 2 ^3 (1 — cos uj) [{u2 - 2)sinw + 2u cos u 9.31 Určete Fourierův obraz funkce: (a) f (x) = sin x pro x G (0,27r), f (x) = 0 pro x ^ (0,27r), /(w) (b) f (x) = cos x pro x G (0, 2*7r), /(x) = 0 pro x ^ (0, 27r), f (u) (c) /(x) = sin x cos x pro x G (0, 2tt) , /(x) = 0 pro x ^ (0, 27r), /(w) ícj (1 — e — 27TÍWNl 1 -U2 — e~ 1 -e — 27TÍUJ u2 -4 Kapitola 9. Fourierovy řady 107 2i d-e-2™l (d) f(x) = sin2 x cos2 x pro x G (0, 27r), /(x) =0 pro x ^ (0, 27r), f(u) — íú \íú — loj — 1 — e_iwylB (e) f(x) = ABx pro x G (0,1), /(x) = 0 pro x ^ (0,1). f (u) ílo - B ln A 9.32 Na příkladech 9.23, 9.24, 9.25 a 9.26 ověřte platnost rovnice (9.10): (a) (9.23): f (u) = --, g{u>) = --, (]Tg){u>) = --\ (b) (9.24): f (u) = g(u) = (7^fl)M = n , ! 2 + ícj 1 + \uj 2 — oj2 + 3ku (c) (9.25): /M = ^ K«) = i^. (7^)M=(1 + i^(1_^ (d) (9.26): /M = —í-^, s(w) = 1, (Tmí) M 1 —w2' ícj' ííj(1 — w2) Kapitola 10 Uvod do komplexní analýzy Algebraický zápis komplexního čísla (komplexní proměnné) zéC má tvar z = x + iy, (10.1) kde x = Re(z) je reálná část komplexního čísla a y = Im(z) je imaginární část komplexního čísla. Číslo z* (značí se také ž) nazýváme číslem komplexně sdruženým k číslu z, kdy z* = x — iy. Zápis stejného čísla lze provést v goniometrickém, případně exponenciálním tvaru, z = r(cos tp + isin + 2kiv) ] + i sin [m (y> + 2kiv) ] } , fceZ. (10.3) Předpokládejme funkci jedné komplexní proměnné f (z) = u(x,y) + \v(x,y) definovanou na oblasti G : x\ < x < x2, y± < y < y2, případně v polárních souřadnicích f (z) = u(r,ip) + \v(r, ip), kdy G : r\ < r < r2, tpi < tp < ip2. Derivaci funkce jedné komplexní proměnné f (z) definujeme potom na této oblasti jako limitu f(z) = i dz"2 z — 1 (z-i)(z+i) lim 2 z^i (z + i) 3i_ 16 (10.15) a dosazením do rovnice (10.11) dostáváme dx 3tt [1 + x2)3 (10.16) • Příklady: 10.1 Zadaná komplexní čísla napište vždy v ostatních tvarech (algebraickém, goniometrickém nebo exponenciálním): (b (d (f (h 5^/3 + 51 3^/2 + 3^/21 -12 1 -6 + 6i V3 2 (1 + VŠi) 5tt 5tt 3 cos--h i sin — 4 4 7T 7T 12 cos--i sin — 6 6 -2 cos--h i sin — 6 6 o , vr 5?r á cos--h i cos — 3 6 7T 7T 10 cos--h i sin — 6 6 6 cos--h i sin — 4 4 8^/3 57T 5ir cos--h i sm — 6 6 12 / 5tt 5tt —= cos--h i sin — VŠ\ 6 6 4?r 4tt\ 4 I cos--h i sm — I 3 3 / 6^/3-61 -(V3 + i) 3 'l-y/Ži 10eT 7TÍ 6e 4 12 5?M -=e 6 4ir\ 4e~ 5iri 3e~ 12e_T 7-kí 2eT 5iri 2e~ Kapitola 10. Úvod do komplexní analýzy 112 (j) 27/10 ( sin — ± i cos — u; 1 20 20 (k) 3 e- 2ttí 3 (1) v^eiir (l + i)7/5 -f (i + Vši) (i + i)7/3 27/10eW 4?r 4tt \ 3 ( cos — + 1 srn — I 7T 7T\7/3 V2 ( cos — + i sin — J 10.2 Napište součet, součin a (a) z\ = 2 + 3i, Z2 = 7 — i zi (b) zi = 12 + i, Z2 = 6 - 3i z\ + z2 (c) zi = 7 + 3i, z2 = 3 - 3i z\ + z2 (d) = 2 + 12i, Z2 = 5 - i + z2 = 11 (e) zx = 1 - i, 4 (f) z\ = 2 I cos podíl následujících komplexních čísel: + z2 = 9 + 2i zxz2 = 17 + 19i = 18 — 2i z1z2 = 75 - 30i = 10 z1z2 = 30 - 12i = 7 + lli ziz2 = 22 + 58i = 6 - 16i + 5i + z2 = 12 — 6i zi^2 2tt Zl 11 + 23Í Z2 50 Zl 23 + 14i Z2 15 Zl 2 + 5i Z2 3 Zl -1 + 31i Z2 13 Zl 8 — 3i Z2 73 Zl+Z2 = ( ^ z\ = 3 I cos 7T 7T\ / 2lV 2tv\ -+isin-), ,2 = 3^cosY+isinYj -1 + 5VŠÍ Z\Z2 = -6 £1 Z2 1 - VŠi 7T 7T\ ( 5lV --i sin — , Z2 = 2 cos--i sin , 6 6/ V 3 3 i 57T . . 57r\ 3-y/3 / r- zi + z2 = — + 1 + ( V3 - o, /ô,-\ zi - (2 + 3VŠ)i Z1Z2 = 3(V3 + 1) — =-^--- Z2 8 10.3 Napište následující mocniny (odmocniny) komplexních čísel: (a) (1+if (b) (-V3 + Í)8 (c) VI (d) VŤ29 (e) V-2 + 2i 8(1-i) 128(-1 + VŠi) 1, -|(l±>/3i) ±3, ±^(1 ± VŠi) (f) ^TTTIi VŠ V2 /3tt \ /3tt ; COS--\-ktv ±lSin--\-ktt V 8 J I 8 7T 2. \ ítt 2, COS|--h — ktv +1S1I1--\- — ktt 15 5 / \15 5 , k = 0,1 k = 0,1,2,3,4 Kapitola 10. Úvod do komplexní analýzy 113 (g) Í75 15i 71 '10 ,5tt 2; \ /5tt 2; COS--\- — klľ\ +1S1I1--h" KTľ 9 3 V 9 3 (h) -5 + 11 71 100 ,'5tt 2; \ /5i 2, COS--h-fc7T + 1SU1--\- — KTľ 18 3 V18 3 i) W-6 + _6_i_ 71 1/2 I -—\-kir| +isin í-—hfc7r 12 V12 , fc = 0,1,2 , fe = 0,1,2 k = 0,1 10.4 Ověřte, jestli mohou být následující komplexní funkce /(z) otevřených podmnožinách komplexní roviny: f{x + iy) holomorfní na (a) 3y — 3xi (b) 3x2 + 3y + 6xyi (c) z2 +lnz + 1 (d) z3 + 5z — sin z (e) k2 + y| (f) z - 1 z + 1 (g) V z + 1 + i (h) exp (- iz2) W exP 0) in + z + 1 ano ne ano ano ne ano ano ano ano ano 10.5 Nalezněte holomorfní funkce komplexní proměnné f (z), pokud je zadaná pouze Re [/(z)] u(x,y) nebo pouze Im[/(z)] = v(x,y): u = x e' 3y (b) u = x2 + 3x — y2 + 5y (c) u = ex(cosy + 2 sin y) (d) u = sin(2x) cosh(2y) (e) u = x2 — y2 + sin(x) cosh(y) (f) u = x3 — 3xy2 + ln \z\ u = x x2 + y2 holomorfní funkce f (z) neexistuje f (z) = z2 + 3z - 5iz + C 9y f (z) =sin2z + C f (z) = z2 +smz + C f (z) = z3 + lnz + C, ZytO f(z)=z--z+C,z^0 Kapitola 10. Úvod do komplexní analýzy 114 (h) v = y — sin(x) sinh(y) f (z) = z + cos z + C (i) v = x + sinh(x) sin(y) f (z) = iz + cos íz) + C 10.6 Pomocí reziduové věty vypočítejte následující integrály (kde C značí uzavřenou křivku obsahující všechny singularity dané funkce f (z), C+ značí uzavřenou křivku obsahující pouze jednu libovolnou singularitu dané funkce): z dz 2tt\ C z2 - 1 z sin z sinh y/5 az 2tt\ ■ z2 + 5 V5 c e C 30 dx (d) (/) -g--dz 27risin(l) X2 + 1 7T , . , SIM 7T , , (f) / -5----di -siní COS X 7T / 1 dx —- cos _00 6x2 + 6x + 3 3y/e \2 r°° dx 27T (h)L^T^ (v-pHldadl.31) ^ 4dx 57T , (x2 + 4)4 512 Kapitola 11 Kombinatorika Kombinatorika je jednou z nejstarších matematických disciplín, která se zabývá vnitřní strukturou tzv. konfigurací diskrétních prvků (například čísel nebo jiných objektů), jejich existencí, hledáním počtu různých typů těchto prvků v závislosti na daných podmínkách, atd. Typickými příklady takových konfigurací jsou kombinace, permutace a variace. Kombinatorické principy tvoří matematický základ definice tzv. statistických rozdělení, používaných k popisu chování fyzikálních systémů například v kvantové mechanice, statistické fyzice, atd. • Kombinace bez opakování: Počet fc-členných kombinací (kombinací k-té třídy) bez opakování z n prvků (k,n £ N U 0), tj. každý prvek se v dané kombinaci může vyskytovat pouze jednou, je dán vztahem kdy poslední výraz v závorce, tzv. kombinační číslo, čteme „n nad h". Kombinační číslo je také určujícím faktorem v tzv. binomické větě, pomocí níž lze n-tou mocninu dvojčlenu x + y rozložit na součet n + 1 členů. Kombinace bez opakování je matematickým základem tzv. Fermiho-Diracovy statistiky, popisující systémy složené z tzv. fermionů, tedy ze vzájemně nerozlišitelných kvantových částic s antisymetrickou vlnovou funkcí a poločíselným spinem (například protonů, neutronů, elektronů, neutrin, atd.). Typickým jednoduchým příkladem může být počet různých dvojic, které lze vytvořit z celkového počtu 30 lidí, kde podmínka „bez opakování" vyplývá z faktu, že každý určitý jedinec se v každé dvojici může vyskytovat pouze jednou. Zároveň lze na kombinaci pohlížet jako na variaci kdy tzv. nezáleží na pořadí prvků, tj. dvojice A-B je totožná s dvojicí B-A. Výsledkem je číslo 435. • Kombinace s opakováním: Počet fc-členných kombinací s opakováním z n prvků, tj. daný prvek se v dané kombinaci může vyskytovat vícekrát (přičemž opět nezáleží na pořadí prvků), je dán vztahem 115 Kapitola 11. Kombinatorika 116 Kombinace s opakováním je matematickým základem tzv. Boseho-Einsteinovy statistiky, popisující systémy složené z tzv. bosonů, tedy ze vzájemně nerozlišitelných kvantových částic se symetrickou vlnovou funkcí a celočíselným spinem (například fotonů, mezonů, gluonů, jader 4He, atd.). Typickým příkladem může být počet různých způsobů, kterými lze koupit sadu osmi sazenic salátu, pokud mají v obchodě (v dostatečném množství) 6 různých druhů sazenic (v každé sadě se může kterýkoli z šesti druhů sazenic vyskytovat v libovolném počtu od 1 do 8). Výsledkem je číslo 1287. • Permutace bez opakování: Obecně definujeme permutaci jako uspořádanou n-tici prvků, kdy celkový počet prvků výběrové množiny je rovněž n. Pokud se tyto prvky v každé takové uspořádané n-tici nemohou opakovat, počet různých takových n-tic (permutací bez opakování) je dán vztahem P(n)=nl (11.4) Příklad: kolik různých uspořádání, obsahujících vždy všechna písmena, existuje pro pětici písmen a, b, c, d, e? Počet takových uspořádaných pětic (n = 5) bez opakování je dán vztahem P(5) = 5!, celkový počet takových uspořádání (permutací) je tedy 120. • Permutace s opakováním: Pokud je mezi n prvky výběrové množiny k skupin, které mají postupně ri\, n2, ..., nk stejných prvků, potom je počet tzv. permutací s opakováním dán vztahem k Ti' K—T PÁ1,n2,...,nk(n) = —,-i"-Ť My n=\ni. (11.5) '■' ' k n\\■ n2\ ■ ... • nul L—' i=l Příklad: kolik různých permutací existuje pro sedmiprvkovou množinu čtyř písmen s možným opakováním a, a, a, b, b, c, d, kdy první písmeno se zde vyskytuje třikrát a druhé písmeno dvakrát? Celkový počet takových permutací bude 7!/(3! • 2! • 1! • 1!) = 420. Výraz (11.5) tvoří rovněž matematický základ zobecněné binomické věty (11.2) pro libovolný počet členů x\ + x2 + ... + xm, kdy pro tzv. multinomický koeficient n \ = n! ki, k2, ..., kmJ kil k2\... km\ musí pro všechna m£Nafcj,n£NU0 opět platit k\ + k2 + ... + km = n. Takto rozšířenou binomickou větu (11.2) potom zapíšeme jako tzv. multinomickou větu ve formě (x1+x2 + ... + xm)n= {k^k^.^kj^^---*™' (11-7) ki+k2 + ...+km=n kde součin m prvků ^™ lze zapsat pomocí symbolu pro násobení jako fl^Li xi ■ • Variace bez opakování: Variaci definujeme obecně jako uspořádanou fc-tici (tj. fc-tici, ve které tzv. záleží na pořadí prvků), vybranou ze sady, obsahující n prvků. Pokud se tyto prvky v každé takové uspořádané Kapitola 11. Kombinatorika 117 fc-tici nemohou opakovat, počet různých takových fc-tic, k < n (variací bez opakování), je dán vztahem V{k-") = J^W: <1L8) Typickým příkladem může být následující úloha: kolik barevných trikolór lze vytvořit z celkem šesti barev? Variace bez opakování v tomto případě vyplývá z definice trikolóry (pokud by se některá ze tří barev opakovala, nepůjde o trikolóru). Zároveň záleží na pořadí jednotlivých prvků, tj. například trikolóra s pořadím barev červená-modrá-zelená je jiná trikolóra než trikolóra s pořadím barev zelená-modrá-červená. Celkový počet trikolór tedy bude 6!/3! = 120. • Variace s opakováním: Počet uspořádaných fc-tic (kdy opět záleží na pořadí prvků) s opakováním z n prvků, tj. kdy se daný prvek v dané fc-tici může vyskytovat vícekrát, je dán vztahem V'(k,n)=nk. (11.9) Typický příklad: kolik dvojciferných čísel lze vytvořit z číslic 1, 2, 3, 4, 5? Opět zde záleží na pořadí jednotlivých prvků, tj. například číslo 21 je jiné číslo než číslo 12, zároveň ovšem musíme zahrnout i čísla 11, 22, atd., kde se číslice opakují. Celkový počet takových dvojciferných čísel bude 52 = 25. • Příklady: 11.1 Kolik kružnic je definováno 12 body, ležícími v jedné rovině, pokud žádné 3 body neleží v jedné přímce? 220 11.2 Určete, kolika způsoby lze ze sedmi mužů a čtyř žen vybrat šestičlennou skupinu, v níž jsou právě dvě ženy. 210 11.3 V bedně je 54 výrobků, z nichž 21 je první jakosti, 27 je druhé jakosti a zbytek je vadných. Kolika způsoby lze vybrat skupinu 6 výrobků tak, aby obsahovala 3 výrobky první jakosti, 2 druhé jakosti a jeden vadný výrobek? 147420 11.4 Hokejové mužstvo má celkem 24 hráčů: 13 útočníků, 8 obránců a 3 brankáře. Kolik různých sestav může trenér vytvořit, jestliže sestava má mít 3 útočníky, 2 obránce a 1 brankáře? 6 552 11.5 Kolik prvků budeme potřebovat, abychom vytvořili šestkrát více kombinací čtvrté třídy (bez opakování prvků) než kombinací druhé třídy? 11 Kapitola 11. Kombinatorika 118 11.6 Trenér curlingu má k dispozici sedm hráčů: Aleše, Bedřicha, Cyrila, Davida, Emila, Filipa a Gustava. Má sestavit čtyřčlenné družstvo. (a) kolik družstev může sestavit, (b) kolik družstev může sestavit, pokud z trojice Aleš, Bedřich a Cyril hraje jen jeden, (c) kolik družstev může sestavit, pokud z trojice Aleš, Bedřich a Cyril hrají nejvýše dva a z dvojice David a Emil jeden nehraje, (d) kolik družstev může sestavit, pokud z trojice Aleš, Bedřich a Cyril hrají nejvýše dva a nehraje současně Filip a Gustav. (a) 35 (b) 12 (c) 18 (d) 21 11.7 Kolik existuje pěticiferných čísel? 90 000 11.8 Kolik čísel lze vytvořit ze sady neopakujících se číslic 7, 3, 5, 2, 4, 8, 1, 9 tak, aby čísla obsahovala letopočet objevení Ameriky? 1680 11.9 Na Kypru se poznávací značky na autech skládají z bloku 3 písmen, za kterým následuje čtyřciferné číslo. První část se vybírá pouze ze čtrnácti písmen A, B, E, H, I, J, K, M, N, P, T, X, Y, Z. (a) Kolik existuje takových poznávacích značek? (b) Kolik značek má každé písmeno jiné? (c) V kolika značkách je na prvním místě samohláska? (d) V kolika značkách je samohláska pouze na 1. a 3. pozici? (a) 143 - 9- 103 = 24 696 000 (b) 14 • 13 • 12 • 9 • 103 = 19 656 000 (c) 4 • 142 • 9 • 103 = 7056000 (d) 42 • 10 • 9 • 103 = 1440 000 11.10 Kolika způsoby můžeme sestavit z patnácti lidí libovolně velkou pracovní skupinu? Ve skupině může být 1 až 15 lidí. 32 767 Kapitola 11. Kombinatorika 119 11.11 V nádobě se nachází 105 částic ideálního plynu. Jaká je pravděpodobnost, že se všechny zcela náhodně se pohybující částice ocitnou v levé polovině nádoby, pokud částicemi budou (a) molekuly NH3? (b) jádra 4He? (a) 2-105 (b) (íc^ + i)-1 11.12 Uvažujme k = 3 mince, kdy každá může nabývat dvou „hodnot", tj. panna nebo orel. Je zřejmé, že pokud hodíme všemi mincemi zároveň, může nastat celkem n = 8 možných výsledků: PPP, PPO, POP, OPP, OOP, OPO, POO, OOO. Každý jednotlivý výsledek nazveme mikrostavem, který zohledňuje stav každé mince (nebo částice, pokud půjde o obecný fyzikální systém). Pokud budeme rozlišovat pouze počet hozených panen nebo orlů, specifikujeme tzv. makrostav (označme ho například Ei),v tomto případě tedy máme 4 možné makrostavy: 3P, 2P+10, 1P+20, 30. Počet mikrostavů, tvořících makrostav, nazýváme statistická váha (násobnost mikrostavů) W{Ei). Pro 4 makrostavy dostáváme v našem případě Wq = 1, W\ = 3, W2 = 3, W3 = 1, kde pořadová čísla jednotlivých makrostavů odpovídají počtu např. panen v daném makrostavu. Pravděpodobnost výskytu určitého makrostavu P{Ei) je dána podílem jeho statistické váhy W{Ei) a celkového počtu mikrostavů n. Entropie S určitého makrostavu Ei bude S = ln W. Vypište počet možných mikrostavů a pravděpodobnosti jednotlivých makrostavů v případě, že házíme (a) čtyřmi mincemi (b) dvaceti mincemi, který makrostav je nejpravděpodobnější? (c) jaká je pravděpodobnost makrostavu s 12 pannami a 8 orly? (d) sto mincemi, který makrostav je nejpravděpodobnější? (e) napište entropii makrostavů Eq, E±, Fmax (a) n = 16, P0 = 1/16, Pi = 1/4, P2 = 3/8, P3 = 1/4, P4 = 1/16 (b) n = 220, P0 = 1/220, Pi = 20/220, P2 = 190/220, P(Ei) 20! i!(20-i)!220' P20 = 1/220, Pmax = P 10 (c) přibližně 0,12 (d) n = 2100, P0 = 1/2100, Pi = 100/2100, P2 = 4950/2100, ..., P(Et ■ • PlOO = 1/2100, Pmax = P50 (e) 0, lnlOO ~ 4,6, ln 100! — 21n50! ~ 66,78 - nejpravděpodobnější makrostav má tedy nejvyšší entropii, nejméně pravděpodobný nejnižší (nulovou) 100! i!(100-i)!2100' 11.13 Mějme 3 částice ideálního plynu a 5 „přihrádek" - kvantových „krabic", označme je například a, b, c, d, e. Částice mohou být do jednotlivých přihrádek rozmístěny libovolným způsobem, odpovídajícím ovšem jejich typu (například není možné aby více Kapitola 11. Kombinatorika 120 fermionů bylo v jedné přihrádce). Každou jednotlivou variaci, případně kombinaci, systému částic označme jako mikrostav. Jako „makrostav" označme soubor mikrostavů, kdy jsou buď všechny tři částice v jedné přihrádce (označme jej jako „makrostav" „3"), nebo jsou dvě částice v jedné přihrádce a třetí v jiné (označme jej jako „makrostav" „2/1"), nebo je každá částice v jedné samostatné přihrádce (označme jej jako „makrostav" „1/1/1"). V uvedeném systému se tedy mohou vyskytovat nejvýše 3 „makrostavy". (a) kolik mikrostavů může nastat postupně pro molekuly NH3, jádra 4He, protony? (b) kolik „makrostavů" může nastat postupně pro molekuly NH3, jádra 4He, protony? (c) jaká je pravděpodobnost výskytu mikrostavů, kdy všechny tři částice budou v jedné určité přihrádce (například a), postupně pro molekuly NH3, jádra4He, protony? (d) jaká je pravděpodobnost výskytu mikrostavů, kdy každá ze tří částic bude samostatně v přihrádkách a, c, e, postupně pro molekuly NH3, jádra 4He, protony? (e) jaká je pravděpodobnost výskytu jednotlivých „makrostavů" postupně pro molekuly NH3, jádra 4He, protony? (a) 125, 35, 10 (b) 3, 3, 1 (c) 1/125, 1/35, 0 (d) 6/125,1/35,1/10 (e) „makrostav" „3": „makrostav" „2/1": „makrostav" „1/1/1" 1/25, 1/7, 0 12/25, 4/7, 0 : 12/25, 2/7, 1 Kapitola 12 Počet pravděpodobnosti1 Rozdělení pravděpodobnosti diskrétni náhodné veličiny X vyjadřuje tzv. pravděpodobnostní funkce P{X) s hodnotami pravděpodobnosti p{xí) = pí, kde YliPi = 1- Rozdělení pravděpodobnosti spojité náhodné veličiny X udává funkce hustoty rozdělení pravděpodobnosti (hustoty pravděpodobnosti) f(x), pro kterou platí f{x) dx = 1, kde $1 je definiční obor veličiny X. Pro hodnoty x ^ $1 platí /(a;) = 0. Významná rozdělení pravděpodobnosti jsou: • Rovnoměrné rozdělení pravděpodobnosti diskrétní i spojité náhodné veličiny X, které přiřazuje všem jejím hodnotám stejnou pravděpodobnost. Rovnoměrné rozdělení má ve všech bodech daného intervalu (a,b), konstantní hustotu pravděpodobnosti f(x)-ílTE pro x £ (a, b) I[ >~\ 0 pro xi (a, b)' [LZ-L) • Poissonovo rozdělení pravděpodobnosti diskrétní náhodné veličiny X, které lze vyjádřit pomocí zvoleného parametru A > 0 jako pt = ^e-\ (12.2) Xíl • Normální (Gaussovo) rozdělení pravděpodobnosti spojité náhodné veličiny X, které je definováno hustotou pravděpodobnosti ve tvaru tzv. Gaussovy funkce 1 (z-M)2 f(x) = —=e-^-, (12.3) OV27T kde parametr fi znamená střední hodnotu veličiny X, parametr o jeho směrodatnou odchylku (viz dále). Ve statistické fyzice se také střední počet rozlišitelných částic (např. molekul) ve stavu s energií E určuje pomocí tzv. Maxwellovy-Boltzmannovy rozdělovači funkce. Pro nerozlišitelné částice platí tzv. Fermiho-Diracovo rozdělení pro fermiony (elektrony, protony, neutrina, atd.) a Boseho-Einsteinovo rozdělení pro bosony (např. fotony). V matematické statistice se často používá tzv. Studentovo rozdělení (viz např. Pánek, 2001), atd. V návaznosti na rozdělení pravděpodobnosti můžeme určit celou řadu statistických nástrojů, pomocí nichž můžeme analyzovat náhodnou veličinu X (reprezentující například soubor naměřených hodnot). Mezi nej důležitější z nich patří: 1V této kapitole jsou použité příklady z knihy: Musilová & Musilová (2006). 121 Kapitola 12. Počet pravděpodobnosti 122 Váha - v případě diskrétni náhodné veličiny X s jednotlivými hodnotami x{ zavádíme tzv. váhu Wi, kterou můžeme zpravidla stanovit na základě tzv. vnitřních nejistot (chyb) Sxí hodnot X{ (například chyby měření, atd.), tedy na základě relace ~ é?- <12-4> Dále zavedeme tzv. sumu vah Sw a tzv. střední váhu ws, N i N Sw Sw = J2wí, ws = —J2wí = ^-, (12.5) i=i i=i kde iV je celkový počet diskrétních hodnot X{. Mezi váhami a hodnotami pravděpodobnosti existuje tedy volná relace - pokud suma vah Sw = 1, potom wí = Pí. Obdobným způsobem můžeme v případě spojité náhodné veličiny zavést tzv. váhovou funkci w(x), jejíž „suma" bude dána jako w(x) dx. Je tedy opět zjevné, že pokud tento integrál bude normován (bude roven jedné) bude platit w(x) = f(x), váhová funkce se takto stává hustotou pravděpodobnosti. Střední hodnota (aritmetický průměr), která se obvykle značí x, (x) nebo také fi. V případě diskrétní náhodné veličiny X bude střední hodnota definována jako suma všech hodnot Xi veličiny X dělená jejich počtem nebo jako suma násobků všech hodnot veličiny X s příslušnými hodnotami pravděpodobnostní funkce, tedy ^ N N (x) = Ň 5ľ Xi = 5ľ XiPi- (12-6) 1=1 1=1 V případě použití druhého vztahu mluvíme také o tzv. očekávané hodnotě, značené E(X), resp. o váženém aritmetickém průměru. Jemný rozdíl mezi těmito pojmy závisí na definici prvku Xi veličiny X, případně na způsobu volby tzv. statistické váhy. Tzv. váhovanou střední hodnotu (váhovaný aritmetický průměr) stanovíme jako 1 N bw i=i Střední hodnotu (neváhovanou a váhovanou) spojité náhodné veličiny X stanovíme jako , . ľ , . , , (o x w(x) dx (x) = E(X)= xf(x)dx, (x) = Jn \> . 12.8 V případě dále uváděných statistických nástrojů je stanovení jejich váhovaných podob zcela obdobné. Rozptyl a směrodatná odchylka jsou nejčastěji označované jako D(X), var(X), případně o~2(X) (rozptyl) a a(X) (směrodatná odchylka). Rozptyl (disperze) je definován jako střední hodnota druhých mocnin odchylek od střední hodnoty (aritmetického průměru) veličiny X, směrodatná odchylka je odmocninou z rozptylu. Pro diskrétní náhodnou veličinu X se stejnou váhou (pravděpodobností) všech hodnot X{ je rozptyl definován jako 1 N DW = jj S>» - <*»2 = <*2> - <*>2 • (12-9) i=i Kapitola 12. Počet pravdepodobnosti 123 V případě různých pravděpodobností diskrétních hodnot náhodné veličiny X bude rozptyl určen vztahem N N D(X) = ■ (Xi - (x))2 = -(x)2 + Yjx2-pl. (12.10) i=i i=i Pro spojitou náhodnou veličinu X je rozptyl definován vztahem D(X)= / (x - (x)y f(x)dx =-(x)z+ / xzf(x)dx. (12.11) In Jn Pro směrodatnou odchylku obecně platí &(X) = \JD(X). Nejpravděpodobnější hodnotu Pmax(AÍ) pro diskrétní náhodnou veličinu X stanovíme jako hodnotu X{ s nejvyšší hodnotou pravděpodobnostní funkce pi, tedy Pmiaí(X) = (xí, max (pi)). V případě spojité náhodné veličiny X určíme nejpravděpodobnější hodnotu Pmiaí(X) jako maximum funkce hustoty pravděpodobnosti f{x) v definičním oboru $1 veličiny X, tedy PmSíx.(X) = max(/(x)) pro x £ íl. Medián (xq^) a čtvrtkvantily (xq^, ^0,75, také nazývané dolní a horní kvartil) jsou hodnoty Xi, v nichž je monotónně uspořádaný statistický soubor rozdělen na příslušné množství stejně početných částí. Medián tedy dělí statistický soubor na dvě stejně početné poloviny. Výhodou mediánu oproti střední hodnotě je jeho neovlivnitelnost extrémně vychýlenými hodnotami. Například u souboru {1, 2, 2, 3, 27} medián žo,5 = 2, střední hodnota (x) = 7. V případě spojité náhodné veličiny X určíme medián a čtvrtkvantily (případně jakkoli jinak definované kvantily) z integrálních rovnic Ž0,5 1 rŽ0,25 1 rŽ0,75 Q f(x)dx=-, / f(x)dx=-, / f(x)dx=-. (12.12) • Distribuční funkce F(x) vyjadřuje pravděpodobnost, že hodnota náhodné veličiny X s daným rozdělením pravděpodobnosti bude menší nebo rovna x. V případě diskrétní náhodné veličiny X bude distribuční funkce F(x) daná předpisem F(x) = P{X < x) = Pi, (12-13) Xi+oo x^—00 P(xi < x < x2) = F(x2) - F(xi). Kapitola 12. Počet pravdepodobnosti 124 • Příklady: 12.1 Střelec provedl ./V = 150 výstřelů na terč, který je tvořen soustavou n = 5 mezikruží MKi, i = 1,..., 5. Mezikruží .\//\, přitom zasáhl AT;-krát, kde Nľ = 15, N2 = 20, N3 = 35, A^4 = 45, A^5 = 35. Za zásah mezikruží MKi získal i bodů. Náhodnou veličinu X s diskrétním rozdělením definujeme jako počet bodů, získaných pro jeden náhodný výstřel. Určete: (a) rozdělení {(xí,Pí)} veličiny X, (b) pravděpodobnost, že pro náhodný výstřel získá střelec alespoň I bodů, 7 = 1,2,3,4,5, (c) střední hodnotu veličiny X, (d) směrodatnou odchylku veličiny X, (e) pravděpodobnost, že při výstřelu získá střelec počet bodů v intervalu i £ (2,4). ' ioy v 15/ v 30y v 10/ v 30 Q 93 8 7 (b) Pi = 1,P2 = —, P3 = —, Pa = —, P5 = — v ' ' 10 30 15 30 (c) 3,43 (d) 1,26 V ' 3 12.2 Na letištních záchodech jsou čtyři kabinky. Je dána distribuční funkce obsazení kabinek: F(0) = 0,1, F(l) = 0,35, F(2) = 0,6, F(3) = 0,95, F(4) = 1. Určete: (a) rozdělení náhodné veličiny X, odpovídající počtu obsazených kabinek, (b) střední hodnotu veličiny X a její rozptyl, (c) pravděpodobnost, že budou obsazeny alespoň dvě kabinky. (a) {(0; 0,1), (1; 0,25), (2; 0,25), (3; 0,35), (4; 0,05)} (b) 2; 1,2 (c) 0,65 12.3 Je dána funkce f{x) = k ■ x pro 0 < x < 2 a f (x) = 0 v ostatních případech. Určete: (a) konstantu k tak, aby funkce byla hustotou pravděpodobnosti, (b) střední hodnotu a rozptyl, (c) nejpravděpodobnější hodnotu, (d) medián a čtvrtkvantily žo,25, žo,75, (e) distribuční funkci. (a) k = -v ' 2 Kapitola 12. Počet pravdepodobnosti 125 w 4-, 2 y ' 3' 9 (c) 2 (d) X0,5 = \/2, X0,25 = 1, Ž0,75 = \/3 (e) F (x) = O V x < O, F(x) = ^x2 V O < x < 2, F (x) = 1 V x > 2 12.4 Je dána funkce f(x) =--pro x > 0 a fix) = 0 pro x < 0. Určete: (x + lr (a) konstantu k tak, aby funkce byla hustotou pravděpodobnosti, (b) distribuční funkci, (c) nejpravděpodobnější hodnotu, medián a čtvrtkvantily xo,25 5x0,75. (a) k = 1 (b) F(x) = 0 V x < 0, F(x) = V x > 0 x + 1 1 (c) 0, X0,5 = 1, X0,25 = g, 2:0,75 = 3 k 12.5 Jsou dány funkce f(x) = pro 1 < x < 2, /(x) = 0 v ostatních případech, g(x) = xl c(x — x2) pro 0 < x < 1, g{x) = 0 v ostatních případech. Určete: (a) konstanty k a c tak, aby funkce byly hustotami pravděpodobnosti, (b) příslušné distribuční funkce, (c) nejpravděpodobnější hodnotu, střední hodnotu, rozptyl a medián pro každé z rozdělení. (a) k = 2, c = 6 (b) Fi(x) = 0 V x < 1, Fi(x) = 2?——^ V 1 < x < 2, Fi(x) = 1 V x > 2, F2(x) = 0 V x x < 0, F2(x) = 3x2 - 2x3 V 0 < x < 1, F2{x) = 1 V x > 1 (c) /:l,21n2,2-41n2 2, f, g : \\ 1, ± 12.6 Házíme dvěma kostkami. Náhodnou veličinou x označme součet bodů na obou kostkách při jednom hodu. Určete: (a) rozdělení veličiny x, (b) distribuční funkci, (c) střední hodnotu, rozptyl a nejpravděpodobnější hodnotu, (d) pravděpodobnost, že součet bodů na kostkách bude ležet v intervalu (5, 7). I'1' »>• (2- á) ■ (3' 5) - (4; ■ (5- 5) - (6- s) ■ (7- s) - (8- s) ■ (9- 5) ■ Kapitola 12. Počet pravdepodobnosti 126 (b) F (x) = O V x < 2, F (x) = — V x G (2,3), F (x) = — V x G (3,4), F(x) = - V 36 12 6 x G (4,5), F (x) = 1 V x G (5,6), F(x) = A V x G (6,7), F(x) = 1 V x G (7,8), F(x) = ^ V x G (8, 9), F(x) = l V x G (9,10), F (x) = ^ V x G (10,11), F(x) = 18 6 12 35 — V x G (11,12), F (x) = 1 V x > 12 (c) 7; 5,83; 7 Příloha A Křivočaré souřadnice Většina jevů v přírodě (a tedy i fyzikálních dějů) neprobíhá přísně pravoúhle a není vhodné a často ani schůdné je jednoduše popisovat pomocí kartézské metriky. V tom případě je výhodné zvolit takovou souřadnou soustavu (zpravidla křivočarou), která co nejlépe odpovídá geometrii popisovaného děje. Nejčastěji používanými křivočarými souřadnými soustavami jsou soustava válcová (cylindrická) a soustava kulová (sférická). Dále existuje řada speciálních křivočarých souřadných soustav, např. eliptická, parabolická, kónická, atd., včetně soustav neortogonálních, tj. takových, kdy jednotlivé souřadnicové směry nesvírají pravý úhel. Zvládnutí matematického aparátu, popisujícího křivočaré souřadnice, jejich vztahy a vzájemné převody, je pro fyzikální praxi nezbytné. V následujících poznámkách si ukážeme základní principy a praktické postupy při počítání v kartézských, válcových a kulových souřadných soustavách. A.l Kartézská soustava Ačkoli kartézská soustava vlastně nepatří mezi křivočaré soustavy, uvádíme ji zde jako základní a nejjednodušší ortogonální souřadnou soustavou, na níž si názorně ukážeme základní vztahy a geometrické principy, které v rámci složitějších, skutečně křivočarých souřadných soustav již pouze analogicky upřesníme a aplikujeme. Její zásadní předností je, že (jednotkové) vektory kartézské báze, ex =x = (1,0,0), ey =y = (0,1,0), ez = Ž = (0, 0,1), (A.l) jsou konstantní (mají stále stejnou velikost a stále stejný směr), derivace těchto vektorů jsou tedy nulové.1 Pro druhou mocninu vzdálenosti dvou bodů v diferenciálním tvaru platí ds2 = dx2 + dy2 + dz2, což lze zobecnit tzv. metrickou formou, ds2 = gij dxl dx^, (A.2) kde indexy i, j značí jednotlivé souřadnicové směry (i, j = x, y, z) a zároveň tak určují jednotlivé složky 3x3 metrického tenzoru. Kovariantní metrický tenzor kartézské soustavy má tedy elementární tvar jednotkové matice. Význam kontravariantního metrického tenzoru g^ kartézské soustavy je formálně určen druhou mocninou velikosti vektoru (čte) ^ dxídxj (čte) (čh/) (cte) ( ^) 1V dalším textu budeme jednotkové vektory zapisovat ve „stříškovém" tvaru, tedy např. x místo ex. 127 Príloha A. Křivočaré souřadnice 128 Zároveň musí pro každou metriku obecně platit gij g11 = 1, kovariantní a kontravariantní metrický tenzor tak budou vždy tvořit vzájemně inverzní matice. V kartézské soustavě budou mít tedy tvar 9ij A 0 °^ 0 °\ 0 i 0 0 1 0 Vo 0 1) vO 0 1/ (A.4) A. 1.1 Diferenciální operátory • Gradient skalární funkce f = f (x, y, z) je v kartézské soustavě definován jako vektor ve tvaru ^, , , -df .9/ .9/ ŕ d f d f d f V f = grad/ = x— + y— +z— = —, —, — dx dy dz \ox dy o z (A.5) Gradient vyjadřuje v každém bodě skalárního pole směr největšího růstu tohoto pole. Gradient vektoru (vektorového pole) A {x, y, z) je definován jako tenzor 2. řádu ve tvaru VA = gradA= (X^ľ+ý^+ž^ ) {Axit + Ayy + AZŽ). (A.6) Protože se jedná o tzv. tenzorový součin, kdy se jednotlivé vektory báze násobí jako matice, z nichž první je sloupcová a druhá řádková, je třeba pro určení prvků tenzoru zachovat jejich pořadí. Pomocí maticového formalismu můžeme tenzor gradientu vektorového pole zapsat jako x VA X ý ž ídAx dAy dAz\ dx dx dx dAx dAy dAz dy dy dy dAx dAy dAz V dz dz dz / (A.7) Divergence vektoru (vektorového pole) A (x, y, z) je definována jako skalár (skalární pole) dK ^ r ,. r . . dAx „ „dAy „ „dAz dAx dAv dAz V • A = div A = x • x —A + ý • ý + ž • = + + dx dy dz dx dy dz (A.í Divergenci vektoru lze v ortogonálních soustavách rovněž chápat jako stopu tenzoru gradientu vektorového pole. V obecných ortogonálních souřadnicích může být zapsána ve formě V • A ■^-(hkAk)+T^hlAl Ô^^A^+T^AK (A.9) S využitím rovnice (2.53) lze tento výraz přepsat rovněž do tvaru V • A _d_ dXn {hkAk) - Tlkh[A[ 53k = -^(hJAJ)-Tl]]hlAl (A.10) Príloha A. Křivočaré souřadnice 129 Členy hi jsou tzv. Laméovy koeficienty (často také nazývané škálovacífaktory, nezaměňovat se stejnojmennými Laméovými koeficienty v mechanice kontinua), pojmenované po francouzském matematikovi Gabrieli Lamé, kde v příslušném metrickém tenzoru platí hihi hlhl (A.11) (proto nyní uvažujeme jen ortogonální soustavy, jejichž metrické tenzory mají nenulové prvky pouze na hlavní diagonále). Výraz Tl-k je tzv. Christoffelův symbol (pojmenovaný po německém matematikovi a fyzikovi Elwin Bruno Christoffelovi), definující tzv. členy křivosti v křivočarých souřadných soustavách, 1 jk Jm j d(Jkm ! 9gjm dxi + dgj. jk dxk dx,n (A.12) kde indexy l, m jsou tzv. volné indexy, které mohou kdykoli nabývat kterékoli z hodnot 1, 2, 3. Explicitní výraz pro divergenci vektoru v obecné ortogonální soustavě lze zapsat formou V • A 1 hihjhk d d d — (hjhkAi) + — (hkhiAj) + ^— (hihjAk) (A.13) Ta je zcela ekvivalentní kompaktnější formě zápisu, - - ld V- A = — — (hjhkAi x/gdxiK (A.14) Složky h'Al (v literatuře se většinou zkráceně uvádí pouze A1) vektoru A odpovídají (viz rovnice (2.53)) KA% = gl:)(hjAj) a g je determinant metrického tenzoru, který je identický s druhou mocninou příslušného Jakobiánu souřadnicové transformace. Platí tedy det gij | = J, \J\ det gli\ = J r-l (A.15) Obecně také platí, že divergencí tenzoru řádu n je tenzor řádu n — 1, divergencí tenzoru druhého řádu tak bude vektor. Kompaktní forma zápisu divergence tenzoru 2. řádu bude mít tvar Ai (A.16) její explicitní zápis v kartézském systému (prakticky se jedná o maticové násobení vektoru s transponovanou maticí; při skalárním součinu dvou vektorů se také jedná o maticové násobení dvou vektorů, kdy druhý z vektorů je transponovaný, tedy sloupcový) bude vypadat V • A, 'dAa x dx + ~ d Ä d Ä d dx dy dz dAXy dAa dy + dz + ý x y z x í A A s?-xy A \ T ý A si-yx A ^yy A siyz ž V Azx A Slzy Azz J ^ dx dAyy dy ,9AyZ\ dz J + z dAZQ dx + dA zy dy + (A.17) dAzz\ dz J ' Príloha A. Křivočaré souřadnice 130 Rotací vektoru (vektorového pole) A{x, y, z) v kartézské soustavě nazýváme vektor V x A = x dAz d A,. dy dz + ý d Ar dAz dz dx + z dA, dAq dx dy (A.18) Obecný výraz pro vektor rotace V x A vektoru A v libovolné ortogonální souřadné soustavě lze definovat způsobem Vxl = €iik t-t- \^j{hkAk)} íčj = €ijk hjhk hjhk d — (hkAk)-TljkhiAi (A.19) V rovnici (A.19) výraz €ijk (kde všechny tři indexy i,j,k mohou odpovídat postupně všem třem souřadnicovým směrům) odpovídá antisymetrickému, (tzv. Levi-Civitovu, viz rovnice (2.44)) symbolu, který nabývá hodnoty 1 pro sudé permutace indexů, —1 pro liché permutace indexů. Díky symetrii indexů ve složkách vektoru rotace a díky úplné antisymetričnosti Levi-Civitova e-symbolu, se výrazy ^ljkAi v rovnici (A.19) vyruší, celý výraz se tak zjednoduší do podoby V x A = e. 1 ijk hjhk d dxj (hkAk) x,- (A.20) V kartézské soustavě, kde hi,h2,hs = 1, bude rovnice (A.20) odpovídat rovnici (A.18). Zapíšeme-li vektor rotace znovu po složkách, dostaneme V x A dAz dAy dAx dAz dAy dAa dy dz ' dz dx ' dx dy (A.21) Laplacián (Laplaceův operátor) je definován jako divergence gradientu, tedy V-V (používá se pro něj symbol A), jedná se tedy o skalární operátor, který může působit na skalární funkce, vektory (po jednotlivých složkách), tenzory (po jednotlivých prvcích), aniž by měnil jejich řád (t.j. skalár zůstává skalárem, vektor vektorem, atd.). V kartézské soustavě má laplacián zcela jednoduchý tvar: analogicky k rovnici (A.8), kde složky vektoru A nahradíme složkami vektoru gradientu, můžeme psát A = V • V = divgrad = x d_ dx d_ dx x- + ý d d dy \ dy + z d2 d_ dz dz d2 d2 dx2 dy2 dz2 (A.22) A.1.2 Plochy, objemy Označme Sk plochu s konstantní hodnotou souřadnice Xk, ohraničenou souřadnicovými křivkami Xí,Xí + Axí,Xj,Xj + Axj. V kartézské soustavě půjde např. o plochu s konstantní hodnotou z = zq, ohraničenou přímkami x = xa,x = xa + Ax,y = ya,y = yo + Ay. Výpočet velikosti takové plochy je zde samozřejmě zcela triviální, půjde o obdélník (čtverec) s obsahem AxAy. Obecný vztah pro výpočet velikosti takové plochy bude mít ovšem tvar xqí+Axí x0j+Axj Sk (A.23) XQi x0j Príloha A. Křivočaré souřadnice 131 kde je druhá odmocnina absolutní hodnoty determinantu příslušné submatice metrického tenzoru. V uvedeném případě by se jednalo o determinant J[- = \/\gugjj — 9íj9jí\ (v případě ortogonálního souřadného systému bude determinant uvedených submatic sám o sobě kladný, pokud použijeme sudé permutace indexů a nediagonální členy dané submatice budou nulové). Integrand rovnice (A.23) definujeme jako elementární plochu dSk = J-j dxidxj. V kartézské soustavě budou zjevně determinanty všech tří submatic .][■ = 1. Dále, označíme-li V objem prostoru, vymezeného plochami s konstantními souřadnicemi Xj,Xj + Axí, xj,xj+Axj, x&, x& + Axk, obecný vztah pro výpočet velikosti takového objemu bude mít tvar x0i+Axi xoj+Axj x0k+Axk V= / / Jdxídxjdxk, (A.24) XOi %0j xok kde J je druhá odmocnina absolutní hodnoty determinantu metrického tenzoru (Jakobián - viz rovnice (A.15)). Integrand rovnice (A.24) vyjadřuje elementární objem dV = Jdxi dxj dx&. V kartézské soustavě opět J = 1, vymezený prostor bude mít tvar pravoúhlého kvádru o objemu AxAyAz. A.1.3 Vektory polohy, rychlosti a zrychlení V kartézské soustavě je zápis vektorů velmi jednoduchý, polohový vektor r, vektor rychlosti v a vektor zrychlení a budou mít postupně tvar, f = ix + yý + zž = (x,y, z), (A.25) df v = — = ix + ýý + iž = vxa + vyý + vzž = (vx, vy, vz), (A.26) dv a = — = xx + yy + zz = a^x + ayy + azz = (ax, ay, az), (A.27) kde x = dx/dt, ý = dy/dí a i = dz/dt. A.2 Válcová soustava Válcová soustava může být vhodná pro popis celé řady osově symetrických a rotačních jevů, např. elektrického a magnetického pole okolo přímých vodičů, vírů v tekutinách, galaxií, hvězdných disků, atd. Souřadnicové směry jsou: p - vzdálenost od osy válcové symetrie, 0 - azimutální úhel, z - výška. Převod z válcové do kartézské soustavy je dán vztahy2 x=pcos0, y = psin0, z = z. (A.28) Pro zpětnou transformaci z kartézské do válcové soustavy platí3 p = V1 x2 + y2, (j) = arctg — pro x 7^ 0, případně (j) = arccos —, z = z. (A.29) x p 2V dalším popisu budeme rozlišovat p pro radiální válcovou souřadnici, r pro radiální kulovou souřadnici. V případě jednotkových bázových vektorů budeme rozlišovat p pro válcovou souřadnici, ř pro kulovou souřadnici. Nekonstantnost jednotkových vektorů válcové i kulové báze je rovněž zvýrazněna skloněným písmem, konstantní bázové vektory jsou zapsány stojatými písmeny. 3Pro vyčerpávající popis vztahu pro azimutální úhel doporučuji např.: Musilová & Musilová (2006). Príloha A. Křivočaré souřadnice 132 Obrázek A.l: Schéma vzájemné transformace jednotkových bázových vektorů kartézské a válcové souřadné soustavy (viz rovnice (A.30) a (A.33)). Vztah pro azimutální úhel 0 je strukturovanější, výraz uvedený v rovnici (A.29) platí jednoznačně pouze pro 1. kvadrant, 0 £ (0,7r/2). Pro ostatní kvadranty je třeba vždy zvážit znaménka souřadnic x a y, pokud např. x < 0 A y > 0, jedná se o 2. kvadrant, atd. Jednotkové vektory válcové báze budou mít v kartézské soustavě tvar (viz sčítání vektorů) p = x cos 0+ý sin 0 = (cos 0, sin 0, 0), = — x sin 0+ý cos 0 = (— sin 0, cos 0, 0), ž = (0, 0,1). (A.30) Jediným konstantním bázovým vektorem bude vektor ž, ostatní bázové vektory mění směr v závislosti na úhlu 0. Nenulové derivace bázových vektorů ve směru souřadnicových os a nenulové časové derivace bázových vektorů budou (z rovnice (A.30)) - = (-sm0,cos0,o) = 0, m=djm=^ — = (- cos 0, - srn ,0) = -p, ^=djm= ~P, sin 0, 0) d í x y \ ~ sin -, dx dx dx y ^Jx2 + y2 ' ^Jx2 + y2 ' J d4> = <9(-sin0,cos0, 0) = ^ sin0 dx dx p Obdobně získáme derivace ve všech ostatních směrech. Zpětná transformace jednotkových bázových vektorů (viz rovnice (A.30)) bude x = p cos 0 — 0sin0, ý = psin 0 + cos 0, ž = ž. (A.33) Metrickou formu pro válcovou soustavu snadno odvodíme, uvědomíme-li si, že vzdálenost dvou bodů v prostoru musí být nezávislá na volbě souřadného systému, tedy ds2 z rovnice (A.2) se musí pro všechny souřadné soustavy rovnat. Z rovnice (A.28) dostaneme, dx = dp cos 0 — psin 0d0, dy = dpsin0 + pcos0d0, dz = dz, (A.34) dosazením do rovnice (A.2) dostáváme válcovou metrickou formu ds2 = dp2 + p2 dcf)2 + dz2, (A.35) Príloha A. Křivočaré souřadnice 133 můžeme tedy napsat kovariantní (gíj) i kontravariantní (g13) metrický tenzor a také (viz rovnice (A. 11)) příslušné Laméovy koeficienty válcové souřadné soustavy, (1 0 °^ 9íj = 0 P2 0 , 9ij = 0 1 0^0 p2 hr, 1, hej, = p, hz V° 0 v Nenulové Christoffelovy symboly válcové metriky z rovnice (A. 12) budou Tp - -o (T"M - i (A.36) (A.37) A.2.1 Diferenciální operátory • Gradient skalární funkce / = / (p, 0, z) ve válcové soustavě odvodíme z rovnice (A.5), kam za jednotkové bázové vektory dosadíme výrazy z rovnice (A.33) a jednotlivé složky gradientu rozvineme řetězovým pravidlem pro derivace. Po rozepsání dostáváme d f d z' V/ p COS ( 0sin< + I p sin 4> + 4> cos < d p dx 90 dx dz dx d f d p d f 90 d f dz d p dy 90 dy dz dy + dz (A.38) Jednotlivé parciální derivace vypočítáme z rovnice (A.29), dp dx dp dy \/x2~Ty2 y COS 0, srn i 90 dx 90 dy sm < x2 + y2 cos 0 ^Jx2 + y2 ""^ dy x2 + y2 p Po dosazení a úpravě dostaneme výslednou podobu gradientu Vf - p^- + 0-— +i— ~ (— -— —} (A 40) d p p d p 0 ž \ dAp 9A0 dAz dp dp dp 1 dAp A^ 19A0 Ap p 90 p ldAz p 90 p p 90 dAp dA^ dAz dz dz dz ) (A.42) Príloha A. Křivočaré souřadnice 134 Stejného výsledku docílíme v tomto případě i jiným postupem, např. s použitím formalismu Christoffelových symbolů (viz rovnice (A.37)), kde, na rozdíl od rovnice (A.19), zapíšeme 1 hjhk [Vj(hkAk)\ Xj-xfc 1 hjhk d dxj {hkAk) - VLhiAi Xj Xfc. (A.43) Postup podle rovnice (A.43) lze ovšem použít pouze pro ortogonální souřadné soustavy, postup podle rovnice (A.41) platí zcela obecně. Divergence vektoru (vektorového pole) A (p, 0, z) je ve válcových souřadnicích ve smyslu rovnice (A.41), analogicky k rovnici (A.8), definována jako skalár (skalární pole) V-A = p-p—^ + 0-0 ^ + -_ op \ p p 0(p 1 d + z • z ■ dA7 _ , A , ldA^ -— (PAP +--s-r + p Op p 0(p dz dAz dz (A.44) Porovnáním s rovnicí (A.42) opět vidíme, že divergence je stopou tenzoru gradientu vektorového pole. Divergencí tenzoru 2. řádu, popsaného maticí 3x3, bude vektor (tenzor 1. řádu). Explicitní formu zápisu divergence tenzoru 2. řádu ve válcových souřadnicích (srovnej s rovnicí (A.17) zde již uvádět nebudu, zájemce odkazuji na literaturu, např. Abramowitz & Stegun (1972), Young (1993), Arfken & Weber (2005), atd. Rotaci vektoru (vektorového pole) A (p, 0, z) ve válcové soustavě, kde hp p, hz = 1, odvodíme podle již uvedeného vztahu (A.20). Dostáváme 1, h„ V x A ldAz p d dA4 dz p + dAn d A, dz dp 0 + 1 d . . , dAp ďp^-lii. z. (A.45) • Laplacián odvodíme (viz rovnice (A.22)), nahradíme-li v rovnici divergence (A.44) složky vektoru A odpovídajícími složkami vektoru gradientu z rovnice (A.40). Dostáváme ^ pdpx^dp) pd(j)\ pdcj)) Z dz\dz) pdp\dp) p2 dej)2 d z2 A.2.2 Plochy, objemy Stejně jako v kartézské soustavě označme Sk plochu s konstantní hodnotou souřadnice Xk, ohraničenou souřadnicovými křivkami xí,xí + Axí,xj,xj + Axj. Ve válcové soustavě půjde např. o plochu s konstantní hodnotou z = zq, ohraničenou polopřímkami 0 = (f>i,(f> = 02 a křivkami (kružnicemi) p = pi,p = p2- Výpočet velikosti takové plochy již není tak zcela triviální, jako v kartézské soustavě, půjde o průnik kruhové výseče s plochou mezi dvěma soustřednými kružnicemi. Pokud budeme uvažovat jinou plochu, např. s konstantní souřadnicí p = po, ohraničenou souřadnicovými plochami 0 = (f>i,(f> = 02, z = z\,z = z2, půjde o část Príloha A. Křivočaré souřadnice 135 válcové plochy. Při výpočtech velikostí těchto ploch vyjdeme z rovnic (A.23) a (A.36), tedy, 02 Z2 02 Z2 Sp = j j ^Jgéá 9zz dýdz = J J pdýdz = pAýAz, 4>i zi 4>i z\ Z2 P2 Z2 P2 SáPá=^y^1^- Pl 01 Pl 01 Ve válcovém souřadném systému budou nediagonální členy submatic .][■ (viz rovnice (A.23)) nulové. Označíme-li V objem prostoru, vymezeného souřadnicovými plochami s konstantními souřadnicemi p±, p2, (f>i, (f>2, z±, z2, výpočet velikosti tohoto objemu bude mít dle rovnice (A.24) tvar P2 02 z2 2 2 V = j j j pdpd(f)dz = p2~ Pl Acf)Az. (A.48) Pl 01 Zl V ortogonálním válcovém souřadném systému můžeme Jakobián J stanovit jako y/gpp gZz = p. Obdobným způsobem můžeme při odpovídajícím stanovení integračních mezí vypočítat v daném souřadném systému velikost jakéhokoli jiného, složitějšího útvaru. A.2.3 Vektory polohy, rychlosti a zrychlení Při popisu vektorů ve válcové soustavě vyjdeme z jejich popisu v soustavě kartézské, zahrneme všechny rovnice pro derivace jednotkových vektorů i vektorových složek (rovnice (A.28)-(A.33)). Polohový vektor a vektor rychlosti ve válcové soustavě budou .....^dfd (pp + zž) . Ä x . Ä . Ä • ~ r = xx + yy + zz = pp + zz, v = — =--- = pp + pp + zx = pp + p = -y- - p4> , H = p4> + 2p4> = — + p4>, az = z = —. (A.51) Príloha A. Křivočaré souřadnice 136 Protože d/dí = d/dt + v - V (řetězové pravidlo pro derivování, v tomto případě pro parciální derivace v = v(t, p, ,z)), potom zrychlení, vyjádřené pomocí složek vektoru rychlosti bude ap~ dt +Vpdp + p 90 +Vzdz p' (A-52j (A.53) dv^ + Vp^p~ p 90 dv^ OZ dvz &z = -TT~ dt dvz + VpW ! vdvz p 90 dvz OZ (v-V)vz (A.54) A.3 Kulová soustava Kulová soustava je vhodná pro popis jevů s centrální symetrií, jako jsou např. fyzikální pole, tvořená hmotnými body, astronomickými tělesy, atd. Mimo jiné se implicitně používá také v kartografii, kde soustava poledníků a rovnoběžek je vlastně soustava azimutálních a sférických úhlových souřadnic (viz dále). Zde je ovšem sférický úhel počítán jiným způsobem, v „matematické konvenci" roste od 0 do tt, v „kartografické konvenci" roste od —tv/2 do tt/2, navíc v opačném smyslu vůči směru nárůstu azimutální souřadnice. Souřadnicové směry jsou: r - vzdálenost od středu kulové symetrie, 9 - sférický úhel, 0 -azimutální úhel. Převod z kulové do kartézské soustavy je dán vztahy x = r sin 9 cos 0, y = r sin 9 sin 0, z = rcos9. (A.55) Pro zpětnou transformaci z kartézské do kulové soustavy platí4 r = \j x2 + y2 + z2, 9 = arccos — , 0 = arctg —. (A.56) \/ x2 + y2 + z2 x Analogicky k rovnici (A.30) budou mít jednotkové vektory kulové báze v kartézské soustavě tvar (viz pravidla pro sčítání vektorů) ř = x sin 9 cos 0 + ý sin 9 sin 0 + ž cos 9 = (sin 9 cos 0, sin 9 sin 0, cos 9), 0 = x cos 9 cos 0 + ý cos 9 sin 0 — ž sin 9 = (cos 9 cos 0, cos 9 sin 0, — sin 9), (A.57) (f) = —xsin 0 + ý cos 0 = ( — sin 0, cos 0, 0). V kulové soustavě není žádný z vektorů báze konstantní. Derivace bázových vektorů ve směru jednotlivých souřadnicových os budou (z rovnice (A.57)) (psin 9, 4>cos9, (A.58) : — ř siné? — 0cos9. dř dr ' dř ~ dř 90 de dr ' do 90 ďe~ ~ ~r' 90 dr ^ = 0 de ' 90 90 4Uvedené vztahy pro ŕ? a neuvádí jednoznačné výrazy pro všechny kvadranty, viz rovnice (A.29). Pro exaktní popis doporučuji např.: Musilová & Musilová (2006). Príloha A. Křivočaré souřadnice 137 \ Obrázek A.2: Vzájemná poloha jednotkových bázových vektorů kartézské a kulové souřadné soustavy (viz rovnice (A.57) a (A.60). Časové derivace bázových vektorů budou dř dř 39 dř d(f> + d4>~ďl ~di ~ dědt dě ~ďi dě de ~ ~ďe~ďi dě d(f> + ~d~4> ~dt d$ dej) d ~dt ~ ~d~4>~ďt 09 + <})(f)sm9, —ř9 + (fxf) cos 9, —řtj) sin 9 — 9 + 0 cos 9 cos sin 0, ý = ř sin 9 sin (f> + 0 cos 9 sin 0 + cos 0, ž = ř cos 9 — 9 sin 0. Metrickou formu pro kulovou soustavu dostaneme diferencováním rovnice (A.55), dx = dr sin 9 cos 0 + r cos # cos cos0 - 0sm0 —— + —— + —— ^ + V / \dr dx oti dx o (p dx J d f dr d f 39 d f d(p dr dy 39 dy d(p dy dr d z ' 39 d z J Jednotlivé parciální derivace vypočítáme z rovnice (A.56), ' d f dr d f 09 d f d (pN + í r sin 9 sin (p + 6 cos 9 sin

r d(p y sin 0 dx x2 + y2 r sin #' d(f) x cos 0 dy x2 +y2 r sin # Po dosazení a úpravě dostaneme výslednou podobu gradientu (A.66) ^, -9f -ldf 1 1 9/ /a/ 19/ 1 a/\ ,A W = ŕ/+0-7é + ^—^Ä7= /'"Té'—• (A.67) ar r ot/ r sm a \ ar r oo r sin a / Opět zde za jednotkové vektory kulové báze nedosazujeme jejich složky z rovnice (A.57), kde jsme je „viděli" ze soustavy kartézské. Analogicky k rovnici (A.6) (tenzorový součin) a s použitím rovnice (A.67) je potom gradient vektorového pole A (r, 9, 0) v kulové soustavě definován jako tenzor 2. řádu ve tvaru f7T+Ô-^ + ^—S^I ) Urr + Ae6 + A^) . (A.68) ar r otí r sm 9 dep J \ / Na rozdíl od válcové soustavy zde operátor gradientu již působí na všechny jednotkové bázové vektory (jejich derivace - viz rovnice (A.58)). Pomocí maticového formalismu Príloha A. Křivočaré souřadnice 139 můžeme tenzor gradientu vektorového pole v kulové soustavě zapsat e 0 r ÔAr e dAe dr 1 d Ar Ag r 86 1 dAr dr IdAe | Ar r 39 r 0 dr Id A, Aa V r sin 9 d(/) r r sin 9 d(/) 1 dAe Aa, — cotg 9 —— r r sm t/ r d9 i ča. a, ^ - H---1--cotg 9 (A.69) Stejného výsledku docílíme i v tomto případě např. s použitím formalismu Christoffe-lových symbolů (viz rovnice (A.37)) dle obecného vztahu (A.43), tento postup lze ovšem použít pouze pro ortogonální souřadné soustavy, zatímco postup podle rovnice (A.68) platí zcela obecně. Divergence vektoru (vektorového pole) A (r, 9, 0) je v kulových souřadnicích ve smyslu rovnice (A.68), analogicky k rovnici (A.8), definována jako skalár (skalární pole) V- A = r -r—--h#-0 —^r- H-- dr \rd9 r + 0-0 1 9A^ Ar Ae r smí/ dm r r 1 _L — Ír2 Ar) + d / , n * \ 1 d A sm 9 Ae) + r siní? d(f> (A.70) Porovnáním s rovnicí (A.69) opět vidíme, že divergence je stopou tenzoru gradientu vektorového pole. Rotaci vektoru (vektorového pole) A (r, 9, 0) v kulové soustavě, kde hr = 1, hg = r, Ha, = r siní?, odvodíme podle rovnice (A.20), v tomto případě dostaneme V x A 1 r siní? - Slil 9 Aa)-- d9 V 07 di,(f> = (f>2- Výpočet velikosti takové plochy již zde není vůbec triviální, půjde o část plochy s dvojí křivostí, ohraničenou dvěma rozbíhajícími se souřadnicovými plochami (v nichž leží křivky se souřadnicemi 0 = (f>i, i, (f>2, tvar takového útvaru odpovídá v tomto případě průniku jehlanu s koncentrickou sférickou mezivrstvou (mezikoulím), vztah pro výpočet velikosti takového objemu bude mít dle rovnice (A.24) tvar r2 6*2 02 3 3 V = JJ j r2 sin 6 dr d6 d0 = ^ ~ Tl (cos 91 - cos 02) A. (A.74) ri 6»i 0i V ortogonálním kulovém souřadném systému obdobně jako ve válcovém systému můžeme Ja-kobián J stanovit jako ^Jgrr gee = r2s'm9. Popsaná metoda umožní při odpovídajícím stanovení integračních mezí vypočítat v kulovém souřadném systému velikosti jakkoli složitějších útvarů. A.3.3 Vektory polohy, rychlosti a zrychlení Při popisu vektorů v kulové soustavě vyjdeme z jejich základního popisu v soustavě kartézské, zahrneme všechny rovnice pro derivace jednotkových vektorů i složek vektorů (rovnice (A.55)-(A.60)). Polohový vektor a vektor rychlosti v kulové soustavě budou -, - - - - -, dr* d (vt) ä x - (■" ■" \ r = ix + yý + zi, = rf, v = — = ——— = řf +rr = řf + r \ 96 + 4>(j) sin 6J . (A.75) Tento závěr lze opět očekávat, uvědomíme-li si, že polohový vektor vychází z počátku souřadnic. Vektory rychlosti a zrychlení jsou zároveň definovány jako _ _^ dv v = vrf + vq6 + Vficj), a = — = arř + ag6 + a^tp. (A.76) Príloha A. Křivočaré souřadnice 141 Derivováním rovnice (A.75) podle času dostáváme jednotlivé složky vektoru zrychlení v kulové souřadné soustavě, ar = f - rÓ2 - r sin 9 + 2ř(f> sin 9 + 2r00 cos 9 = —^ + ř^siné? + r9(f> cos 9. (A.79) Protože d/dt = d/dt + v ■ X7 (řetězové pravidlo pro derivování, v tomto případě pro parciální derivace v = v(t, r, 9, 0)), potom zrychlení, vyjádřené v kulové souřadné soustavě pomocí složek vektoru rychlosti bude 2 2 OT or r Ó9 rsmO ocp r dvg dvg Vgdvg Va, dvg VrVg ^ cotg 9 , a6» = -^T + wr"^--1---H7T "I--r-7"^7 +---"-. (A-81 aí or r o9 rsmO ocp r r V OTU OTU VflOTU , OTU UriU COtg 0 a = "ST--1---^--^"a^X"1---1--• (A.82) ot or r o9 rsmO ocp r r Bylo by jistě možné popsat mnohem více podrobností, např. operace s vektory a tenzory v rámci popisovaných soustav, atd., zde jsou ukázány alespoň některé postupy spíše z praktického pohledu. V dalších odstavcích ukážeme stručně alespoň jednu neortogonální souřadnou soustavu, jejíž popis byl do jisté míry vyvolán tvorbou numerické výpočetní sítě pro hydrodynamické modelování konkrétního fyzikálního jevu. A.4 Eliptická soustava Dále stručně uvedeme tři specifické ortogonální soustavy, které mohou souviset s předchozí tématikou nebo s uvedenými příklady (případně mohou mít zajímavé fyzikální uplatnění) - eliptickou, parabolickou, a „anuloidovou". Dvourozměrná eliptická souřadná soustava (viz obrázek A.3) je definována dvěma třídami souřadnicových křivek s konstantními parametry o G (0, oo) a r G (0, 2tv) (toto značení není zcela ustálené, v různých literaturách může být různé), se dvěma společnými ohnisky v bodech ( — a, 0), (a, 0). V trojrozměrné verzi přibude ještě (válcová symetrie vzhledem k ose z) azimutální úhlový parametr (p. Transformační rovnice z kartézské do eliptické soustavy v trojrozměrném případě budou x = a cosh a cos r cos 0, y = a cosh a cos r sin 0, z = a sinh a sin r. (A.83) Z definice hyperbolického sinu a kosinu (1.15),(1.16) a z exponenciálního vyjádření sinu a kosinu (viz Eulerovy vztahy v příkladu 8.5) snadno odvodíme zpětné transformační vztahy, které ovšem budou mít (v pravotočivém pořadí proměnných a, 0, r) komplexní tvar (rovinu Príloha A. Křivočaré souřadnice 142 y Obrázek A.3: Schéma dvourozměrné eliptické soustavy v rovině x, y, společná ohniska jsou v bodech (—a, 0), (a, 0). Modře vyznačené jsou eliptické křivky s konstantním parametrem a, s posloupností (od nejužších k nejširším) a = 0; 0,2; 0,4; 0,6; 0,8; 1, červeně vyznačené jsou hyperbolické křivky s konstantním parametrem t, s posloupností (zprava do leva) odT = 0doT = 7rs intervalem tt/Í2. V trojrozměrné verzi (viz popis) potom vyobrazenému směru y odpovídá směr z. p-z, kde p = \Jx2 + y2, si můžeme představit jako Gaussovu rovinu), a = — 2 t, P + iz , x,P-'lz argcosh--h argcosh- y arctg —, x 2i argcosn--argcosn- (A.84) Metrická forma takové eliptické soustavy bude mít tvar ds2 = a2 [(cosh2 a sin2 r + sinh2 a cos2 r) (der2 + dr2) + cosh2 a cos2 r dej)2 = a2 [(sinh2 a + sin2 r) (der2 + dr2) + cosh2 a cos2 r d02] = = a2 [(cosh2 a — cos2 r) (der2 + dr2) + cosh2 a cos2 r d02] . (A.85) Kovariantní metrický tenzor Qíj a příslušné Laméovy koeficienty eliptické souřadné soustavy v pořadí směrů a, (f>, r budou, 9íj a2 (sinh2 a + sin2 r) 0 0 0 o a2 cosh2 o cos2 r 0 0 a2 (sinh a + sin r) (A.86) ha = a\Jsinh2 o + sin2 hA a cosh o cos r, hT = a\Jsinh2 cr + sin2 (A.87) Kontravariantní metrický tenzor diagonální metriky bude tenzor s převrácenými hodnotami prvků na hlavní diagonále. Jakobián souřadnicové transformace z kartézské do eliptické soustavy bude J = a3 (sinh2 o + sin2 r) cosh o cos r = a3 (cosh2 o — cos2 r) cosh o cos r, (A. Príloha A. Křivočaré souřadnice 143 jakobiánem zpětné transformace bude výraz J 1. Nenulové Christoffelovy symboly eliptické metriky (viz rovnice A.12) budou, 2 (smh cr + sin rj 2 [sinhAa + srn rj v v rCT _ sinh 2cr cos2 r _ cosh2crsin2T 0 0 vv 2 (smír o + sir r) vv 2 (smír cr + sir r) 9 9 Diferenciální operátory gradientu skalární funkce, divergence a rotace vektoru a laplaciánu budou mít (s použitím formalismu Laméových koeficientů pro ortogonální soustavy a také rovnic A.14 a A.20) v této eliptické souřadné soustavě postupně tvar, / §1 91 §1 \ V f = —== 9(7 -^-, dT = , (A.90) V a V sinh2 a + sin2 r acoshcrcosr a-^/sinh2 cr + sin2 r / ^- ( \Jsinh2 cr + sin2 r cosh cr ACT + V-A = d a (sinh a + sin r) cosh a + a cosh cr cos r + \Jsinh2 cr + sin2 r cos r A7 a (sinh2 cr + sin2 r) cos r (A.91) V x A cosh crj^ (cos r A^) — \Jsinh2 cr + sin2 r ^áf- + - a \/sinh2 cr + sin2 r cosh cr cos r ( Vsinh2 cr + sin2 r AT j - Jp (Vsinh2 cr + sin2 r ACT a (sinh cr + sin r) 0+ + - Vsinh2 cr + sin2 r Aéf - cos r^(cosh cr A^) t \/sinh2 cr + sin2 r cosh a cos - T, (A.92) A _9_ (cosh i + _9_ Or COS T <9r (sinh2 cr + sin2 t) cosh cr cos r + Ô2 dip_ ; cosh2 cr cos2 r (A.93) Ostatní operátorové identity a geometrické parametry odvodíme analogickým způsobem jako v případě válcové nebo sférické souřadné soustavy A.5 Parabolická soustava Parabolická souřadná soustava je ve dvourozměrné verzi (viz obrázek A.4) definována dvěma třídami parabolických souřadnicových křivek s konstantními parametry u & v (toto značení opět není zcela ustálené, v různých literaturách může být různé) a se společným ohniskem v bodě (0, 0). V trojrozměrné verzi přibude ještě (válcová symetrie vzhledem k ose z) azimutální úhlový parametr 0. Príloha A. Křivočaré souřadnice 144 Obrázek A.4: Schéma dvourozměrné parabolické soustavy v rovině x,y. Modře vyznačené jsou křivky s konstantním parametrem u = 0,5 (nejužší); 1; 1,5; 2; 2,5; 3 (nejširší), červeně vyznačené jsou křivky se stejnou posloupností konstantních parametrů v. V trojrozměrném případě (viz popis) potom vyobrazenému směru y odpovídá směr z. u2 — v2 Transformační rovnice v trojrozměrném případě budou x = uv cos 0, y = uv sin 0, z = Zpětné transformační vztahy v pravotočivém pořadí proměnných u,v,(f> budou mít tvar (A.94) \/x2 + y2 + z2 + z, v-Metrická forma parabolické soustavy bude mít tvar \J x2 + y2 + z2 — z, y arctg —. x ds2 = (u2 + v2) (du2 + dv2) + u2v2 d4>2. (A.95) (A.96) Kovariantní metrický tenzor Qíj a příslušné Laméovy koeficienty parabolické souřadné soustavy v pořadí směrů u, v, -"f v ďW + (A.103) Ostatní operátorové identity a geometrické parametry odvodíme obdobně jako v případě válcové nebo sférické souřadné soustavy. A.6 „Anuloidová" soustava V tomto případě také nebudeme uvádět úplný popis všech vztahů a operátorů, i s ohledem na to, že daná soustava je příliš „specifická", resp. týká se pouze jednoho typu geometrického tělesa, tzv. anuloidu (toroidu) - viz obrázek A.5 (popis soustavy rovněž odkazuje k příkladům 6.49 a 6.60). Ukážeme pouze, jak je možné flexibilně adaptovat principy, odvozené pro předchozí „univerzální" geometrické systémy na (v podstatě jakýkoli) speciální případ. Anuloidem nazýváme 'p = \jx2 + y2 Obrázek A.5: Příčný řez anuloidem v rovině p-z, jednotlivé směry odpovídají válcové soustavě. Príloha A. Křivočaré souřadnice 146 těleso, které vznikne rotací kružnice okolo osy, která leží v rovině této kružnice a nemá s ní společný bod (vznikne tak válcově symetrická trubice - torus, připomínající „duši pneuma-tiky"). Označíme-li R poloměr osy toru, a poloměr trubice (toru), r radiální vzdálenost uvnitř trubice vzhledem k ose trubice, t úhlovou souřadnici vnitřku trubice a značení ostatních směrů bude odpovídat standardní cylindrické notaci, tj. p bude odpovídat radiální vzdálenosti od osy celého anuloidu, 0 bude azimutální úhel anuloidu a z vertikální souřadnice (vše je vyznačené v obrázku A.5), můžeme za anuloidové (proměnné) souřadnice považovat r,(f>,t (v pravotočivém smyslu). Transformační vztahy můžeme zapsat následovně, x = (R + r cost) cos 0, y = (R + r cosi) sin0, Vztahy pro zpětnou transformaci budou mít v tomto případě tvar y r siní. x2 + y2 R) + z2 arctg ■ t arcsm (A.104) (A.105) x2 + y2 R) +z2 Kovariantní metrický tenzor Qíj a příslušné Laméovy koeficienty anuloidové souřadné soustavy budou, 9íj 1 0 0 (R + r cosi)2 0 0 1, h $ = R + r cos t, ht (A.106) Vzhledem k tomu, že jde o diagonálni metriku, bude kontravariantním metrickým tenzorem g11 zpětné transformace rovněž tenzor s převrácenými hodnotami prvků na hlavní diagonále. Jakobián souřadnicové transformace z kartézské do anuloidové soustavy tedy bude J = r {R + r cos t), (A.107) jakobiánem zpětné transformace bude opět výraz J 1. Ostatní parametry lze snadno odvodit analogickým způsobem jako v předchozích soustavách. A. 7 Příklad neortogonální soustavy Podívejme se nyní na jiný možný geometrický případ, který může vyžadovat zavedení neortogonální souřadné soustavy. Jedná se o geometrický popis rozsáhlého plynného disku, rozprostírajícího se okolo velmi rychle rotující a tudíž silně zploštělé hvězdy, který je v blízkosti hvězdy velmi tenký a ve velkých vzdálenostech od hvězdy se výrazně vertikálně rozšiřuje. Zároveň je samozřejmě rotačně (válcově) symetrický. Obrázek A.6 schématicky znázorňuje tuto soustavu ve vertikální rovině p-6 (0 = konst.), souřadnicové směry zde jsou: p - radiální cylindrická souřadnice, 0 - azimutální úhel, 6 - sférický úhel, který je ovšem počítán v kladném a záporném směru od rovníkové roviny. Volné parametry (kromě zvoleného rovníkového poloměru hvězdy Req) jsou maximální cylindrická radiální vzdálenost p = i?max a maximální sférický úhel, označený jako 6max (zrcadlově k němu je #mm). Soustava je válcově symetrická, osa symetrie je kolmá k rovině disku (z = 0 A 6 = 0) a prochází středem hvězdy (p = 0). Můžeme ji tedy nazývat například cylindricko-kónickou soustavou5 (standardní, tzv. kónická souřadná soustava znamená něco poněkud jiného - jde o ortogonální soustavu, definovanou soustřednými 5 Jako zkrácený pracovní název budeme v dalším textu používat výraz disková soustava. Radiální a azimutální souřadnice jsou shodné se soustavou válcovou, jednotlivé souřadnicové směry tedy značíme p, , 9, jednotkové bázové vektory značíme p, (f>, 6. Príloha A. Křivočaré souřadnice 147 Obrázek A.6: Schématický obrázek cylindricko-kónické souřadné soustavy v rovině p-6 (cp = konst.). kulovými plochami a dvěma třídami vzájemně ortogonálních obecně eliptických kuželových ploch s osami x a z, s vrcholy v počátku souřadného systému). Transformační rovnice z této cylindricko-kónické do kartézské souřadné soustavy jsou (pro lepší grafickou přehlednost budeme v rovnicích této souřadné soustavy pro tangens používat v anglické literatuře zavedené označení tan, namísto v české literatuře běžného tg) x = pcoscf), y = psincf), z = ptané?. (A.108) Pro zpětnou transformaci z kartézské do diskové soustavy dostáváme6 p = \/x2 + y2, 0 = arctg—, 9 = arctg— . (A.109) x \/x2 + y2 Analogicky k rovnicím (A.30) a (A.57) budou mít jednotkové vektory diskové báze v kartézské soustavě tvar (viz pravidla pro sčítání vektorů) p = x cos 0 + ý sin 0, (f) = —x sin 0 + ý cos 0, 0 = — (x cos 0 + ý sin 0) sin 6 + ž cos 6. (A.110) 6V tomto místě platí totéž, co v případě válcových a kulových souřadnic. Príloha A. Křivočaré souřadnice 148 Zpětnou transformací jednotkových bázových vektorů (viz rovnice (A.110)) dostáváme p sin 9 + 0 x = p cos 0—0 sin 0, ý = p sin 0 + (f) cos < cos 9 (A.lll) V diskové soustavě není žádný z vektorů báze konstantní. Derivace bázových vektorů ve směru jednotlivých souřadnicových os budou (z rovnice (A. 110)) dp dp 00 # = o, 00 op 90 de n dě — = o, - op 90 -p, -0sin0, ^ = 0 ^ = 0 00 ' (A.112) de 00 p + Ô sin 0 cos 0 Časové derivace bázových vektorů budou d p d p 00 0Í ~ 00 0Í dej) 00 00 ~dt ~ 00 0Í dě _ dě 00 00 00 0Í ~ 00 0Í + 00 0Í -p4>, po cos 0 (A.113) >sin0 - 00tan0. Metrickou formu pro diskovou soustavu odvodíme diferencováním rovnice (A.108) P dx = cos 0 dp — p sin 0 d0, dy = sin 0 dp + p cos 0 d0, dz = tan 0 dp + cos 20 d9, (A.114) dosazením do rovnice (A.2) dostáváme nediagonální diskovou metrickou formu ve tvaru ds2 dp2 2psin0 , jn 2 / 2 d02 + ľ 0 n dpde + p2 d02 + (A.115) COS2 0 ' COS3 0 ^ 1 r ^ f ' cog4 q J Kovariantní a kontravariantní metrické tenzory soustavy se souřadnicemi v pořadí p, 0, 0 budou / i o cos2 9 $ 0 p psin 0 cos° 0 o p cos4 0 / 1 0 sin 0 cos 0 ■ cos3 0 cos4 0 / \ p p Jacobiho matice transformace z kartézské soustavy a matice inverzní transformace budou / cos0 sin0 0 \ 0 0 sin 0 cos 0 \ P 0 cos2 0 J (A.116) Ji: /cos0 —psin0 sin 0 p cos 0 ytan 0 0 0 \ 0 P J, -i cos 20/ COS sin < sm0 cos 0 0 P P „ cos 0 sin 0 cos 0 sin 0 sin 0 cos 0 cos 0 (A.117) Príloha A. Křivočaré souřadnice 149 příslušné jakobiány tedy budou, J = I det Ji. det gij\ P J -i det JI cos2 9 Nenulové Christoffelovy symboly diskové metriky jsou T<ť r-p'ť \ — re íre \ — ^ r'J — 1 p4> v1 4>p) — 1 p6» \l eP) — p > 1 00 — p, r det sin 9 cos 0. cos2 0 (A.118) (A.119) Protože se nejedná o ortogonální metriku (vyjádřenou diagonálním metrickým tenzorem), nedefinujeme zde žádné Laméovy koeficienty A.7.1 Diferenciální operátory • Gradient skalární funkce / = f(p, 0, 9) v diskové soustavě odvodíme stejným způsobem, jako v předchozích soustavách. Jednotlivé nenulové parciální derivace pro diskovou soustavu z rovnice (A.109) budou dp dx dp dy d9 dx d9 dy d9 dz \/x2 +y2 y \/x2~Ty2 COS Afj sin 9 d9 P d9 P (A.121) Divergence vektoru (vektorového pole) A (p, 0, 9) je v diskových souřadnicích opět definována jako skalární součin vektoru gradientu s obecným vektorem, tedy V • A d _|_ ^ 1 ® cos ^ & d p p d (f> p d9 ApP + A^cj) + Ae6 (A.122) Príloha A. Křivočaré souřadnice 150 kde ovšem, na rozdíl od ortogonálních systémů, nejsou obecně skalární součiny rozdílných vektorů báze nulové, tedy neplatí e^e3 = Sj. Jmenovitě v tomto systému bude nenulový soucm pe ■ sin 9. (A.123) Přímým výpočtem a po úpravách dostáváme -> -> 1 d , , , 1 dAa, cos 9 dAa sin 9 pop p o (p p oO p d , , \ ndAp -(p^ + cos*-^ (A.124) Na rozdíl od ortogonálních soustav není v tomto případě divergence jednoduchou stopou tenzoru gradientu vektorového pole (A.121), nýbrž je třeba ještě přičíst prvky na vedlejší diagonále (respektive ty, které odpovídají nenulovým prvkům metrického tenzoru (A.116)), násobené skalárním součinem příslušných jednotkových vektorů, v tomto případě rovnicí (A. 123). Rotaci vektoru (vektorového pole) A (p, 0, 9) v diskových souřadnicích nemůžeme odvodit podle rovnice (A.20) (soustava není ortogonální), v tomto případě musíme provést přímý výpočet z definice rotace vektoru, d a 1 d a cos 9 d VxA=^% + (f)-p^+eirď9^^p+A^+Ase (A.125) kde musíme nejprve provést všechny (nenulové) derivace jednotkových bázových vektorů (viz rovnice (A.112)), potom vektorové součiny. Ponecháme-li pouze nenulové komponenty, tj. vypustíme-li nulové derivace jednotkových bázových vektorů a také členy se stejnými bázovými vektory a tedy s nulovým vektorovým součinem, dostáváme explicitní výraz dp p ldAp\ x h(\dAe cos 9 d A, + 6 x p p d(f> p d9 cos 9 dAp dAe Ae p d9 dp p + (A.126) Vektorové součiny bázových vektorů zde ovšem nebudou tak jednoduché, jako v případě ortogonálních soustav, na základě rovnice (A. 110) pro sudé permutace dostaneme p sin 9 + 0 - -x p + 0 sin 9 - - pxcj) = ---—, cj)x6 = ---—, 6 x p = 4>cos9. (A.127 cos 9 cos 9 Po dosazení a úpravách dostaneme výslednou podobu rotace vektoru v diskové soustavě, V x A =p e tan 9 P cos 9 P d . . , dAp ď-p{pA^-^f 1 + - 1 dAe dA4 d9 ' 0dAp COS0^9~ p cos 9 ďp-^ d . . , dAp p \cos9 d + sin 9 ( 1 dAe d A, + p \cos9 d d9 (A.128) Príloha A. Křivočaré souřadnice 151 Laplacián odvodíme z rovnice divergence (A. 122), ve které nahradíme složky vektoru A odpovídajícími složkami vektoru gradientu z rovnice (A. 120), výsledný tvar (není nutné zde opakovat podrobný vektorový zápis, postup je zcela obdobný, jako v předchozích případech), zapsaný v kompaktní formě bude A - - 1 d ( d \ 1 d2 cos 9 d ( nd\ sin20 d2 ,A A = v-v = WP{%) + 7W +—dě {cos9oě )-—Wdě- (A-129) A.7.2 Plochy, objemy Stejně jako v předchozích soustavách odvodíme velikosti základních ploch a základního objemu prostorové buňky, tj. plochy a objem, ohraničené jednotlivými souřadnicovými rovinami (včetně stejného způsobu značení, další značení viz také obr. A.6). Objem jedné buňky souřadnicové sítě bude P2 02 6*2 3 3 V= [p2dp [dep /'-^ = ^2_^l(02-01)(|tan02|-|tan01|). (A.130) cos Pl Determinanty submatic metrického tenzoru, odpovídající jednotlivým plochám prostorové buňky (způsob značení je popsán v rámci popisu válcové a kulové soustavy) budou p2 p p Jp = 2~ä' ^ = 2~ä' ^0 = ä (A.131) ' cosz ti v cosz ti cos ti a plochy jednotlivých buněk sítě budou mít velikost 4>2 02 Sp = p2j d0y-^=p2(02-01)(|tanř?2|-|tan01|), (A.132) 01 01 d2 P2 o o /df) ľ n — n —^-ej pdp = ^-^(|tan02|-|tanř?1|), (A.133) 0i Pi P2 02 Q 1 ľ A ľ AA P? ~ Pl ~ ^) í A iq„\ 5"^ =-- pdp d(p= —---—. (A.134) cos ti J J 2 cos ti Pl 01 A.7.3 Vektory polohy, rychlosti a zrychlení Při popisu vektorů v diskové soustavě vyjdeme jako obvykle z jejich základního popisu v soustavě kartézské, zahrneme všechny rovnice pro derivace jednotkových vektorů i vektorových složek (rovnice (A. 108)-(A. 113)). Polohový vektor v diskové soustavě bude - . - . - pp + Špsmti r = xx + yy + zz = -t—-. (A.135) cosz ti Tento závěr již není tak názorný a snadno představitelný, jako v případě předchozích typů souřadnic. Vektor rychlosti v bude _ _ (p + ptitanti\ ~ ■ h (ptané pŠ \ v = p \ -Ta— + ^ + 6 -T +-3~B ■ (A.136 V cosz ti \ cos ti cosd ti / Príloha A. Křivočaré souřadnice 152 Vektory rychlosti a zrychlení musí být zároveň definovány jako dv v = vpp + va,(p + vgO, ä = — = app + a^4> + agQ. (A.137) Derivováním rovnice (A. 136) podle času dostáváme jednotlivé složky vektoru zrychlení v diskové souřadné soustavě p + tan 9[p9 + 29(p + pŠ tan ( cos2 9 ■b2 = 9 (A.139) Protože dv/dt = dv/dt + v ■ V v, můžeme napsat zrychlení, vyjádřené v diskové souřadné soustavě, pomocí složek vektoru rychlosti dv0 ot op p dvp v a, dvp cos 9 dvp + vg Vf)V0 s[n Q p 39 P + P ôva Ova v a Ova cos 9 Ova vpva vaVq sin 9 ~sr + vp-^—1---+ Ve--ää ^----' ot op p o (p p o9 p p v-v-' dvg dvg va, dvg cos 9 dvg v@ sin 9 VpVg sin2 9 ~dt 1 + vp— H---— + ve--—---h op p 0(f> p 39 p p (A.140) (A.141) (A.142) (v-V) ve Členy na pravých stranách rovnic (A.140)-(A.142), spojené svorkou, vyjadřují (nelineární) ad-vekci, zbývající členy reprezentující tzv. fiktivní (setrvačné) síly - odstředivá síla, Coriolisova síla, Eulerova síla. Porovnáním rovnic (A.108) a (A.139) můžeme zapsat složky vektoru rychlosti vp, v^, vg v diskové soustavě pomocí složek vektoru rychlosti vPjCy\, v^tCy\, vz ve standardní válcové souřadné soustavě (odstavec A.2). Dostáváme tak vzájemný vztah mezi velikostmi složek rychlosti v obou soustavách, v p = vPj cyi + - vz = vPj cyi + vz tan 9, v a, = cyi, ve \fp2~+ P cos 9 (A.143) Vezmeme-li dále v úvahu vertikální hydrostatickou rovnováhu v takovém disku, vz = 0, pohybové rovnice (A.140)-(A.142) budou identické s odpovídajícími pohybovými rovnicemi (A.52)-(A.54) ve standardní válcové geometrii. Příloha B Stručný úvod do parciálních diferenciálních rovnic Parciální diferenciální rovnice jsou, na rozdíl od obyčejných diferenciálních rovnic (viz kapitola 3), diferenciální rovnice, obsahující parciální derivace funkcí více proměnných. Jedná se například o rovnice vývojové (transportní - viz příloha C), které jsou jednosměrné v čase a zpravidla směřují k nějakému ustálenému stavu - rovnice 1. řádu (např. tzv. Burgersova rovnice) nebo o rovnice 2. řádu, tedy tzv. parabolické parciální diferenciální rovnice, nebo o parciální diferenciální rovnice, popisující periodické děje (vlnová rovnice) - tzv. hyperbolické parciální diferenciální rovnice, nebo se jedná o tzv. eliptické parciální diferenciální rovnice (Poissonova rovnice, Laplaceova rovnice), atd. Dělení parciálních diferenciálních rovnic na jednotlivé typy je i z praktického hlediska podstatné, poněvadž každý z nich se zpravidla řeší jiným způsobem. B.l Parciální diferenciální rovnice 1. řádu B.l.l Homogenní parciální diferenciální rovnice 1. řádu Nejjednoduššími parciálními diferenciálními rovnicemi jsou lineární homogenní rovnice 1. řádu dvou nezávisle proměnných x, y, vyskytují se zde tedy pouze první (parciální) derivace v lineárním výrazu Řešením takové rovnice bude funkce u(x,y). Funkci dvou proměnných, reprezentovanou plochou, můžeme charakterizovat pomocí vrstevnic x = x (s), y = y (s), kde s je parametr. Funkce u[x(s), y (s)] je tedy na vrstevnicích konstantní, můžeme ji považovat za funkci jedné proměnné (parametru s), du[x(s),y(s)] = dudx | dudy _ ^ ds dx ds dy ds ' kdy hledáme řešení systému obyčejných diferenciálních rovnic (tzv. charakteristické soustavy) = a{x,y), ^ = b(x,y), (B.3) ds ds které označujeme jako charakteristiky (také 1. integrál). Obecnou rovnici charakteristik potom definujeme jako tp(x, y) = C a obecné řešení rovnice dvou proměnných lze zapsat jako u(x, y) = 153 Príloha B. Stručný úvod do parciálních diferenciálních rovnic 154 <í> [tp(x, y)~\, kdy funkci <í> lze považovat za libovolnou funkci jedné proměnné ip. V případě rovnice n nezávisle proměnných bude mít obecné řešení tvar u(xi,...,xn) = $[ifi (xi,..., xn),. . . , ifn-l(xi, ..., xn)]. (BA) • Příklady řešení lineárních homogenních parciálních diferenciálních rovnic: 1. Mějme zadanou jednoduchou homogenní rovnici dvou nezávisle proměnných, ^+^ = 0, (B.5) ox oy charakteristická soustava tedy bude dx/ds = x2, dy/ds = y2, jejím řešením budou charakteristiky — l/x = s + Ci, —l/y = s + C2 a po vyloučení parametru s dostáváme l/y — l/x = C = ip(x,y). Výsledné obecné řešení tedy bude u(x,y) = $(I-I). (B.6) \y x) 2. Jiný jednoduchý příklad může představovat například homogenní rovnice £ = 6Ä (B.7) ox oy jejíž charakteristická soustava bude dx/ds = 1, dy/ds = — 6x2, kdy řešením první rovnice soustavy bude charakteristika x = s + C\ a protože dy = —6 (s + Ci)2ds, druhá charakteristika bude y = — 2s3 — 6s2 C\ — 6sCf + C2. Vyjádříme-li z první charakteristiky s = x — Ci a tento výraz dosadíme do druhé charakteristiky, dostáváme rovnici y + 2x3 = 2 Cf + C2 = C = (f(x,y). Výsledné obecné řešení tedy bude u(x,y) = $(y + 2x3). (B.8) K tomuto výsledku lze ovšem dospět mnohem rychleji, uvědomíme-li si, že v případě homogenní rovnice dostaneme vydělením rovnic charakteristické soustavy obyčejnou diferenciální rovnici 1. řádu, tedy (dy/ds)/(dx/ds) = dy/dx = —6x2 a tedy y = —2x3 +C. 3. Mějme zadánu homogenní rovnici tří proměnných x, y, z, .du .du .du . . (,-^ + (x-^ + (y-x)^=0, (B.9) s okrajovou podmínkou u(0, y, z) = yz. Charakteristická soustava v tomto případě bude dx/ds = (z — y), dy/ds = (x — z), dz/ds = (y — x), po jejím sečtení dostáváme dx/ds + dy/ds + dz/ds = 0 a po integraci podle s dostáváme x + y + z = C\. Protože zadaná rovnice obsahuje tři proměnné, potřebujeme ještě jednu obecnou rovnici charakteristik, například vynásobením každé charakteristiky odpovídající proměnnou dostaneme výrazy x dx/ds = (z — y)x, y dy/ds = (x — z)y, z dz/ds = (y — x)z. Po jejím sečtení (opět s nulovým součtem), po její integraci podle s a po vynásobení dvěma (kdy x' = dx/ds, atd.) dostáváme 2xx' + 2yy' + 2zz' = 0 a tedy x2 + y2 + z2 = C zavedeme počáteční (okrajovou) podmínku, například u(x, 0) = x. Potom můžeme psát u(x, 0) = C2 [x — u(x, 0) • 0] = x a tedy C2{x) = x. Dostáváme rovnici u = x — ut, výsledné jednoznačné řešení v tomto případě bude je libovolná funkce charakteristik. Přepíšeme nyní rovnici pomocí okrajové podmínky jako funkci charakteristik a konstant, tj. <í>(a — x, 1 — x2/2) = <ř(Ci, C2) = 0, z první charakteristiky potom vyplývá x = a — C±, z druhé charakteristiky dostáváme C2 = l-(a-Ci)2/2. Poslední výraz můžeme přepsat jako l-(a-Ci)2/2-C2 = &(Ci,C2) = 0, po dosazení do charakteristik dostaneme explicitní výraz 1— [a2 —2a(y —x) + (y —x)2]/2 — u + x2/2 = 0. Výsledné řešení okrajové úlohy pro zadanou rovnici potom bude u(x, y) = xy + &{y - x) - V + 1. (B.22) 2. Nehomogenní parciální diferenciální rovnice dvou nezávisle proměnných má tvar y—--x~q~ = y2 ~ x<2■< s okrajovou podmínkou u(x, a) = x2 — a2, (B.23) kde a je konstanta. Ze systému rovnic (B.19) vyplývá charakteristická soustava dx/ds = y, dy/ds = —x, du/ds = y2 — x2. Všimněme si, že v tom případě platí y dx/ds+x dy/ds = du/ds. Rovnice dy/dx = —x/y, její integrace dává první charakteristiku x2 + y2 = C\. Rovnici y dx/ds + x dy/ds = du/ds můžeme zapsat jako d{xy)jds = du/ds, po její integraci dostáváme druhou charakteristiku u — xy = C2. Obecné řešení parciální diferenciální rovnice bude mít tvar: §(x2+y2,u - xy) =0, (B.24) kde <í> je libovolná funkce charakteristik. Přepíšeme nyní opět rovnici pomocí okrajové podmínky jako funkci charakteristik akonstant, tj. <ř(x2+a2, x2 — a2 — &x) = Q(Ci,C2) = 0. Z první charakteristiky vyplývá x = ±\/C± — a2, z druhé charakteristiky potom vyplyne rovnice pro obě charakteristiky ve tvaru C\ — 2a2 =F a\/Ci — a2 — C2 = 0. Po dosazení původních výrazů do charakteristik dostaneme explicitní výraz x2 + y2 — 2a2 =F a(±\Jx2 +y2 — a2) — u + xy = 0. Výsledné řešení okrajové úlohy pro zadanou rovnici bude u(x, y) = x2 + y2 + xy — &\J x2 + y2 — a2 — 2a2. (B.25) Príloha B. Stručný úvod do parciálních diferenciálních rovnic 157 • Příklady řešení nelineárních nehomogenních parciálních diferenciálních rovnic: 1. Nehomogenní nelineární parciální diferenciální rovnice dvou nezávisle proměnných má tvar du du , í a2 "\ xu——h yu— = —xy, s okrajovou podmínkou u [x, — = h, (B.26) ox oy \ x J kde a, h jsou konstanty Ze systému rovnic (B.19) vyplývá charakteristická soustava dx/ds = xu, dy/ds = yu, du/ds = —xy. Integrace rovnice dy/dx = y/x dává první charakteristiku y j x = C\. Všimněme si, že v tomto případě platí y dx/ds + x dy/ds = 2uxy, tuto rovnici můžeme tedy zapsat jako d(xy)/ds = — 2udu/ds = — du2/ds, po její integraci dostáváme druhou charakteristiku u2 + xy = C2. Obecné řešení parciální diferenciální rovnice bude mít tvar: §(V-,u2 + xy} =0, (B.27) kde <í> je libovolná funkce charakteristik. Přepíšeme nyní opět rovnici pomocí okrajové podmínky jako funkci charakteristik a konstant, tj. <ř(a2/x2, h2 + a2) = &(Ci,C2) = 0. Z první charakteristiky vyplývá x = ±^&2/Ci, z druhé charakteristiky potom vyplyne rovnice pro obě charakteristiky ve tvaru h2 + a2 — C2 = 0. Po dosazení původních výrazů do charakteristik dostaneme explicitní výraz h2 + a2 — u2 — xy = 0. Výsledné řešení okrajové úlohy pro zadanou rovnici bude u(x, y) = ydi2 + a2 - xy. (B.28) 2. Nehomogenní nelineární parciální diferenciální rovnice dvou nezávisle proměnných má tvar du du yu—--xu— = x — y, s okrajovou podmínkou u(x, x) = h, (B.29) ox oy kde h je konstanta. Ze systému rovnic (B.19) vyplývá charakteristická soustava dx/ds = yu, dy/ds = —xu, du/ds = x — y. Opět zde integrace rovnice dy/dx = —xjy dává první charakteristiku x2 + y2 = C\. Rovnici dx/ds + dy/ds = d[x + y)/ds můžeme zapsat jako d(x + y)/ds = u{x — y) = u du/ds, po její integraci dostáváme druhou charakteristiku u2 + 2x + 2y = C2. Obecné řešení parciální diferenciální rovnice bude mít tvar: $(x2+y2,u2+ 2x+ 2y) = 0, (B.30) kde <í> je libovolná funkce charakteristik. Přepíšeme nyní opět rovnici pomocí okrajové podmínky jako funkci charakteristik a konstant, tj. <ř(2x2,h2 + 4x) = $(Ci,C2) = 0. Z první charakteristiky vyplývá x = ±-^/C\/2, z druhé charakteristiky potom vyplyne rovnice pro obě charakteristiky ve tvaru h2 ± 4^/C\/2 — C2 = 0. Po dosazení původních výrazů do charakteristik dostaneme explicitní výraz h2 + 2\[2\Jx2 + y2 — u2 — 2x — 2y = 0. Výsledné řešení okrajové úlohy pro zadanou rovnici bude u(x,y) = \J2yj2(x2 + y2) - 2x - 2y + h2. (B.31) Analogickým způsobem lze řešit (téměř) jakoukoli parciální diferenciální rovnici 1. řádu. Podstatné je vždy nalezení jisté symetrie v zadání rovnice, která umožní sestavení charakteristických rovnic a nalezení příslušných charakteristik. Zájemce o hlubší porozumění této problematice odkazuji například na skripta Arsenin (1977); Pospíšil (2006); Franců (2011). Príloha B. Stručný úvod do parciálních diferenciálních rovnic 158 B.2 Parciální diferenciální rovnice 2. řádu B.2.1 Klasifikace parciálních diferenciálních rovnic 2. řádu Obecná parciální diferenciální rovnice 2. řádu funkce u(x, y) (pro jednoduchost se omezíme pouze na funkce dvou proměnných) má tvar: 02u 02u 02u M*, v)— + ffli2(^ v)q^ + «22(x, y)^2 + dx2 du du (B.32) nebo, ve zjednodušené notaci, používane v dalším textu (ux — dujdx^ iixx = d2u/dx2, atd.) : an{x, y)uxx + a12{x, y)uxy + a22(x, y)uyy+ + bi(x, y)ux + b2(x, y)uy + c(x, y)u + d(x, y) = 0. Typ rovnice (v případě funkce dvou proměnných) je určen následujícími podmínkami: (B.33) «11 cl22 — ai2 = 0 rovnice parabolická, ana22 — ai2 < 0 rovnice hyperbolická, ana22 — ai2 > 0 rovnice eliptická. (B.34) (B.35) (B.36) Úpravou obecného tvaru rovnice transformací do nových proměnných prostřednictvím kvadratické formy lze získat kanonický tvar rovnic: an(x, y)ux — a22(x, y)uyy + ... = 0 rovnice parabolická, au(x,y)uxx — a22(x,y)uyy + • • • = 0 rovnice hyperbolická, au{x,y)uxx + a22(x,y)uyy + • • • = 0 rovnice eliptická. (B.37) V případě rovnice více proměnných je situace komplikovanější, typ rovnice je jednoznačně určen tzv. maticí kvadratické formy, resp. druhem její definitnosti. Příkladem transformace obecného polynomu druhého stupně na kvadratickou formu může být: 3x2 + 2xy + 2y2 = 3 ( x2 + ^xy + ^y2 Substitucí , (x + \y = £i v = Í2 x + ^y 1 o 2 o -y + -y 9y 3y 1 x + -v ) + dostáváme 3£2 H— £|> coz ^ze zapsat jako: 3 3 [x + -y) + -y (B.38) (6 £ (B.39) Obdobným způsobem můžeme transformovat obecnou parciální diferenciální rovnici 2. řádu: pokud je diagonální matice kvadratické formy pozitivně nebo negativně definitní, tj. její vlastní hodnoty (viz rovnice (2.17)-(2.19)) jsou buď všechny kladné nebo všechny záporné, potom se jedná o rovnici eliptickou. Pokud je diagonální matice kvadratické formy indefinitní (tj. kdy Príloha B. Stručný úvod do parciálních diferenciálních rovnic 159 některé vlastní hodnoty jsou kladné, některé záporné), potom se jedná buď o rovnici hyperbolickou (odlišuje se znaménko pouze jedné vlastní hodnoty) nebo ultrahyperbolickou. Pokud je diagonální matice kvadratické formy semidefinitní (některé vlastní hodnoty jsou nulové), potom se jedná se o rovnici parabolickou (jedna vlastní hodnota je nulová), případně tzv. parabolickou v širším smyslu. Kanonický tvar jednotlivých typů rovnic, například pro obecnou funkci čtyř proměnných u = u(xi,X2,x$,x 4), vypadá potom schématicky následovně: d2u d2u d2u du , , , , + + ± ô--1"......= 0 parabolická, (B.40) dx\ dx\ dx2 0x4 d2u d2u du du /t, —t H--~ ± —— ± —--h......= 0 parabolická v širším smyslu, (B.41) oxf OX2 0x3 0x4 d2u d2u d2u d2u m 1T2 +1T2 +1T2 ~ 1T2 += 0 hyperbolická, (b.42) oxf OX2 OX3 ox\ Q2u Q2u Q2u Q2u ——~ + ——9 — t-^t — t-^t +......= 0 ultrahyperbolická, (b.43) oxf OX2 OX3 ox\ d2u d2u d2u d2u in + ir2+ir^ + ir^ +......= 0 eliptická. b.44 oxf OX2 OX3 ox\ V dalším výkladu ukážeme řešení některých vybraných parciálních diferenciálních rovnic parabolických, hyperbolických a eliptických. • Fyzikální podoba parabolických parciálních diferenciálních rovnic Nejobvyklejší tvar parabolické parciální diferenciální rovnice (např. rovnice vedení tepla) je: ut = k(uxx +uyy + ...), (B.45) kde „konstanta" k (což nemusí být doslova konstanta, člen k pouze neobsahuje funkci proměnných x, y,...) má význam: k = X/(cpp), kde A znamená součinitel tepelné vodivosti, cp tepelnou kapacitu (při stálém tlaku) a p hustotu. B.2.2 Metoda fundamentálního řešení (metoda Greenovy funkce) Řešení rovnic pomocí formalismu Fourierovy transformace a konvoluce funkcí, zavedených v odstavci 9.2, si detailněji ukážeme na následujících řešených příkladech (v dalším textu budeme vždy uvádět zkratkami LS levou stranu rovnice a PS její pravou stranu) parabolických parciálních diferenciálních rovnic: • Homogenní rovnice, nehomogenní obecná počáteční podmínka: Homogenní úlohou rozumíme rovnici bez bez zdroje tepla, tj. bez pravé strany, nehomogenní pravá strana znamená dodatečný zdroj tepla. V případě homogenních počátečních respektive okrajových podmínek je příslušná funkce v čase t = 0, případně na definovaných okrajích, nulová. t > 0, x G M, s nehomogenní počáteční podmínkou u(0, x) = (f(x). (B.46) Príloha B. Stručný úvod do parciálních diferenciálních rovnic 160 oo LS: ut(t,0= ut(t,x)e-ixíídx = ut{t,x). (B.47) J —oo oo f00 r 100 ľ PS: uxx(t,£)= / uxx{t,x)e-'lxtdx = ux e_ia* +i£ / ux(t, x) e_ia* dx = -f u(£). -00 -__- -°° J v —00 (B.48) Dostáváme tedy obyčejnou diferenciální rovnici 1. řádu s jednoduše separovatelnými proměnnými: ut(Q = —a2^2 jejíž řešení snadno určíme jako u(£) = C e~a ^ t, respektive u(t, £) = C(0e~a C *• Funkci C(0 určíme z počáteční podmínky (B.46): u(0, £) = 0, x é R, s homogenní počáteční podmínkou u(0, x) = 0. Předpokládáme řešení ve tvaru: u(t,x) = / w(t,x,a) da, kdy Jo ut(t,x) = w(t,x,t) + / wt(t, x, a) da, uxx{t, x) = / wxx(t,x,a) da, potom platí Jo Jo w(t,x,t)+ / wt(t, x, a) da = a2 / wxx(t, x, a) da + f(t, x), to odpovídá Jo Jo [wt(t, x, a) — a2wxx(t, x, 0,iě1,s nehomogenní počáteční podmínkou u(0, x) = ip(x). (B.54) Z linearity vyplývá, že funkci lze rozdělit následujícím způsobem: u(t, x) = v(t, x) + w(t, x), (B.55) vt(t, x) = a2vxx + /, v(0,x)=0 1. funkce, (B.56) wt = a2wxx, w(0,x) = (f(x) 2. funkce. (B.57) Z počáteční podmínky (p(x) = v(0, x) + w(0, x) = w(0, x), kde ovšem v(0, x) = 0, dostáváme: /OO f't f'00 V>(y)G(x,y,t)dy+ / f(a, y) G(x, y, t - a) da dy = -oo JO J—oo 2aV7rí f°° (x-y)2 1 ľ* f°° fa y) _Jx-yT_ / (p(y)e ~^Tdy + — / J) ,y' e ^^dady. (B.58) J-oo 2a J0 J_OQ y/t — a B.2.3 Řešení parabolických parciálních diferenciálních rovnic Fourierovou metodou (metodou separace proměnných) Metodu, která se velmi často používá při řešení parciálních diferenciálních rovnic 2. řádu, si opět detailněji ukážeme na řešených příkladech parabolických parciálních diferenciálních rovnic. • Homogenní jednorozměrná úloha, homogenní okrajové podmínky, obecná počáteční podmínka: ut = a2uxx, t > 0, x e (0,£), u(0,x) = tp(x), u(t,0) = 0 = u(t, i). (B.59) Předpokládáme, že funkci u(t, x) lze vyjádřit jako součin dvou separovaných funkcí, kdy každá je funkcí jen jedné z obou proměnných: u(t,x) = T(ť)X(x). Obě strany rovnice (B.59) lze potom vyjádřit následujícím způsobem: T' X" T'X = a2TX", separujeme do podoby —^ = = —A. Řešení PS potom bude (B.60) X" + XX = 0, z toho vyplývá X(x) = v{x) = A sin VXx + B cos VXx. (B.61) Zahrnutím okrajové podmínky získáme příslušné koeficienty PS: X(0) =B = 0, X(í) = AsmVxt = 0, a tedy x/x = ^, (B.62) kde konstanta A může nabývat libovolné hodnoty (např. 1). PS můžeme tedy zapsat jako Xk = vk = sin í — xj . (B.63) LS řešíme jako obyčejnou diferenciální rovnici 1. řádu, T- = -a2X, z toho vyplývá T = Cfce~a2Afeí = C^e"^)2*, (B.64) Príloha B. Stručný úvod do parciálních diferenciálních rovnic 162 Obě takto nalezené separátní funkce potom zapíšeme jako součin: u(*>x) = ^Ck e~ sin k=l í ku (B.65) Fourierův koeficient Ck získáme pomocí počáteční podmínky: v{x) =^2Ck sin kir t ^Ckvk, a tedy Ck 1 \vk\r Jo (B.66) k=l v 7 k=l Normu \\vk\\ funkce vk řešíme jako normu spojitě definovaného vektoru (viz rovnice (2.1)), tedy vk fSin2(^)d^í,Cfc^^(0Sin(^)d,. (B. Výslednou funkci můžeme potom zapsat ve tvaru , \ 2 ^ ŕ nt,x) = il^ 1 k=iJo kir sin | ~x 67) (B.68) • Homogenní dvourozměrná úloha, homogenní okrajové podmínky, obecná počáteční podmínka: Vedení tepla v pravoúhlých směrech: ut = a2 (uxx +uyy), t > 0, x £ (0, í\), y 6 (0,4>), u(0,x,y) = (f(x,y), u(t, 0, y) = 0 = u(í, ^i, y), u(t, x, 0) = 0 = u(í, x, ^2)- (B.69) Předpokládáme tři funkce, které lze separovat (srovnej rovnici (B.60)), takže každá je funkcí jen jedné ze tří proměnných: u = TXY. Obě strany rovnice (B.2.3) lze potom vyjádřit následujícím způsobem: T'XY = a2(TX"Y + TXY"), tedy = ^ + I! = _(Al + A2). (B.70) azl X Y -Ai Y" Dále předpokládáme: X^_ ~X Y což si můžeme dovolit vzhledem k následným úpravám LS rovnice. Dostáváme tak řešení PS, -A, (B.71) Xm = sin 17171 mr x ) , Yn = sin [-j^y LS opět řešíme jako obyčejnou diferenciální rovnici 1. řádu, T_c -aa[Wa+Wa]* Všechny tři nalezené separátní funkce potom zapíšeme jako součin U[x,y,t) — / j ^~mn' m,n=l x srn um niv x ] srn I —y (B.72) (B.73) (B.74) Príloha B. Stručný úvod do parciálních diferenciálních rovnic 163 Fourierův koeficient získáme pomocí počáteční podmínky: 1 í'íx fÍ2 (mir£\ . (nirrj Cmn = j^—^ JQ JQ VÍZ, v) sin ^-j^ j sin j d£ dr,, (B.75) Norma ||wmn|| funkce vmn je analogicky k rovnici (B.67) dána jako "•"""l|2 = C Csin2 (¥)si"2 (x) dí d" = \ [íl°' x \ I* = '-T- 0, xe(0,£), u(0,x) = 0, u(t,0)=T1, u(t,í)=T2. (B.78) Funkci linearizujeme: u(t, x) = v(t, x) + w(t, x), kdy funkce w(t, x) přejde na stacionární funkci w(x) a bude splňovat okrajové podmínky následujícím způsobem: w(t, 0) = Ti, w(t, i) = T2, u(í, 0) = v{t, i) = 0. (B.79) Pro stacionární funkce dále platí, w{x) = Ti + T2 ~Tlx, v(0, x) = -Ti + Tl ~T2x, (B.80) v(0, x) + w{0, x) = 0 a tedy u(0, x) = -w(0, x). (B.81) Rovnice (B.78), rozdělená pro obě funkce v a w bude mít tvar vt = a2vxx, wt = a2wxx, (B.82) kdy ovšem prostorové derivace funkce w budou wx = 2 \ wxx = 0, vt = a2vxx, v = XT, (t, 0) = 0 = v(t, í), (B.83) X(x) = A sin y/\x + B cos \/~Ax a tedy X^ = sin (~~£~X^J ' (B.84) ^^^fte^ftsin^J. (B.85) Príloha B. Stručný úvod do parciálních diferenciálních rovnic 164 O i x Obrázek B.l: Schématické znázornění průběhu funkce w(x). Z počáteční podmínky dále vyplývá ;(0, x) = Cfc sin k=l ku -T\ H--—-—-x, z toho vyplývá: Ck £ -Ti +---x I srn I — £ ) d£ 2T A COS ( —£ 0 2Ti -l)fc- 1 2 Ti + 2(Ti - T2 £2 V i + T- [ cos f ^ ) d£ ku 2Ti -l)fc- 1 -ir-1 + 2 (Ti - r2 £2 i2 klT V ' V ' \k7T J KklT J (B.86) Výsledná funkce bude mít tvar: u(t,x)=T1+ 2~ ľx + ^2 k=l ku J V ku ku e x í > " sm | ~x (B.87) • Nehomogenní jednorozměrná úloha s konstantním zdrojem tepla Tq, s homogenními podmínkami: ut = a2uxx + T0, t > 0, x G (0, £), u(0, x) = 0, u(í, 0) = 0 = u(í, £). (B.88) (B.89) u(t,x)=TX, X(x) = A sin V\x + -B cos VXx, Xk{x) = Vk{x) = sin [ — x Zvolíme rovnici ve tvaru fc=i a pomocí další podmínky získáme nehomogenní obyčejnou diferenciální rovnici Cfc(i) + a2XkCk(t) = Fk(t), kde -Ffc(t) je Fourierův koeficient nehomogenity. (B.90) (B.91) Príloha B. Stručný úvod do parciálních diferenciálních rovnic 165 Tuto rovnici dále řešíme: f (t, x) = Y Fk(t) sin i^—xj a tedy Fk(t) = -J f(t,£) sin ľ^-i ) d£ cos — £ 2Jh l-(-l) Nejdříve řešíme homogenní rovnici 2 ČÁČ*) = -a2 (y) ck^a tedyCk^ = K® e~(Ä^)2í, z toho vyplývá Dosazením do nehomogenní rovnice dostáváme 2 _/ afc7T n 2 e v t * + if'(í)e-m * + Ä"(í)e" 27b 1 - (-11 , z toho vyplývá k'(t) 2Tr o e \ 2Tn s afc7r J kir Z počáteční podmínky Ck(0) = 0 vyplývá: 1- -1 k-k 1- -1 e{—) t + k2. Ck(t) = k2 i \ 2 2T( o akír J kir l-(-D ' akír \2 + k2e~y —) *, kde Cfc(0) = 0, tedy i Y2Ta s akír J kir Výsledná funkce bude mít tvar: u(t,x)=Y 1-e-m l-(-l) 2A 2T0 / í i \ 2 2T( k=l. kir \ akír 0 akír J kir l-(-D -l)k-l kir sin I Yx Cfe(i) (B.92) (B.93) (B.94) (B.95) (B.96) (B.97) (B.98) • Nehomogenní jednorozměrná úloha s nekonstantním zdrojem tepla, homogenní podmínky: ut = a2uxx +tx, t > 0, x G (0, £), u(0, x) = 0, u(t, 0) = 0 = u(t, £), (kiv \ Y xj ■ Opět řešíme rovnici ve tvaru: / kiľ k=l u(t,x) = YCk{t) sin ŕ—z ) , tedy Cfc'(í) + a2XkCk{t) = Fk(t). (B.99) (B.100) ÍB.101) Príloha B. Stručný úvod do parciálních diferenciálních rovnic 166 Tuto rovnici dále řešíme: /(*> x) = Fk 0, x £ (0, i), u(0, x) = (f(x), u(t, 0) = «i(í), u(í, £) = u2(t). (B.109) Príloha B. Stručný úvod do parciálních diferenciálních rovnic 167 Funkci u(t, x) opět rozložíme: u(t, x) = v(t, x)+w(t, x), kde w bude splňovat okrajové podmínky. vt + wt = a2vxx + a2wxx + / a tedy vt = a2vxx + f + a2wxx - wt. (B.110) y__• v nehomogenita ip(x) = v(0,x) + w(0,x) a tedy v(0,x) = (p(x) — w(0,x). (B.lll) Pomocí funkcí v a w řešíme úlohu v principu stejně jako v předchozích případech. B.2.4 Jednoduché příklady prostorových úloh • Teplota v nekonečném rotačním válci (použití Besselových funkcí) : ut = a2 (^UpP H—Up^j , radiální část laplaciánu ve válcových souřadnicích (A.46), (B.112) t > 0, p G (0, c) kde c je poloměr válce, u(0, p) = f (p), u(t, 0) = 0 = u(t, c). (B.113) Separací proměnných: u = R(p)T(t) a tedy = ^- + = -X2, (B.114) azl R p R po úpravě dostáváme PS: pR" + R' + X2pR = 0, LS: T' + a2\2T = 0. (B.115) Substitucí Xp = z dostáváme tzv. Besselovu rovnici s indexem v = 0, dR dR d2R i2d2i? , , d2 i? dR /„x — = A — , -—r- = A -—7, z toho vyplýva z-—r +---h zR = 0, (B.116) dp dz dpz dzz dzz dz kdy obecná Besselova rovnice má tvar x2y" + xy' + (x2 — v2)y = 0. Jejím řešením je funkce w = (2) E fc!r(t + fc + i) (2) 'z níž vyplývá Mx) = g W (2) ' (R117) /•oo kde výraz r(i/ + fc + l) je tzv. T funkce, definovaná jako T (x) = I e~ttx~1 dt. V našem případě Jo dostáváme řešení PS ve tvaru R(z) = Jo(z), a tedy R(p) = J0(Ap), a tedy J0(A„p) = Rn(p), (B.118) s okrajovou podmínkou Jo(Anc) = 0, kde Xn je kořenem této rovnice. Řešením LS bude funkce Tn(t) = e-a2A»*. (B.119) Pomocí tzv. Fourierova-Besselova rozvoje, definovaného jako YľkĹi Cn3v(Xnx) = f(x) v každém bodě spojitosti funkce f(x), získáme koeficient Cn (Fourierův-Besselův koeficient), kdy obecně platí: 2 fc Cn = ^~ô—77—š I š Jí/(AnO/(0 n = 1,2,3..., v našem případě tedy (B.120) cz3u+1(Xnc) Jo Cn= 2t2/x \ / ^Jo(AnO/(Odg- (B.121) c2Jf(Anc) Jo Príloha B. Stručný úvod do parciálních diferenciálních rovnic 168 Výsledená funkce bude mít podobu Jo(AnP) • Chladnutí koule (homogenní rovnice) : [ČJo(A„0/(OdČ] e" -a^Xít (B.122) O 0 0 0 0 Uf = a Au, t > O, x + y + z < R kde R je poloměr koule , u(0, x, y,z)= f (^\Jx2 +y2 + z2^ = f (r) a tedy u(0, r) = f (r), r G (O, R), u(t, x, y, z) = u(t, r), u(t, 0) = Ti, u(t, R) = 0. Jednotlivé parciální derivace budou: x ux = urrx + uvipx + Uflůx, z toho vyplývá tt^ = ur —, Au = Ur x2 +y2 + z2 x Urr ~ ~\~ U: r „2 + ur r2 — x2 + ur 3rz x2 + y2 + z2) urr -|—ur, z toho vyplývá ut = a I urr H—ur ) (sférická radiální část Laplaciánu - viz rovnice (A.72)). Pomocí substituce v (r) = ru(r) a tedy u = v/r dostáváme: 1 ut = -vt, ur r 1 1 -—^V + -Vr, Urr = —zV--tcV. ry, £t [y y>0 íy& 1 2 1 2 2 1 1 2 -^tV + -urr = —zv ty£t iy y>0 y> 1 2 2 2 1 ut = a urr -\—wr , tedy —vt = a —^--^vr -\—vrr---\—~vr , a tedy \ iy J iy \ fyřj íy£ iy fyřj fy £ J vt = a2vrr, kde v(0, r) = rf(r) v(t, 0) = rTi, v(t, R) = 0. Pomocí linearizace u = z + w dále dostáváme: u(0, r) = f(r), tedy z(0, r) = u(0, r) — w(r), w{r) = T\--— r, z(0, r) = f(r) — T\ + — r, ž(0, r) = rz(0, r). R R v(0,r) = rf(r), tedy z(0,r) = r f (r) ~ Ti + j^r dává: v = ru = r (z + w), zt = a zrr, z(t, 0) = z(t, R) = 0, z = TX, a tedy z(r,r) = ^TC^e-^) *sin f^-r) • fc=i Fourierův koeficient bude mít tvar: 2 fR /(r) " Tl + ^r sin ^"T^7, I dr. Výsledná funkce bude mít tvar: u(t,r) = Ti - rt -tí J o f(r) ~ Ti + e V r J sm I —r j dr. fc=i (B.123) (B.124) (B.125) (B.126) (B.127) (B.128) (B.129) (B.130) (B.131) (B.132) (B.133) (B.134) (B.135) (B.136) (B.137) Príloha B. Stručný úvod do parciálních diferenciálních rovnic 169 T Ti O R r Obrázek B.2: Schématické znázornění průběhu funkce w(r). B.2.5 Řešení hyperbolických parciálních diferenciálních rovnic Fourierovou metodou Následující dva jednoduché řešené příklady ilustrují princip použití této metody v případě hyperbolických PDR: • Homogenní vlnová rovnice : utt = a2uxx, t > 0, x 6 (0,£), s Cauchyho počátečními podmínkami (viz odstavec 3.1.1) u(0,x) = tp(x), ut(0,x) = tp(x) a s Newtonovými okrajovými podmínkami (viz odstavec 3.2.1), kde a, (3 7^ 0, a u(t, 0) + f3 ux(t, 0) = 0, a u(t, t) + f3 ux(t, i) = 0. Separací proměnných: T" X" u(t,x) = T(t)X(x) a tedy a2T X po úpravě dostáváme LS: T" + a2X2T = 0, PS: X" + X2X = 0. Z rovnice (B.142) dostáváme Tk{t) = ak cos (\ka t) + bk sin (\ka t), Xk(x) = ck cos (\k x) + dk sin (\k x), (B.138) (B.139) (B.140) ÍB.141) ÍB.142) (B.143) Obdobným způsobem jako v rovnici (B.62) dostáváme z Newtonových okrajových podmínek (B.140) a + /3A = 0, (3 — ol\ = (3 + a 7^ 0, z toho vyplývá sin (Xk x) = 0, a tedy \k Pro prostorovou funkci tedy dostáváme řešení (viz rovnice (B.63)) Xk = ck cos í —x j +dk sin í — x ku (B.144) (B.145) Príloha B. Stručný úvod do parciálních diferenciálních rovnic 170 Obecné řešení lze tedy zapsat ve tvaru oo u(t,x) = k=l . akir \ (akir Ak cos ——t + Bk sin I ——t £ ) * \ l Z Cauchyho počáteční podmínky (B.139) dostáváme oo u{0,x) = ^2A, ŕ kiľ \ f ku cos ( ~x J + snl I ~x (B.146) k=l f kiľ \ (ku cos I ~x J + sin I ~'C (x) (B.147) a z podmínky (B.139) dostáváme i \ \ ~~ Cífe7r Ut(0,x) = 2^ ~Bk k=l kir \ ( kir cos | ~jr~x J + sm I _^T'C l \ \ ~" Cřfc7T „ k=l (B.148) Fourierovy koeficienty Ak, Bk najdeme z rovnic (B.147) a (B.148) (viz také rovnice (B.66)), Ak 1 V(0vk(£)d£, Bk £ ak7r\\vk\ 4>(0 vk(0 d^. (B.149) o o Normu \\vk\\ funkce vk řešíme jako normu spojitě definovaného vektoru (viz rovnice (2.1)), tedy \vk\ 0 JO kiv \ f kir cos | — £ I + sm I — £ d£= / d£ = £. 'o (B.150) Po dosazení dostáváme rovnici (B.146) ve tvaru oo u(t,x) = r f ku \ í ku cos —-x + sm -—x k=l V i v ^ l l ^/^)^)cos(^t)d^ + -i-/ . o o ip(0vk(0 sin( )d£ (B.151) • Nehomogenní vlnová rovnice s homogenními počátečními podmínkami : uu = a2uxx + / (kde / je zdroj energie vlnění), t > 0, x G (0, £), s homogenními Cauchyho počátečními podmínkami (viz odstavec 3.1.1) u(0,x) = 0, ut(0,x) = 0 a s Newtonovými okrajovými podmínkami (viz odstavec 3.2.1), kde a, /3 7^ 0, au(í,0) + f3ux(t,0) = 0, au(t,£) + f3ux(t,£) = 0. Obdobně jako v předchozím příkladu: u(t,x)=TX, X{x) = Asin (Ax) + B cos (Ax), X^(x) = Vfc(x) = sin ( — x (B.152) (B.153) (B.154) (B.155) Príloha B. Stručný úvod do parciálních diferenciálních rovnic 171 Zvolíme rovnici ve tvaru: i(t,x) = J2ck(t) k=l vk{x) = ^2 Cfc(í) sin ( —a k=l kir (B.156) Pomocí další podmínky získáme nehomogenní obyčejnou diferenciální rovnici: Ck(t) + a2\2,Ck(t) = Fk(t) kde Fk(t) je tzv. Fourierúv koeficient nehomogenity, (B.157) no í ^ Ík-k \ 2 f f (t, x) = ^2 Fk(t) sin i^—xj a tedy Fk(t) = -J f(t,£) sin ( -f Š ) d£- (B.158) Dále bychom museli řešit nehomogenní diferenciální rovnici 2. řádu (B.157) (například metodou variace konstant - viz oddíl 3.2.1) pro konkrétní funkci Fk(t). Zahrnutím počátečních podmínek (B.153) dostáváme řešení rovnice (B.157) alespoň v obecné integrabilní podobě: Ck(t) i akir Fu(a) sin akir (t-a) (B.159) Jejím dosazením do rovnice obecného řešení (B.156) dostaneme (viz řešení rovnice obdobného typu v případě parabolických parciálních diferenciálních rovnic v oddíle B.2.3): OD t k=l V 7 n akir (t-a) (B.160) B.2.6 Ukázka možných způsobů řešení jednoduchých eliptických parciálních diferenciálních rovnic Následující řešené příklady demonstrují některé základní způsoby počítání eliptických PDR: • Laplaceova rovnice : Laplaceova rovnice je v kartézských souřadnicích v nejjednodušší formě definovaná ve tvaru uxx(x,y) + uyy(x,y) = 0. ÍB.161) V tomto příkladu budeme řešit Laplaceovu rovnici na obdélníkové oblasti s rozměry a, b, se smíšenými Dirichletovými a Neumannovými podmínkami (viz odstavec 3.2.1), v podobě u(x, 0) = 0, u(0, y) = 0, ux(a, y) = 0 (y / 0), uy(x, b) = u0 sin (^-z) (x / 0). (B.162) Separací proměnných u(x,y) = X(x)Y(y) dostáváme rovnici X" Y" X"Y + XY" = 0 a tedy — = -— = A, (B.163) kde konstanta A může nabývat hodnot A = 0, A > 0, A < 0. Príloha B. Stručný úvod do parciálních diferenciálních rovnic 172 1. A = 0: Budeme předpokládat separované funkce X{x) a Y{y) ve tvaru polynomů, vzhledem k okrajovým podmínkám budou dostatečné polynomy 1. stupně, tedy X{x) = Ax + B, Y{y) = Cx + D. Z okrajové podmínky u(0, y) = 0 vyplývá B = 0\/C = D = 0, pokud ovšem C = D = 0, potom Y{y) = 0 a tedy u(x, y) = 0 všude. Pokračujeme-li s B = 0, dostáváme Ax{Cy + D) = 0 a tedy, zahrneme-li další okrajovou podmínku u(x,0) = 0, musí být AxD = 0. Případ A = 0 dává X{x) = 0, tedy u(x,y) = 0 všude. Uvažujeme-li také D = 0, potom u(x, y) = AxCy = 0, z další okrajové podmínky ux(a, y) = 0 vyplývá ACy = 0, tedy A = 0VC = 0, v obou případech ovšem u(x, y) = 0. Případ A = 0 dává tedy pouze triviální řešení. 2. A > 0: Z rovnice (B.163) dostáváme obecné řešení ve tvaru , y) = A cosh(\/Äx) + B smh(y/~\x) C cos(\/Ä~y) + D sin(\/Ä~y) u[x, (B.164) Z okrajové podmínky u(0,y) = 0 vyplývá A[C cos(V\y) + Dsm(\/Xy)] = 0, tedy C = D = 0, potom ovšem y(y) = 0 a tedy y) = 0 všude. Pokračujeme-li s A = 0, dostáváme y) = B smh.{y/\x)\C cos{y/\y)+D sin(\/Ay)]- Zahrneme-li další okrajovou podmínku 0) = 0, musí být BCsmh(VXx) = 0. Případ B = 0 dává X(x) = 0, tedy u(x,y) = 0 všude. Pokračujeme-li s C = 0, potom u(x,y) = BD smh(y/\x) sin(y/\y) a z další okrajové podmínky ux(a,y) = 0 vyplývá y/XBD cosh(\/Aa) sin(\/Ay) = 0, tedy Í3 = 0VL> = 0, v obou případech ovšem u(x, y) = 0. Případ A > 0 dává tedy také pouze triviální řešení. 3. A < 0: Z rovnice (B.163) dostáváme obecné řešení ve tvaru u(x, y) = A cos(v/Ä~x) + B sm.(V\x) C cosh(\/Ä"y) + D sinh(\/Ä"y) (B.165) Z okrajové podmínky u(0,y) = 0 vyplývá A[C cosh(\/Ä"y) + -Dsinh(\/Ä~y)] = 0, tedy C = Ľ = 0, potom y (y) = 0 a tedy u(x,y) = 0 všude. Pokračujeme s A = 0, dostáváme u(x,y) = Bsm(y/Xx)[Ccosh(\/Ä~y) + Dsinh(\/Ä"y)]. Zahrneme-li další okrajovou podmínku u(x, 0) = 0, musí být BC sin(v/A~x) = 0. Případ B = 0 dává X(x) = 0, tedy u(x, y) = 0 všude. Pokračujeme-li s C = 0, potom u(x, y) = BD sm(V\x) sinh(\/Ä~y) a z další okrajové podmínky ux(a,y) = 0 vyplývá y/XBD cos(\/A~a) sinh(\/Ä"y) = 0, tedy B = 0Vľ = 0(v obou případech ovšem u(x, y) = 0) V cos(\/Ä"a) = 0. Poslední případ dává řešení cos sin (y/Xa (2k r- (2fe-l)7T 0, tedy VA = ^—-—'— k=l 1 7T 2a sinh (2k 2a 1W 2a Uplatníme-li také čtvrtou okrajovou podmínku, dostáváme 2^ /ífc sm k=l 7T sin-x U 2a 2a sinh a tedy , kde Kfc (2fc - 1)tt 2a (B.166) Z argumentů funkce sinus vyplývá řešení pouze pro k = 1, tedy i^i Výsledné řešení se zahrnutím všech okrajových podmínek bude -i u(x,y) = u0 sinh (— | V 2a J sin — smh — 2a J V 2a (B.167) uq j sinh[7r6/(2a)]. (B.168) Príloha B. Stručný úvod do parciálních diferenciálních rovnic 173 Další typickou eliptickou parciální diferenciální rovnicí může být například tzv. Poissonova rovnice typu Au(x, y) = f{x, y), tedy nehomogenní eliptická rovnice, nejčastěji používaná ve formě gravitační Poissonovy rovnice, A<ř = AirGp, kde <í> je gravitační potenciál, p je hustota hmoty a G je gravitační konstanta, nebo Poissonovy rovnice elektrostatického potenciálu, A<ř = —p/e, kde p je hustota elektrického náboje a e je permitivita. Řešení vícerozměrné Poissonovy rovnice je analogické k řešení Laplaceovy rovnice a také například k řešení nehomogenní hyperbolické parciální diferenciální rovnice: • Poissonova rovnice s konstantní pravou stranou: Řešme jednoduchou rovnici, definovanou na oblasti x > y2, tj. na oblasti ohraničené parabolou x = y2 s vrcholem v bodě (0, 0), jejíž osu tvoří kladná část osy x, Uxx(x,y) + uyy(x,y) = 2, (B.169) s Dirichletovou podmínkou na hranici oblasti, u{y2,y) = 0. Předpokládejme řešení ve formě všech členů polynomu 2. stupně s neurčitými koeficienty, u(x, y) = A + Bx + Cy + Dx2 + Exy + Fy2, (B.170) kdy po jeho parciálním derivování ve smyslu rovnice (B.169) snadno zjistíme: F = 1 — D. Po dosazení okrajové podmínky, tedy z rovnice A + Cy + {B - D + 1) y2 + Ey3 + Dy4 = 0, (B.171) dostáváme nenulové hodnoty koeficientů pouze pro B = — 1, F = 1. Hledaná rovnice tedy bude u(x, y) = y2 - x. (B.172) • Poissonova rovnice s konstantní pravou stranou na kruhové oblasti, s nehomogenní okrajovou podmínkou: Uvnitř kruhové oblasti s poloměrem R platí následující rovnice, Uxx(x,y) + uyy(x,y) = 4, (B.173) kdy na hranici oblasti platí Dirichletova podmínka u(x, y±) = 1, z níž vyplývá y\ = ±VR2 — x2. Analogicky k parabolickým rovnicím s nehomogenními okrajovými podmínkami rozdělíme hledanou funkci u{x,y) na součet dvou funkcí, například U{x,y) a v{x,y), pro které bude platit: u(x, y) = U (x, y) + v(x, y), Uxx + Uyy = 4, U (x, yi) = 0, vxx + vyy = 0, v(x, y{) = 1. (B.174) Obdobně jako v předchozím případě předpokládáme pro každou funkci úplný polynom 2. stupně s neurčitými koeficienty, U(x, y) = A + Bx + Cy + Dx2 + Exy + Fy2, (B.175) v(x, y) = a + bx + cy + dx2 + exy + fy2, (B.176) což dává F = 2 — D, f = —d. Po dosazení okrajové podmínky můžeme rovnice (B.175) a (B.176) přepsat ve tvaru A + Bx ± (C + Ex) \JR2 - x2 + 2 (D - 1) x2 + (2 - D) R2 = 0, (B.177) a + bx±(c + ex) \JR2 - x2 + 2dx2 - dR2 = 1, (B.178) jednotlivé nenulové koeficienty budou: A = —R2, D = 1, F = 1, a = 1. Po sečtení rovnic (B.177) a (B.178) dostáváme hledanou výslednou funkci u{x,y) = 1 - R2 +x2 +y2. (B.179) Príloha B. Stručný úvod do parciálních diferenciálních rovnic 174 • Poissonova rovnice s obecnou pravou stranou, Newtonovy okrajové podmínky: Řešme obdobným způsobem Poissonovu rovnici ve tvaru uxx(x, y) + Auyy(x, y) = xy, (B.180) s okrajovými podmínkami u(0,y) = y2, ux(0,y) = 0. Abychom po derivování dostali členy potřebného stupně, musíme nyní ovšem předpokládat řešení ve tvaru úplného polynomu 4. stupně s neurčitými koeficienty, u(x, y) = A + Bx + Cy + Dx2 + Exy + Fy2 + Gx3 + Hx2y + Ixy2 + Jy3+ +Kx4 + Lx3y + Mx2y2 + Nxy3 + Qy4. (B.181) Příslušné druhé derivace tedy v tomto případě budou uxx = 2D + 6Gx + 2Hy + 12Kx2 + 6Lxy + 2My2, (B.182) uyy = 2F + QIx + 6Jy + 2Mx2 + 6Nxy + 12Qy2. (B.183) Pro jednotlivé koeficienty dostáváme 4 2 1 — 24N D = -4F, G = —I, H = -12J, K = —M, L =-, M = -2AQ. (B.184) 3 3 6 Dosazením Dirichletovy okrajové podmínky dostáváme A = 0, C = 0, -F = 1,J = 0, Q = 0, z relací (B.184) ihned vyplývá D = —4, H = 0, K = 0, M = 0. Dosazením Neumannovy okrajové podmínky dostáváme B = 0, E = 0, I = 0, ./V = 0, z relací (B.184) následně vyplývá G = 0, L = 1/6. Dosazením nenulových koeficientů do rovnice (B.181) dostáváme hledanou výslednou funkci, u(x, y) = -4x2 + y2 + -x3y. (B.185) 6 Podrobně je tato problematika popsána např. v učebnici Franců (2011). Příloha C Praktické základy numerických výpočtů Smyslem této kapitoly není podávat systematický popis základních metod numerické matematiky, v tomto směru odkazuji zájemce například na skriptum Humlíček (2009) nebo odpovídající učebnice (například Přikryl, 1985; Vitásek, 1987; Čermák & Hlavička, 2006, atd), ale pouze stručně a názorně ilustrovat některé principy a možné postupy při praktickém numerickém modelování nejčastěji se v praxi vyskytujících (a výše popsaných) analytických okruhů. Vybrané příklady jednoduchého modelování jsou také doprovázeny ukázkami programovacích skriptů pro daný konkrétní problém, případně obrázky a grafy výsledných modelů. Rozsáhlé využívání numerické matematiky ve většině přírodovědných a technických disciplín přineslo také tvorbu celé řady hotových knihoven, rozepsaných do hlavních programových jazyků; jejich přehledné úložiště se nachází například na stránkách GAMS (Guide to Available Mathematical Software) http://gams.nist.gov/. Některé z nich jsou komerční (a poměrně komplexní), například NAG (Numerical Algorithmus Group) https: //www.nag. com/content/ nag-library nebo IMSL (International Mathematics and Statistics Library) http://www. roguewave.com/products-services/imsl-numerical-libraries, jiné jsou volně dostupné a bývají zpravidla zaměřené na specifickou oblast, například FFTPACK http://www.netlib. org/fftpack/ - (rychlá) Fourierova transformace, LAPACK (viz odstavec Cl) - lineární algebra, MINPACK http://www.netlib.org/minpack/ - nelineární rovnice, atd. Jejich úplná nebo i částečná implementace do vlastních algoritmů může výrazně urychlit a usnadnit jejich tvorbu i kvalitu. Cl Numerické metody lineární algebry V tomto oddíle nebudeme probírat jednotlivé metody numerických řešení lineární algebry, k základům tohoto tématu existuje rozsáhlá literatura (např. Humlíček (2009)), popisující jak vlastní numerické rovnice tak jejich stabilitu a podmíněnost (tj. zejména stanovení přesnosti numerických maticových algoritmů), odhady chyb, atd. V současnosti existuje řada hotových balíčků (procedur), sestávajících z jednotlivých podprogramů (knihoven), určených pro řešení dílčích nebo i kombinovaných algebraických úloh (například řešení soustav lineárních rovnic, řešení tzv. tridiagonálních matic (tj. matic s nenulovými prvky pouze na hlavní a obou sousedních diagonálách), hledání determinantů, inverzních matic, vlastních hodnot a vlastních vektorů, atd. Jedním z nejvýkonnějších takových balíčků, který zde podrobněji představíme, je programový balíček LAPACK (Linear Algebra PACKage, Anderson et al. (1999)), který se vyvinul ze starších balíčků EISPACK a LINPACK a je určen pro fortran 77, fortran 90, existují 175 Príloha C. Praktické základy numerických výpočtů 176 také C++ verze. Existují rozšířené verze tohoto balíčku nebo další knihovny na něm postavené, se zabudovanými podprogramy pro paralelizaci na výkonných počítačových clusterech (viz odstavec C.5), například ScaLAPACK, MAGMA, MORSE, CHAMELEÓN, atd. Programový balíček LAPACK je volně dostupná softwarová knihovna, její instalaci provedeme buď ze softwarového centra používané systémové distribuce, nebo z adresy http://www. netlib.org/lapack. Při překladu (kompilaci) používaného programového souboru zadáme odkaz na LAPACK, například: gfortran název_souboru . f 95 -llapack. Popis jednotlivých podprogramů a jejich využití (např. knihovna DGBSV pro řešení soustav reálných lineárních rovnic o libovolném počtu proměnných nebo DGTSV, vhodná pro řešení tridia-gonálních matic, atd.) jsou dostupné v uživatelských příručkách, například v Anderson et al. (1999). • Ukázka schématu podprogramu DGBSV, určeného pro řešení soustavy lineárních rovnic (překlad): N (vstup, INTEGER) = počet rovnic = řád čtvercové matice A, N > 0. KL (vstup, INTEGER) = počet spodních diagonál matice A, KL > 0. KU (vstup, INTEGER) = počet horních diagonál matice A, KU > 0. NRHS (vstup, INTEGER) = počet sloupců pravé strany (tj. matice B), NRHS > 0. AB (vstup/výstup, DOUBLE PRECISION) = pole dimenze (LDAB,N). Na vstupu: matice A v pásovém uložení, v řádcích od KL+1 do 2*KL+KU+1: řádky 1 až KL pole nemusí být vypsány, j-tý sloupec pole A je uložen jako j-tý sloupec pole AB následovně: AB(KL+KU+l+i-j, j) = A(i,j) pro max(l, j-KU) < i < min(N, j+KL). Na výstupu: detaily faktorizace - matice U je uložena jako horní trojúhelníková pásová matice s KL+KU horními diagonálami v řádcích od 1 do KL+KU+1, multiplikátory M, použité během faktorizace jsou uchovány v řádcích od KL+KU+2 do 2*KL+KU+1 (viz schéma níže). LDAB (vstup, INTEGER) = určující dimenze pole AB. LDAB > 2*KL+KU+1. IPIV (výstup, INTEGER) = pole dimenze (N), indexy pivotů, které definují permutační matici; i-tý řádek matice byl zaměněn za řádek IPIV(i). B (vstup/výstup, DOUBLE PRECISION) = pole dimenze (LDB,NRHS), na vstupu je N : NRHS matice pravé strany B. Na výstupu, pokud INFO = 0, je N : NRHS řešení matice X. LDB (vstup, INTEGER) = určující dimenze pole B. LDB > max(l,N). INFO (výstup, INTEGER) = 0: úspěšný výstup, < 0: pokud INFO = -i, pak i-tý argument má nepovolenou hodnotu, > 0: pokud INFO = i, U(i,i) je přesně 0. Faktorizace je ukončena, ale faktor U je přesně singulární, řešení nemohlo být vypočteno. DALŠÍ DETAILY: Príloha C. Praktické základy numerických výpočtů 177 Schéma pásového uložení je ilustrováno na následujícím příkladu, kdy M = N = 6, KL = 2, KU = 1: Na vstupu: Na výstupu: * * * + + + * * * ul4 u25 u36 * * + + + + * * ul3 u24 u35 u46 * al2 a23 a34 a45 a56 * ul2 u23 u34 u45 u56 all a22 a33 a44 a55 a66 ull u22 u33 u44 u55 u66 a21 a32 a43 a54 a65 * m21 m32 m43 m54 m65 * a31 a42 a53 a64 * * m31 m42 m53 m64 * * Prvky pole označené * nejsou používané ve výpočetním procesu; prvky označené + nemusí být uvedeny na vstupu, ale jsou nutné ve výpočetním procesu pro uložení prvků pole U z důvodu nedostatku místa, vyplývajícího z výměny řádků. • Ukázka schématu podprogramu DGTSV, určeného pro řešení tridiagonálních matic: N (vstup, INTEGER) = počet rovnic = řád čtvercové tridiagonální matice A, N > 0. NRHS (vstup, INTEGER) = počet sloupců pravé strany (tj. matice B), NRHS > 0. DL (vstup/výstup, DOUBLE PRECISION) = na vstupu pole prvků spodní (sub) diagonály matice A, dimenze N-l, na výstupu je toto pole přepsáno N-2 prvky druhé horní diagonály horní trojúhelníkové matice U, dané LU faktorizací. D (vstup/výstup, DOUBLE PRECISION) = pole dimenze N, na vstupu obsahuje diagonální prvky matice A, na výstupu je toto pole přepsáno diagonálními prvky matice U. DU (vstup/výstup, DOUBLE PRECISION) = pole dimenze N-l, na vstupu obsahuje N-l prvků horní (super) diagonály matice A, na výstupu je toto pole přepsáno N-l prvky první horní diagonály horní trojúhelníkové matice U. B (vstup/výstup, DOUBLE PRECISION) = pole dimenze (LDB,NRHS), na vstupu je N : NRHS matice pravé strany B. Na výstupu, pokud INFO = 0, je N : NRHS řešení matice X. LDB (vstup, INTEGER) = určující dimenze pole B. LDB > max(l,N). INFO (výstup, INTEGER) = 0: úspěšný výstup, < 0: pokud INFO = -i, pak i-tý argument má nepovolenou hodnotu, > 0: pokud INFO = i, U(i,i) je přesně 0. Faktorizace je ukončena, ale faktor U je přesně singulární, řešení nemohlo být vypočteno. Obdobným způsobem jsou sestaveny i ostatní knihovny. Příklady řešení a programových skriptů s odkazem na LAPACK uvádíme v dalších odstavcích C.2.1, C.2.2, C.2.3, C.2.4, C.4.1, C.4.7, atd. C.2 Interpolace a regrese Interpolací rozumíme nahrazení složitější funkční závislosti závislostí jednodušší, tedy aproximace dané funkce jinou vhodnou funkcí. Interpolační aproximací rozumíme interpolaci diskrétní funkce, tj. funkce, dané konečným souborem bodů definičního oboru a jim přiřazených funkčních hodnot (reprezentovaných zpravidla tabulkou), pomocí funkce, nabývající v těchto bodech stejných hodnot jako původní zadaná funkce. Nej vhodnějšími interpolačními funkcemi jsou Príloha C. Praktické základy numerických výpočtů 178 polynomy, např. tzv. Lagrangeův a Newtonův interpolační polynom (Humlíček, 2009; Vitásek, 1987) nebo tzv. splajny. V následujícím odstavci C.2.1 je stručně ukázán často používaný tzv. kubický interpolační splajn. Regresí (regresní analýzou) nazýváme hledání takové funkce (tzv. regresní funkce), která nejlépe vystihuje vztah mezi dvěma skupinami proměnných, např. závislost náhodných veličin (naměřených hodnot) na čase, atd. Předem je dáno, která proměnná je nezávislá (vysvětlující nebo také regresor) a která je závislá (vysvětlovaná nebo také odezva). Jednoduchá regrese popisuje závislost odezvy na jednom regresoru, naproti tomu vícenásobná regrese popisuje situaci, kdy odezva závisí na více regresorech. Podle charakteru a průběhu zkoumané závislosti volíme typ regresního modelu, například lineární regresi (proložení závisle proměnných hodnot přímkou), regresi polynomem rt-tého stupně, atd., a také nejvhodnější statistickou metodu, například metodu nejmenších čtverců nebo tzv. robustní regresi, která eliminuje extrémně vychýlené hodnoty, atd. (viz také pojmy a statistické metody, uvedené v kapitole 12 nebo například na stránkách http://physics.muni.cz/~mikulas/zvc.html. C.2.1 Kubický interpolační splajn (z anglického spline) je jednou z nejčastěji používaných interpolačních funkcí Jedná se o tzv. po částech (piecewise) interpolační polynom 3. stupně Si ve tvaru S(x) = si(x) proxi < x < X2, s2(x) proa?2 < x < x%,..., sn_i(x) proxn_i < x < xn, (Cl) definovaný jako jehož druhé derivace označíme jako Mj. Je to tedy soustava kubických funkcí, které na sebe v zadaných bodech navazují jak funkční hodnotou, tak první a druhou derivací. Podle okrajových podmínek rozlišujeme různé typy těchto splajnů, například tzv. přirozený splajn je určen okrajovými podmínkami M\ = Mn = 0, parabolický ukončený splajn je určen okrajovými podmínkami M\ = M2, Mn = Mn_i (extrapolace nultého řádu), kubický ukončený splajn je určen okrajovými podmínkami M\ = 2M2 — M3, Mn = 2Mn_i — Mn_2 (extrapolace 1. řádu nebo také lineární extrapolace), atd. Z podmínek spojitosti funkčních hodnot i prvních a druhých derivací v bodech Xi, vyplývá pro i = 0, ... , n — 1 následující: tyto vnitřní podmínky jsou dále doplněny dvěma uvedenými okrajovými podmínkami, danými typem splajnu. Porovnáním všech uvedených podmínek ve všech uzlových bodech [xí, m] dostáváme soustavu lineárních rovnic pro neznámé druhé derivace Mj ve vnitřních uzlových bodech: sí(x) = clí(x - xí)3 + bi(x - xí)2 + a(x - xí) + di, (C.2) (C.3) (C.4) (xi+1 - xí) Mi+1 + 2 (xi+1 - xí-i) Mi + (xí - xí-i) Mi-i = 6 Vi+i ~ V i Vi - Vi-i ) (C.5) Tuto soustavu rovnic lze zapsat pomocí tzv. tridiagonální matice ve tvaru (kde zavedeme Axj = Príloha C. Praktické základy numerických výpočtů 179 Xi+i - xí, A+xí = xi+i - Xi-i, Ayi = yi+1 - yi} Aj = Ayj/Axj, A • = Ayi/Axi - Ayj/Axj) (2Axn-i Axn-i Axn_i 2A+xn_i Axn_2 Axn_2 2A+xn_2 Axn_3 \ ( Mn\ Mn_i Mn_2 Ax2 2A+x2 Axi Axi 2Axi/ M2 V M1 ) /-A„_i\ a: a: n—l n-2 n-2 n—3 A? (C.6) kterou řešíme například pomocí vhodné knihovny balíčku LAPACK (odstavec Cl). Jednotlivé koeficienty rovnice (C.2) potom snadno dopočítáme: ,, Vi+i ~ V i . on j \ xi+i -Xj Mi Mi+1 - Mj dí = yi, cí =--(Mí+i + IMi)---, bi = —, a,i = —--. (C.7) xi+1-Xi 6 2 6{xi+1-Xi) V případě konstantního kroku nezávisle proměnné Xi+\ — X{ zjednoduší do podoby 6 h = konst. se rovnice (C.5) Mi+1 + 4M,t + Mj_i = (yi+1 - 2yt + Vi_{) , (C.8) matice (C.6) bude mít potom tvar (2 1 1 4 1 1 4 1 \ ( Mn\ Mn_i Mn_2 V 1 4 1 1 2/ M2 _6_ ^2 / -Ayn_i \ Ay„_i - Ayn_2 Ay„_2 - Ayn_3 V Ay2 - Ayi Ayi (C.9) / Příklad programového skriptu pro přirozený kubický interpolační splajn (fortran 95): program nat3_splajn [deklarace názvu programu implicit none [příkaz, který ruší automatické přiřazování [písmen i, j, k, 1, m, n pro celočíselné [proměnné a ostatních písmen pro reálné [proměnné (tj. proměnné s desetinným roz-! vojem) [tabulka hodnot [xj,í/j]: ![1,1], [2,3], [3,4], [4;1,5], [5;1,5], [6,5], [7,7], [8,5], [9,2], [10,0] integer :: i, j, np [deklarace celočíselných proměnných: i = [pořadové číslo nezávisle proměnné, j = [pořadové číslo lineární rovnice, np = cel-!kový počet diskrétních hodnot. parameter (np=10) [zadání fixní hodnoty np, kterou nelze !v programu dále měnit! Príloha C. Praktické základy numerických výpočtů 180 LDB=N,RHS=1) integer :: IPIV(N) integer :: INFO,KL,KU,LDAB,LDB,N,RHS [deklarace celočíselných proměnných procedury LAPACK - viz sekce Cl. parameter(KL=np-3,KU=np-3,N=np-2,KUKL=KL+KU+l,LDAB=2*KL+KU+l,& [zadání fixních hodnot celočíselných para-! metrů Izadání parametru jako pole o N prvcích double precision, dimension(np) :: x, y [deklarace reálných veličin x, y jako pole ! (vektoru) o np prvcích s tzv. dvojitou [přesností, umožňující výpočet čísla na 16 [desetinných míst a do mocniny cca 10300 !(v závislosti na parametrech počítače). double precision, dimension(np) :: M(N,N),AB(LDAB,N),B(LDB,RHS) [zadání parametrů jako dvojrozměrných [polí double precision :: f(np), res(np), a(np), b(np), c(np), d(np) [jiný způsob deklarace reálných veličin jako [pole (vektoru) o np prvcích s dvojitou [přesností. [deklarace reálných skalárních veličin. [zadání fixní hodnoty intervalu nezávisle [proměnné, kterou nelze v programu dále [měnit! [vektor hodnot nezávisle proměnné. double precision :: h parameter (h=l.d0) x=(/(l.d0*i, i=l,np)/) y=(/l.d0, 3.d0, 4.d0, 1.5d0, 1.5d0, 5.d0, 7.d0, 5.d0, 2.d0, O.dO/) !y-ové (naměřené) hodnoty, np = počet [naměřených hodnot [cyklus výpočtu druhých derivací M, zápis do i=l,N doj=l,N if(j.eq.i)then M(i,j)=4.d0 elseif(j.eq.i-l)then M(i,j)=l.d0 elseif(j.eq.i+l)then M(i,j)=l.d0 else M(i,j)=0.d0 endif end do end do do i=l,N do j=l,N AB(KUKL+i-j, j)=M(i,j) end do end do do i=l,N [tridiagonální matice [výpočetní cyklus procedury LAPACK [výpočet pravé strany B(i,l)=6.d0/h**2.d0*(y(i)-2.d0*y(i+l)+y(i+2)) Príloha C. Praktické základy numerických výpočtů 181 cubic natural spline 8 I-1-1-1-1-1-1-1-r ^ 4 1 23456789 10 x Obrázek Cl: Graf kubického přirozeného interpolačního splajnu, popsaného v oddíle C.2.1. end do Ivolání podprogramu DGBSV (viz oddíl Cl): call DGBSV(N,KL,KU,l,AB,LDAB,IPIV,B,N,INFO) if(INFO.ne.O) write(*,*) "INFO=",INFO,"!!!" a(l)=B(l,l)/6.dO/h Ivýpočet koeficientů a,b,c,d v 1. poli splajnu b(l)=0.d0 c(l)=(y(2)-y(l))/h-B(l,l)/6.dO*h d(l)=y(l) do i=2,np-2 Icyklus výpočtu koeficientů a,b,c,d !v prostředních polích splajnu a(i)=(B(i,l)-B(i-l,l))/6.dO/h b(i)=B(i-l,l)/2.dO c(i) = (y(i+l)-y(i))/h-(B(i,l)+2.dO*B(i-l,l))/6.dO*h d(i)=y(i) end do a(np-l)=-B(N,l)/6.dO/h Ivýpočet koeficientů v posledním poli b(np-l)=B(N,l)/2.dO c(np-l)=(y(np)-y(np-l)) /h-2.dO*B(N,l)/6.d0*h d(np-l)=y(np-l) do i=l,np-l !zápis koeficientů do souboru fort.l write(l *) a(i), b(i), c(i), d(i) end do C.2.2 Lineární regrese metodou nejmenších čtverců Souborem n diskrétních hodnot odezvy (vysvětlované proměnné) m, i = 1, ... ,n, který je určen výčtem uspořádaných dvojic proložíme přímku (polynom 1. stupně) fl(x) = kx + q tak, aby součet S druhých mocnin tzv. reziduí, tj. vzdáleností bodů yi od funkčních hodnot f{xi) v bodech X{ byl minimální (2. mocniny se zde používají kvůli nezávislosti na znaménku Príloha C. Praktické základy numerických výpočtů 182 odchylky). Dostáváme tedy rovnici s = E [y* - z1^)]2 = E fa - (kx* + ^ = min< (c-10) i i pro dvě neznámé hodnoty k a q. Minimalizaci této funkce provedeme položením dS/dk = 0 a zároveň dS/dq = 0, výsledek můžeme zapsat pomocí maticového formalismu jako Xixi n ) \nl \ Eií/i (Cil) Snadno tak nalezneme výrazy pro oba hledané parametry v závislosti na uspořádané n-tici [xj)ž/j] (např. na naměřených hodnotách v závislosti na čase nebo poloze), u — n E« Xiy,i ~ E«Xi E« yi „ — E« xí E« yi ~ E«x,i E« y(i)> f(i), res(í) sumr es end program linearnLregrese [zápis sumy reziduí do souboru [ukončení programu C.2.3 Polynomiální regrese metodou nejmenších čtverců Postup uvedený v předchozím odstavci C.2.2 lze zobecnit pro polynom libovolného (m-tého) stupně, kdy analogii rovnice (CIO) můžeme přepsat do tvaru (horní indexy zde vždy znamenají Príloha C. Praktické základy numerických výpočtů 184 linear fit ^ 10 10 12 Obrázek C.2: Vykreslení příkladu lineární regrese, tj. proložení uvedených 10 bodů přímkou s parametry vypočítanými metodou nej menších čtverců. mocniny) 3=0 mm, (C.13) i=l kde koeficienty p j jsou koeficienty j-tého stupně polynomu, u lineární regrese tak platí po = q, pi = k (viz rovnice (C.ll)). Zároveň je jasné, že počet rovnic ./V v proceduře LAPACK odpovídá m + 1. Minimum rovnice (C.13) nalezneme, položíme-li dS/dpj = 0, získáme tak soustavu m + 1 = N lineárních rovnic, které můžeme vyjádřit pomocí maticového zápisu ve tvaru /E^2m i Xl V Ei< m+1 Ylí xí Ylí x'i E. xi n J (Pm\ Pl \poJ Ei xíVí V Eíž/í / (C.14) Výpočetní cyklus procedury LAPACK (viz programový skript v kapitole C.2.2) můžeme takto zobecnit do následující podoby (fortran 95): do i=l,N-l doj=l,N M(i,j)=SUM(x**(2*N-i-j)) end do end do i=N do j=l,N-l M(N,j)=SUM(x**(N-j)) end do M(N,N)=np Imatice levé strany, M(N,N) Príloha C. Praktické základy numerických výpočtů 185 do i=l,N [výpočetní cyklus procedury LAPACK do j=l,N AB(KUKL+i-j, j)=M(i, j) end do end do do i=l,N Ivýpočet pravé strany procedurou LAPACK B(i,l)=SUM(x**(N-i)*y) end do V ostatních bodech zůstává programová procedura popsaná v odstavci C.2.2 prakticky nezměněna. C.2.4 Robustní regrese V případě, že chceme eliminovat vliv velmi vychýlených („ustřelených") hodnot, zvolíme tzv. váženou nebo také robustní regresi. Robustních regresních modelů existuje celá řada (viz například Huber & Ronchetti (2009)), za všechny zde uvedeme jednoduchou tzv. Tukeyho metodu M-odhadu (Tukey's bisquare method), založenou na vážení reziduí pomocí dvojí druhé mocniny. Nejprve spočítáme nevážená rezidua ľ es jí = m — f (x i) (stejně jako např. v odstavcích C.2.2, C.2.3), potom použijeme následující váhovou funkci: Wi res_í res_í \ 6 med / (C.15) kde med je medián absolutní odchylky reziduí, kterou můžeme zvolit jako samotné reziduum, nebo odchylku každého rezidua od jejich vlastního mediánu. Váha W{ = 0, pokud absolutní hodnota rezidua |res_i| > 6med. Extrémně odchýlené hodnoty jsou takto zcela vyřazeny, méně vychýlené hodnoty jsou ponechány, avšak se sníženou váhou. Naprogramování tohoto robustního (váženého) regresního modelu je snadné, do pravé strany rovnice (C.14) vložíme vypočítané váhy: 2m E.x V E m+1 m Ej xi Ej xí Y.íxí n ) (Pm\ Pl \poJ /E. xTwíVí\ Ei xíwíVí V Y-íwíVí I (C.16) Pro výpočet mediánu existují v každém programovacím jazyce hotové moduly, jako příklad lze použít následující podprogram, výsledek je zahrnut do rovnice (C.15) (fortran 95): subroutine median(i, j, k, np, res, med) implicit none integer :: i, j, k, np double precision :: res(np) double precision, intent(out) :: med double precision :: temp (Seřazení čísel ve vzestupném pořadí: do j=l,np-l Príloha C. Praktické základy numerických výpočtů 186 W-bisquared pol2 fit 20 ^ 10 - 15 - 5 ■ least squares robust 0 0 2 4 6 8 10 12 x Obrázek C.3: Porovnání kvadratické regrese, tj. proložení polynomem 2. stupně s parametry vypočítanými metodou nejmenších čtverců podle odstavce C.2.3 (červená čára) a robustní Tukeyho metodou podle odstavce C.2.4 (modrá čára). Soubor 10 bodů o souřadnicích [1;3,5], [2;3,6], [3;4,2], [4; 17], [5;7,4], [6;8,7], [7;10,3], [8;12,8], [9;19], [10;17,1] obsahuje silně odchýlené hodnoty (hrubé chyby), na které robustní křivka reaguje slabě nebo vůbec. do k=j + l,np if(res(j)>res(k))then temp=res(k) res(k)=res(j) res(j)=temp endif end do end do (Výpočet mediánu v případě sudého nebo lichého počtu čísel: if(mod(np,2)==0)then med= (res (np/2) +res (np/2+1)) /2. dO else med=res (np/2+1) endif end subroutine medián [Následuje výpočet vah: w(y(i))=(l.dO-(y(i)/6.dO*med)**2.dO)**2.dO, atd. C.3 Numerické metody výpočtů funkcí jedné proměnné C.3.1 Hledání kořene funkce jedné proměnné - Newtonova metoda Kořeny obecně nelineární funkce (rovnice) f{x) = 0 často nelze vyjádřit formou explicitního analytického vzorce, k nalezení řešení takové rovnice musíme potom použít některou z numerických (iteračních) metod, kdy pomocí určitého počtu počátečních aproximací hledaného kořene xq generujeme posloupnost x±, x2, x%, ... , která ke kořenu xq konverguje. V některých Príloha C. Praktické základy numerických výpočtů 187 případech je třeba zadat interval a, b, který podle předběžného předpokladu obsahuje hledaný kořen, čím lépe se k němu na počátku přiblížíme, tím rychleji daná metoda konverguje. V následujících příkladech předpokládejme reálnou spojitou funkci f{x) s odpovídajícím počtem spojitých derivací na vymezeném intervalu, s hledaným kořenem /(xq) = 0. Počáteční odhad intervalu (intervalů) kde se kořen (kořeny) mohou nalézat provedeme například grafickou metodou: pomocí vhodného výpočetního programu nebo vypisováním funkčních hodnot do tabulky vykreslíme funkci f{x) a vyhledáme její přibližné průsečíky s osou x. Například u funkce, dané předpisem /(*) 3x2 + 2x - 3 (C.17) snadno zjistíme že existuje jeden reálný kořen, který musí s určitostí ležet uvnitř intervalu x0 e (2,3). Existuje celá řada možných numerických postupů (například metoda sečen, atd.), asi nej-známější je tzv. Newtonova metoda neboli metoda tečen. Vyjdeme z počáteční aproximace xq a postupně počítáme x±, x2, x%, ... . Známe-li určitou aproximaci xk a chceme určit lepší aproximaci xk+i, proložíme bodem [xk, f{xk)} tečnu ke křivce y = f (x), průsečík této tečny s osou x považujeme potom za hodnotu xk+\. Dostáváme tak rovnici popsané tečny ve tvaru f'{xk) {3x1 6xk + 2} Xk ~ Xk+l ' (C.18) z níž odvodíme vztah pro výpočet každého následujícího kroku (iterace), f{xk) Xk+l = xk f'(xk)' (C.19) Ukončení výpočtu s tím, že hodnota poslední iterace xk+\ je prohlášena za hledaný kořen xq s požadovanou přesností, nastane (podle velikosti malého čísla e které stanovuje požadovanou přesnost) například v těchto případech: nebo (C.20) Příklad možného způsobu naprogramování rovnice (C.17) - program fortran 95: program newton implicit none integer :: i [deklarace názvu programu [deklarace celočíselných proměnných v záhlaví pro-Igramu: i = pořadové číslo prostorového kroku double precision :: x, dx, f, df [deklarace reálných veličin, kde výrazy x = xk, dx != xk - xk+1, f = f(xk), df = f'(xk), s dvojitou [přesností lodhad vstupní hodnoty xk [výpočetní cyklus x=3.d0 do i = i+1 f = x**3 - 3.d0*x**2 + 2.d0*x - 3.d0 Ivlastní rovnice (C.17) df = 3.d0*x**2 - 6.d0*x + 2.d0 Iderivace funkce (C.17) dx = f/df Irovnice (C.19) x = x-dx lnová hodnota xk if (dabs(dx)dt.l.d-12) exit !stop kritérium: \xk — xk+i\ < 10" 12 Príloha C. Praktické základy numerických výpočtů 188 end do write (100,*) x !zápis kořene funkce f do souboru fort.100 stop [zastavení celého procesu end program newton Ikonec programu Tabulka výsledků programu výpočtu kořene funkce f(xk) pro jednotlivé iterace k: k Xk f{xk) xk - xk+i 3.0000000000000000 3.0000000000000000 0.27272727272727271 2.7272727272727275 0.42599549211119836 5.3581553581554045E-002 2.6736911736911733 1.4723079585858834E-002 1.9886039436713345E-003 2.6717025697475019 1.9848200396133109E-005 2.6880854327577796E-006 2.6716998816620690 3.6242120415863610E-011 4.9083680000275664E-012 2.6716998816571604 1.7763568394002505E-015 2.4057679206275180E-016 C.3.2 Numerické derivování Nejjednodušší numerická aproximace 1. derivace má podobu tzv. dopředně diference, f(x) « ^ + ^)-^), (C.21) kde h = Ax > 0, s chybou aproximace Sf'(x) vyjádřenou pomocí Taylorova rozvoje x, tj. Sf'(x) = —(h/2)f"(£), kde £ £ (x, x + h). Podobně jednoduchá je i tzv. zpětná diference, f(x) « fW-H*-h), (C.22) s chybou stejného řádu. Aproximací s vyšší přesností je tzv. centrální diference, f {x) « + (c.23) s chybou aproximace ô f (x) = — (h2/G)f'"(£), kde £ G (x — h, x + h), zabírající ovšem dva prostorové kroky (buňky) výpočetní sítě. Analogickým způsobem můžeme odvodit i 2. derivaci ve tvaru /<<(:,:) « /(* + fc)-2/W+/(x-» (&24) s chybou aproximace Sf"(x) = —(h2/12)f(4:\£i), kde £ G (x — h, x + h). Existují i přesnější a propracovanější diferenční schémata (viz například van Leer, 1977, 1982; Vitásek, 1987; LeVeque, 2002), například jednostranná aproximace 1. derivace, která je 2. řádu přesnosti, /'co«-m*)+*f(*+h)-n*+2h)t (C25) s chybou aproximace 5f'(x) = (h?/3)f"'(£), kde £ E (x,x + 2h), nebo aproximace 1. derivace, která je 4. řádu přesnosti, ve tvaru -f(x + 2h) + 8/(x + ft) - 8/(x -h)+ f{x - 2h) f (x) «-—-, (C.26) Príloha C. Praktické základy numerických výpočtů 189 tedy s chybou, která je řádu h4, atd. Jejich nevýhodou ovšem je, že zabírají několik prostorových intervalů (buněk) výpočetní sítě a také při složitých a objemných výpočtech díky nim mohou narůstat nároky na dobu výpočtu. Je proto vždy nutné zvážit výpočetní schéma, adekvátní dané úloze a její požadované přesnosti, odpovídající ale reálným možnostem používaného výpočetního zařízení. C.3.3 Numerické integrování je vždy založené na nahrazení složitě ohraničeného geometrického útvaru (plochy pod křivkou dané funkce v případě jedné proměnné) jednodušším útvarem, nebo součtem takových útvarů. Používá se také název numerická kvadratura, ve smyslu konstrukce plošných (tedy dvourozměrných, kvadraturních) útvarů. Ukážeme zde příklady pouze nejběžnějších (většinou ovšem zcela dostačujících) způsobů numerické integrace funkce jedné proměnné pomocí tzv. Newton-Cotesových vzorců, existuje samozřejmě celá řada jiných metod numerické integrace, například Gaussovy kvadraturní vzorce, Rombergova kvadratura, atd. Nebudeme zde uvádět ani přesnosti a způsob stanovení chyb, atd., vše je standardně dostupné v literatuře. Newton-Cotesovy (kvadraturní) vzorce Obdélníková metoda: Tato metoda se formálně nepočítá mezi tzv. Newton-Cotesovy vzorce, představuje sice nejjednodušší ale zároveň nejméně přesnou numerickou integrační metodu, kdy se určitý integrál dané funkce (tj. velikost plochy pod křivkou grafu funkčních hodnot funkce f{x) v rámci intervalu (a,b)) aproximuje obdélníkem. Tuto aproximaci můžeme zpřesnit, rozdělíme-li například interval (a, b) na zvolený počet n dílčích stejných intervalů, vypočítáme obdélníkovou aproximaci pro každý interval zvlášť a výsledky sečteme, tedy b f{x) áx*^Yj(a + k —) . (C.27) n ^—' \ n k=o Lichoběžníková metoda: představuje přesnější numerickou integrační metodu, kdy se určitý integrál dané funkce aproximuje lichoběžníky (body funkce se spojí úsečkami). Rozdělíme-li interval (a, b) na zvolený počet n dílčích stejných intervalů, vypočítáme lichoběžníkovou aproximaci opět pro každý interval zvlášť a výsledky sečteme, tedy /b 7 _ n-l f{x) dx « " Y [f(xk+i) + f(xk)] , (C28) n k=0 kde xk = a + k(b — a)/n. Simpsonovo pravidlo: založené na kvadratické (parabolické) interpolaci dílčích intervalů integrované funkce. V případě integrace polynomů dává tato metoda velmi přesné výsledky. Složená aproximace Simpsonovým pravidlem, kdy interval (a, b) je rozdělen na sudý počet n dílčích intervalů, má tvar (kdy xk = a + k(b — a)/n) ŕ b- n/2 / /(x)dx«—^^[/(x2fc_2)+4/(x2fc_1)+/(x2fc)]. (C.29) Ja n k=i Príloha C. Praktické základy numerických výpočtů 190 Simpsonovo 3/8 pravidlo (nebo také druhé Simpsonovo pravidlo): založené na kubické interpolaci dílčích intervalů integrované funkce. Složená aproximace Simpsonovým 3/8 pravidlem, kdy interval (a, b) je rozdělen na n dílčích intervalů, kde n je dělitelné třemi, má tvar (kdy opět x\. = a + k(b — a)/n) ŕ 3(b - ) n^ / f(x) dx « -±-^>- £ [f(X3k_3) + 3/(x3fc_2) + 3/(x3fc_i) + f(x3k)} . (C.30) Ja n k=i Obdobným způsobem lze sestrojit Newton-Cotesovy formule libovolně vyšších řádů, vyšší řády než 4 (Booleovo pravidlo) se nicméně používají málo, jejich nevýhodou je velmi rychle (až exponenciálně) narůstající chyba integrace. C.3.4 Jednoduché numerické metody řešení obyčejných diferenciálních rovnic Existuje opět celá řada způsobů numerického řešení obyčejných diferenciálních rovnic, například řešení rovnic Eulerovou metodou nebo tzv. Runge-Kuttovou metodou (metodami), atd., viz například (Humlíček, 2009). Ukážeme zde pouze často používanou jednoduchou tzv. metodu střelby a také příklad řešení obyčejné diferenciální rovnice 2. řádu pomocí tridiagonální matice (viz odstavec C.2.1): • Metoda střelby je jednoduchá metoda, založená na „vystřelení" dané funkce v závislosti na okrajových podmínkách z pevného bodu daným směrem. Hledáme potom takové koeficienty funkce, které zajistí „dopadnutí" funkce do požadovaného bodu. Metodu si ukážeme na příkladu řešení tzv. Lane-Emdenovy rovnice, což je obyčejná diferenciální rovnice 2. řádu, obvykle zapsaná v implicitním tvaru ^2j-(x2T-)+yn = 0 atedy y" + -y' + yn = 0, (C.31) xz dx \ dx) x kde x je nezávisle proměnná, y je závisle proměnná a n je konstanta. Tato rovnice je řešitelná analyticky pouze pro n = 0, 1, 5, pro všechna ostatní n musí být řešena numericky. Pro n = 0 získáme řešení přímou integrací se zahrnutím uvedených okrajových podmínek, pro n = 1 řešíme sférickou Besselovu diferenciální rovnici (viz rovnice (B.117)), Pro n = 5 dostáváme řešení prostřednictvím tzv. Emdenovy transformace, kde y = Ar^s, kde r, s jsou nové proměnné a uj = 2/(n — 1). Budeme počítat rovnici (C.31) pro n = 1,5 a s Newtonovými okrajovými podmínkami y(0) = 1, y'(0) = 0. Algoritmus je tedy „vystřelen" z bodu [0,1] vodorovně, nová hodnota y je v každém prostorovém kroku vypočítána jako y = y + (Ay/ Ax)Ax. Druhá derivace y" je rozepsána jako (y')', tedy (Ay/Ax)/Ax a každá nová hodnota y" je v každém prostorovém kroku počítána jako (Ay/Ax)/Ax = (Ay/Ax)/Ax— [{2/x)(Ay/Ax)+yn], což můžeme po vynásobení celé rovnice Ax přepsat jako (Ay/Ax) = (Ay/Ax) — [(2/x)(Ay/Ax) +yn]Ax. Numerický algoritmus rovnice (C.31) lze tedy zapsat například takto (fortran 95): • program Emden [deklarace názvu programu implicit none double precision :: x, y, dydx, n, dx [deklarace reálných veličin x, y, dydx, n, dx !s dvojitou přesností, kde dydx = y' = Ay/Ax !a dx = Ax Príloha C. Praktické základy numerických výpočtů 191 x=0.dO y=l.dO dydx=0.dO dx=l.d-3 n=1.5d0 do x=x+dx y=y+dydx*dx dydx=dydx- (2 .dO*dydx/x+y**n) *dx if(x>15.d0)exit write(l,*) x,y,dydx end do end program Emden [deklarace parametrů a okrajových podmínek [výpočetní cyklus [stop kritérium, dané předběžným odhadem [zápis do souboru fort.l [konec cyklu • Příklad řešení obyčejné diferenciální rovnice 2. řádu pomocí tridiagonální matice Řešení okrajové úlohy obyčejné diferenciální rovnice 2. řádu p{x)y"{x) + q{x)y'{x) + r{x)y{x) = f{x), na intervalu a, b, s Dirichletovými okrajovými podmínkami y{a) = A, y{b) = B, lze snadno řešit pomocí tridiagonální matice (viz odstavec C.2.1). Jako příklad řešení uvedeme jednoduchou rovnici s konstantními koeficienty y" + 3y' + 2y = (20x + 29) e3x (snadno řešitelnou i analyticky), na intervalu (0,1), s Dirichletovými okrajovými podmínkami y(0) = 0, y(l) = 1. Po přepsání dané rovnice do diferenčního schématu s n + 2 body prostorové sítě, kdy jednotlivé derivace levé strany rozepíšeme podle rovnic (C.23) a (C.24), dostáváme rovnici ve tvaru P ~ ^Qj Vi-i + (h2r - 2p) yi+ (p+ Vi+i h2fi, (C.32) kde i = 0, 1, ... , n, n + 1, s koeficienty p = 1, q = 3, r = 2as konstantním prostorovým krokem h = (xn+i — xo)/{n + 1) = [x(l) — x(0)]/(n + 1), při n = 99 tak bude h = 0,01. Označímedi jednotlivé závorky na levé straně rovnice (C.32) postupně P, Q, R, dostáváme rovnici s tridiagonální maticí na levé straně ve tvaru (Q R P Q \ R \ ( yi \ 2/2 p Q R P Q) Vn-l ( h2h - Py0 \ h2h h2fn-l \h2fn - Ryn+i Numerický algoritmus zapíšeme následujícím způsobem (fortran 95): (C.33) • program tridiag implicit none integer :: i, ni, N, LDB, NRHS, INFO [deklarace názvu programu [deklarace celočíselných proměnných: i = [pořadové číslo nezávisle proměnné, ni = [celkový počet diskrétních hodnot, ostatní [jsou parametry podpropgramu DGTSV [balíčku LAPACK - viz odstavec Cl Príloha C. Praktické základy numerických výpočtů 192 parameter(ni=99,N=ni,LDB=N,NRHS=l) [hodnoty celočíselných proměnných double precision :: x(ni) ,y(ni), h(ni), p(ni), q(ni), r(ni), f (ni), B(LDB,NRHS), DL(N-l), DU(N-l), D(N) [deklarace reálných proměnných jako pole !s dvojitou přesností double precision, parameter :: xO=0.dO, xn=l.dO, yO=0.dO, yn=l.dO [deklarace reálných konstant s dvojitou [přesností do i=l,N [výpočetní cyklus prostorového kroku x(i)=xO+(xn-xO)*i/dfloat(ni+l) [příkaz dfloat mění celočíselnou proměnnou !na reálnou end do do i=l,N [hlavní výpočetní cyklus h(i)=x(i)-x(i-l) [konstantní krok h je zde zapsán obecně p(i)=l.dO [konstantní koeficienty jsou zapsány obecně q(i)=3.d0 r(i)=2.d0 f(i)=(20.d0*x(i)+29.d0)*dexp(3.d0*x(i)) if(i.eq.l) then [spodní okrajová podmínka B(i,l)=h(i)**2.d0*f(i)-(p(i)-q(i)*h(i)/2.d0)*y0 elseif((i.gt.l).and.(i.lt.N)) then [hlavní pole B(i,l)=h(i)**2.d0*f(i) else [horní okrajová podmínka B(i,l)=h(i)**2.d0*f(i)-(p(i)+q(i)*h(i)/2.d0)*yn endif end do do i=l,N-l [zadání spodní diagonály DL(i)=p(i)-q(i)*h(i)/2.dO end do do i=l,N [zadání hlavní diagonály D(i)=r(i)*h(i)**2.d0-2.d0*p(i) end do do i=2,N [zadání horní diagonály DU(i)=p(i)+q(i)*h(i)/2.dO end do [volání podprogramu DGTSV balíčku LAPACK (viz oddíl Cl): call DGTSV(N, NRHS, DL, D, DU, B, LDB, INFO) if(INFO.ne.O) write(* *) "INFO=",INFO,"!!!" write(l,*) xO, yO do i=l,N y(i)=B(i,i) write(l,*) x(i), y(i) [zápis do souboru fort.l end do write(l,*) xn, yn end program tridiag Príloha C. Praktické základy numerických výpočtů 193 C.4 Numerické metody výpočtů funkcí více proměnných - řešení parciálních diferenciálních rovnic C.4.1 Hledání kořenů soustavy funkcí více proměnných Newtonova-Raphsonova metoda Newtonova (Newtonova-Raphsonova) metoda představuje velmi účinný nástroj také pro řešení obecné soustavy (nelineárních) rovnic. Soustavu P, obsahující n rovnic můžeme obecně zapsat jako P (x) 0, (C.34) kde i = 1, ... ,n a kde x je vektor proměnných Xj. Pomocí Taylorova rozvoje rovnice (C.34) do prvního řádu dostáváme obecný výraz pro fc-tou iteraci (k-tf iterativní krok) řešení systému rovnic Pj, který můžeme zapsat kompaktní formou JkAxk -Pk-X(x k-x\ (C.35) kde vektor Af představuje korekci řešení pro každou proměnnou x j vzhledem k předchozímu iterativnímu kroku. Explicitní zápis vektoru Af k bude mít tvar Ax „k-l „fc-l\T (C.36) Výraz Pk v rovnici (C.35) představuje vektor k-té iterace všech systémových rovnic Pk, zatímco výraz Jk značí odpovídající Jacobiho matici, jejíž každý prvek Jkj můžeme snadno analyticky vyjádřit ze systému rovnic Pk, položíme-li Jk dPk dxj (C.37) Rešíme-li například (jako jednoduchý modelový příklad) soustavu rovnic: -4 + 6y2 - 12z = 16, x 5x3 3y + z1 9, 0, (C.38) můžeme explicitní podobu rovnice (C.35) zapsat jako / 4x3 12yo 15 x2 V 3x2 -3 14y0 2z0 / x — íxAQ + %yl -I2z0- 16\ y - yo = - 5 xj] - 3 y0 + 4 ~ 9 (C.39) ) zo/ \ x3 + 7y^-z0 J kterou dále řešíme iterativně například pomocí balíčku LAPACK (viz odstavec Cl) jako soustavu tří lineárních rovnic pro tři neznámé Ax = x — xo, Ay = y — yo a Az = z — zq, z nichž potom v každém kroku získáme nové hodnoty x, y a z jako x = Ax + xq, y = Ay + yo, z = Az + Zq. • Příklad možného způsobu naprogramování soustavy rovnic (C.38) - program for-tran 95: program eqsystem implicit none [deklarace názvu programu Príloha C. Praktické základy numerických výpočtů 194 [deklarace celočíselných proměnných v záhlaví programu, viz odstavec Cl: integer :: i,j,K,INFO,KL,KU,LDAB,LDB,N,NRHS parameter(N=3,KL=2,KU=2,K=KU+KL+l,LDAB=2*KL+KU+l,LDB=N,NRHS=l) integer :: IPIV(N) [deklarace reálných veličin s dvojitou přesností: double precision :: AB(LDAB,N),B(LDB,NRHS),DER(N,N),C(N),xO,yO,zO [přesměrování konečného zápisu výsledku do souboru "solve.dat": open(10,file= "solve.dat" ,status= "unknown") xO=-4.0dO y0=-3.0d0 zO=14.0dO do [matice derivací levých stran: [odhad vstupní hodnoty x\~ [odhad vstupní hodnoty y^ [odhad vstupní hodnoty Zk [výpočetní cyklus DER(1,1 DER(1,2 DER(1,3 DER(2,1 DER(2,2 DER(2,3 DER(3,1 DER(3,2 DER(3,3 =4.d0*x0**3.d0 =12.d0*y0 =-12.dO =15.d0*x0**2.d0 =-3.0 =2.d0*z0 =3.d0*x0**2.d0 =14.d0*y0 =-l.d0 [transformovaná pásová matice AB podle schématu LAPACK, viz odstavec Cl: doj=l,N do i=max(l,j-KU),min(N,j+KL) AB(K+i-j,j)=DER(i,j) end do end do [matice pravých stran: B(l,l)=-(xO**4.dO+6.dO*yO**2.dO-12.dO*zO-16.dO) B(2,l)=-(5.dO*xO**3.dO-3.dO*yO+zO**2.dO-9.dO) B(3,l)=-(xO**3.dO+7.dO*yO**2.dO-zO) [vlastní výpočet pomocí podprogramu DGBSV, viz odstavec Cl:: call DGBSV(N,KL,KU,l,AB,LDAB,IPIV,B,N,INFO) if (INFO.ne.O) write(*,*) "INFO=",INFO,"!!!" C=(/(dabs(B(i,l)),i=l,N)/) if (maxval(C) Jt.l.d-15) exit [stop kritérium: max| Ax, Ay, Az\ < 10 15 x0=B(l,l)+x0 yO=B(2,l)+yO zO=B(3,l)+zO [zápis výsledných hodnot x, y, z, Ax, Ay, Az z jednotlivých iterací do souboru: write(10,*)xO,yO,zO,(B(i,l),i=l,N) Príloha C. Praktické základy numerických výpočtů 195 end do stop [zastavení celého procesu end program eqsystem Ikonec programu • Tabulka vypočtených hodnot Xk,yk,zk, Axk, Ayk, Azk ze všech provedených iterací: k Xk Vk Zk 1 -3.5568659855769229 -2.8660523504273505 14.644631410256411 2 -3.4484177335542667 -2.8088357894123277 14.321062859837509 3 -3.4422190042021756 -2.8052747443938260 14.300861993036721 4 -3.4421993029036972 -2.8052630869488695 14.300795971729505 5 -3.4421993027044500 -2.8052630868291244 14.300795971052235 6 -3.4421993027044500 -2.8052630868291244 14.300795971052233 k Axk Azk 1 0.44313401442307693 0.13394764957264957 0.64463141025641035 2 0.10844825202265625 5.7216561015022732E-002 -0.32356855041890148 3 6.1987293520911584E-003 3.5610450185016187E-003 -2.0200866800787826E-002 4 1.9701298478486783E-005 1.1657444956463351E-005 -6.6021307216049201E-005 5 1.9924720437988766E-010 1.1974503494876664E-010 -6.7726957780015056E-010 6 6.2835370146182566E-017 1.8208999862070576E-016 -1.3650720993330078E-015 Počáteční odhad (pokud neznáme, jako v případě standardních fyzikálních dějů, nějaké „předem očekávané" hodnoty) může být poměrně obtížný - na našem příkladě můžete vyzkoušet, že pokud zvolíme např. všechny počáteční hodnoty rovny 1, výpočet zkonverguje rovněž, ovšem bude zapotřebí 24 325 iterací (namísto 6 iterací pro uvedené blízké celočíselné počáteční odhady). C.4.2 Principy konečných diferencí Jednoduchý příklad - jednorozměrná rovnice se dvěma proměnnými: í-čas, x-délka - Burgersova (transportní) parciální diferenciální rovnice ^+jy&í)=0, {CA0) ot ox kde u je konstanta (rychlost). Numerický tvar funkce f(t,x) je reprezentován na jednorozměrné síti, tvořené M prostorovými body, x0, xi, ...,xM (x0 < xx < x2 < ... < xM)- (C41) Výpočet proběhne opakovaně během ./V časových kroků, t1=t0 + At, t2 = t1 + At = t0 + 2At, ..., tN = t0 + NAt. (C.42) Numerické řešení veličiny f(t,x) v obecném j-tém prostorovém a n-tém časovém {x = Xj, t = tn) kroku označíme Taylorův rozvoj funkce f(t, x) má tvar f(x + h,t) = f(x, t) + hd-^ + ^^g^ + 0{h3) + ... (C.43) /(* -h,t) = f(x,t) - + %d^fi ~ 0(h3) + ..., (C.44) Príloha C. Praktické základy numerických výpočtů 196 kde h = Ax je přírůstek prostorové proměnné x (viz rovnice (1.1)) a symbol O značí zanedbatelný, dále nezapočítávaný příspěvek členů vyšších řádů. V numerické matematice jsou derivace nahrazeny tzv. diferencemi (viz odstavec C.3.2): df(x,t) f(x + h,t)-f(x,t) fj+l-f; dx h Typy diferencí pro aproximace derivací 1. řádu: d£ n dx . d£n dx . d£n dx • Ax apod. (C.45) /jVi - ./;•) /ax (/f - fj-i) /Ax /jVi - fj-i) /(2Ax) dopředné diference, zpětné diference, centrální diference. (C.46) Příklad numerického diferenčního schématu pro aproximace derivací 2. řádu: d2f dx2 {fJ+l-2ff + fJ_l)/{AxÝ. (C.47) Numerické diferenční schéma uvedené transportní (advekční) rovnice (C.40) má tvar (/ľ ' - /;) _ (/ľ . " /; .) At 2Ax (C.48) kde časový krok je počítán jako dopředná diference a prostorový krok je počítán jako centrální diference. Po jednoduché úpravě dostáváme diferenční rovnici (C.48) v programovatelném tvaru: n+l f? uAt 2Ax VJJ (/IVi - f?-i) ■ (C.49) Příklad možného způsobu naprogramování rovnice (C.49) - program fortran 95: program explicit implicit none integer :: j, n, t [deklarace názvu programu integer :: nj, nn [deklarace celočíselných proměnných v záhlaví pro-[gramu: j = pořadové číslo prostorového kroku, n = [pořadové číslo časového kroku [deklarace celočíselných proměnných v záhlaví pro-[gramu: nj = celkový počet prostorových krokuj, nn != celkový počet časových kroků n parameter (nj=100, nn=200) [zadání číselných hodnot pro nj, nn double precision :: x(nj), f(nj), u(nj) [deklarace reálných veličin x, f a u jako pole (vek- Itoru) o nj prvcích s dvojitou přesností double precision :: dt [deklarace reálné veličiny dt (časového kroku) parameter (dt = l.dO, u = 1.2d0) [deklarace pevně zadaných hodnot konstantních [reálných veličin Príloha C. Praktické základy numerických výpočtů 197 0.5 - 100 Obrázek C.4: Graf (časový snímek) postupné hustotní vlny, popsané Burgersovou rovnici (C.40), modelované metodou explicitního Eulerova schématu na principu konečných diferencí (rovnice (C.49), víz také programový skript, uvedený v tomto odstavci). V grafu je zřetelná nestabilní vlnová poruha, jejíž oblast i amplituda neustále narůstá (viz sekce C.4.3). do j=l,nj x(j) = dfloat(j) if (j.le.nj/2) then f(j) = l.dO else f(j) = O.ldO endif end do t=0.d0 do t = t+dt do j=2,nj-l f(j) = f(j)-u(j)*dt*(f(j + l)-f(j- end do f(l) = f(l) f(nj) = f(nj-l) do j=l,nj write (100,*) x(j), f(j) end do write (100,*) write (100,*) if (t.gt.200.d0) exit Icyklus počáteční podmínky (počáteční funkce) [příkaz dfloat mění celočíselnou proměnnou na [reálnou !tzv. logická podmínka (kde de. znamená <) Izadaná počáteční funkce: pro x < 0.5nj —> f = 1.0, !pro x > 0.5nj —> f = 0.1 ! konec cyklu Izměna typu proměnné t na real s dvojitou přesností Ivnější (časový) cyklus [vnitřní (prostorový) cyklus l))/(x(j+l)-x(j-l)) Ivlastní rovnice (C.49) Ikonec vnitřního cyklu [vnitřní tzv. pevná okrajová podmínka Ivnější tzv. volná okrajová podmínka Icyklus zápisu do souboru s názvem fort.100 Izápis spočítaných hodnot Idvouřádková mezera, nutná pro animaci časových I cyklů Ivystoupení z časového cyklu, pokud t > 200 Príloha C. Praktické základy numerických výpočtů 198 end do [ukončení časového cyklu stop [zastavení celého procesu při t > 200 end program explicit Ikonec programu Toto tzv. explicitní Eulerovo numerické schéma je sice jednoduché, přehledné a názorné, je ale vždy numericky nestabilní (viz obrázek C.4 a odstavec C.4.3): C.4.3 von Neumannova analýza stability Jednoduchá analytická metoda, založená na předpokladu periodické numerické poruchy, tedy na Fourierovské dekompozici (rozkladu) numerické chyby Metoda byla publikována v roce 1947 matematiky Johnem Crankem a Phyllis Nicolsonovou, za spoluautorství významného matematika, fyzika a průkopníka digitálních počítačů Johna von Neumanna. Předpokládejme obecné poruchy stability (periodické perturbace, vibrace) vlnového charakteru ve tvaru ^eikjAx, (C.50) kde je amplituda vlny, k je vlnové číslo libovolné hodnoty. Pokud |£| > 1, pro n —> oo bude Kľ->oo, (C.51) porucha se neustále zvětšuje, numerické schéma je nestabilní. Pokud líl < 1, (C52) numerické schéma je stabilní. Po dosazení poruchové vlnové funkce do explicitního řešení (C.49) dostáváme uAt ^n+l _ gri^ e'ikjAx _ ""^ ťn e'ik(j+l)Ax _ e'ik(j-l)Ax ^ a po vydělení celé rovnice (C.53) výrazem ^netki^x dostáváme í = 1 - ^ (e*** - e-^x) = 1 - ^ -(fcAx). (C.54) Protože \a + ib\ = V a2 + b2, bude druhá mocnina rovnice (C.54) rovna výrazu lď = l+(^)2sin2(fcAx), (C.55) kde pravá strana zjevně bude téměř vždy větší než 1 (výjimečně se bude rovnat 1). Je tedy zřejmé, že v případě explicitního numerického schématu musí vždy platit, že |£| > 1, toto schéma je tedy vždy nestabilní. C.4.4 Laxova metoda Numerická varianta explicitního schématu, které podstatně stabilizuje, je pojmenovaná podle matematika Petera Davida Laxe. Základem je jednoduchá změna ve struktuře časového členu. Clen fj1 v explicitním řešení je zde nahrazen aritmetickým průměrem sousedních hodnot, ft1 = \ (ZlVi + íl-i) - ^ CfjVi - f?-i) , (C56) Príloha C. Praktické základy numerických výpočtů 199 0.5 - 100 Obrázek C.5: Časový snímek postupné hustotní vlny, popsané Burgersovou rovnici (C.40), modelované Laxovou metodou (rovnice (C.56)). Křivka hustoty je na rozdíl od explicitního schématu stabilní, je zde však příliš velká tzv. numerická difúzivita, projevující se značným rozmytím (rozostřením) původní vlny ostře schodovitého tvaru (srovnej s grafem C.9 v odstavci C.4.8), způsobená přidáním výrazu, odpovídajícímu druhé derivaci advekčního členu, tj. kdy výraz (fj+i + /j_i)/2 v rovnici (C.56) můžeme chápat jako f j + (fj+1 - 2fj + /j-i)/2. von Neumannova analýza stability v tomto případě dává £ = cos(fcAx) - i^— sm(kAx), a tedy |£|2 = cos2(kAx) + í ^— j sin2(fcAx). (C.57) Schéma je zjevně stabilní, pokud pro tzv. Courant-Friedrichs-Lewyho číslo uAt/Ax (zkráceně Courantovo číslo, cfl) platí uAt Ax < 1 Courantův teorém stability. (C.58) Stejná rovnice (C.40), modelovaná Laxovou metodou (C.56) je zobrazena v grafu C.5. C.4.5 Metoda zpětného kroku (Upwind method) Metoda zpětného kroku používá v prostorovém (advekčním) členu zpětnou diferenci, J j ~ J 3 U 3 J 3 -1-'' von Neumannova analýza stability v tomto případě dává (cíl = a): £ = 1 — a + a cos(fcAx) — ia sin(fcAx), a tedy, |£|2 = [1 - a + a cos(fcAx)]2 + a2 sm2(kAx). Z požadavku |£|2 < 1 vyplývá nerovnost 2a(l -a)[l- cos(fcAx)] > 0. (C.59) (C.60) (C.61) Protože pokud a > 0 potom cos(fcAx) < 1, schéma bude stabilní, pokud obdobně jako v odstavci C.4.4 Courantovo číslo a < 1. Príloha C. Praktické základy numerických výpočtů 200 0.5 - 100 Obrázek C.6: Časový snímek postupné hustotní vlny (C.40), modelované metodou implicitního schématu (rovnice (C.65)). Křivka hustoty je stabilní a na rozdíl od Laxova schématu není zdaleka tak rozšířená. Amplituda postupného klubka vlnové poruchy v levé části grafu se v čase snižuje, jeho rozsah se nemění. C.4.6 Laxova-Wendroffova metoda Dvoukroková metoda, pojmenovaná podle již zmíněného Petera Laxe (viz odstavec (C.4.4)) a dalšího matematika Buřta Wendroffa, kombinuje výhody Laxova a explicitního schématu následujícím způsobem: 1. krok (Laxův) a 2. krok (explicitní): 1 f'1' 2 _ _ ( f n , f n Ji+l — O Wi+1 J j 1 uAt (živí - m f: - von Neumannova analýza stability v tomto případě dává (cfl = a): n+l f? uAt f ^2 _ j n^2 £ = 1 — 2a sin 2 kAx a sm kAx +1 cos kAx 1 - 4a2 sin2 kAx 2 ■ 2 a sin kAx cos Z požadavku |£|2 < 1 opět vyplývá podmínka stability a < 1. a tedy kAx (C.62) (C.63) (C.64) C.4.7 Implicitní schéma Princip tzv. implicitního schématu je založen na tom, že hodnoty veličiny / v prostorovém (advekčním) členu na pravé straně rovnice (C.49) jsou zadány v čase ín+1, tedy de facto v budoucnu, tj. po provedení aktuálního výpočtu, J ] n+l f n J i uAt Jj+1 Jj-1 13 2Ax V^+i von Neumannova analýza stability v tomto případě dává (cfl = a): 1 1 — ia sin(fcAx) 1 + ias\ií{kAx) 1 + a2 sm2{kAx) a tedy |£|2 1 + a2 sin2(fcAx) (C.65) (C.66) Príloha C. Praktické základy numerických výpočtů 201 Z rovnice (C.66) je tedy zcela zřejmé, že implicitní schéma musí splňovat podmínku stability |£| < 1) Je tedy vždy numericky stabilní. Nevýhodou je komplikovanost výpočtu v každém časovém kroku, kdy tuto předem neznámou hodnotu počítáme pomocí tzv. tridiagonální matice (viz odstavce C.l, C.2.1.) a fn+l fn+l a fn+l _ m í r\ Rn\ 2-Jj-i-Jj --^Jj+1--Jj l^-o/J některou z metod nebo knihoven numerické lineárni algebry (viz odstavec C.l). Stejná rovnice (C.40), modelovaná implicitní metodou (C.65)-(C.67) je zobrazena v grafu C.6. C.4.8 Příklad pokročilejšího numerického schématu • V současnosti existuje celá řada modernějších, přesnějších a stabilnějších numerických metod (viz např. Thompson, 2006): • Použití tzv. oddělených sítí (staggered mesh), umožňující oddělení toků různých veličin (flux splitting), například vektorových a skalárních polí, atd. • Postupné přidávání jednotlivých členů pravých stran fyzikálních rovnic, reprezentujících různá silová pole (operátor splitting): Cf1 -/°)/At = Li(/°) (ŕ-n/At = L2(f) (C.68) (/m_/m-l)/Ar = Lm{fm-1), kde L j představuje jednotlivé aproximace členů pravé strany rovnice pomocí principu konečných diferencí, m je celkový počet členů na pravé straně rovnice a horní indexy udávají pořadové číslo dílčího časového kroku. • Princip oddělených sítí (staggered mesh): na A-síti „sedí" vektorové veličiny, na B-síti „sedí" skalární veličiny (viz obrázek (C.7). • Příklad dvoukrokové metody (tj. kdy výpočet následujícího časového kroku je rozdělen na dva mezikroky: explicitně vypočítaný tzv. prediktorový krok, následovaný implicitním tzv. korektorovým krokem) pro výpočet transportní rovnice (C.40) skalární veličiny /: fB _ fB fB — fB AA = \_\-\ A+ = J^+1 _ \, (C.69) xj xj-i xj+i xj kde A_, A+ jsou symboly pro zpětnou a dopřednou diferenci. Pro výpočet prediktorového kroku použijeme například tzv. van Leerovu derivaci (van Leer, 1982), definovanou jako: . (A_A+) = A2A~AA+ , if A_A+ > 0 d*L = { A_+A+ (C.70) 0, if A_A+<0. van Leerova derivace je tedy nenulová, pokud je funkce / monotónní a je nulová v těch polích prostorové sítě, kde funkce / prochází extrémy. Důležitou vlastností van Leerovy derivace je, že zachovává monotónnost derivací a zabraňuje vzniku lokálních extrémů: Príloha C. Praktické základy numerických výpočtů 202 9 b int 9 b int int b g int b g 1 2 3 4 nj 1 2 3 4, o bodyA-sítě nj+l nj+2 nj+3 nj+4 nj+l nj+2 nj+3 nj+4 body B-sítě Obrázek C.7: Schéma uspořádání tzv. oddělených sítí (staggered mesh). A-síť, určená pro počítání vektorů, je zobrazena černě, B-síť, určená pro počítání skalárních veličin, je zobrazena červeně. Dvojitou čárou je ohraničena vnitřní výpočetní doména ('inť), symbolem 'b' je označena zóna pro počítání okrajových podmínek, symbolem 'g' je označena tzv. ghost zone, což je další přidaná zóna pro okrajové podmínky, nutná pro počítání diferenciálních rovnic 2. řádu nebo v případě symetrických podmínek, například vůči ose sítě (periodické okrajové podmínky), středu sítě, atd. Přesné uspořádání zón pro okrajové podmínky se může v detailech lišit právě podle typu okrajových podmínek (pevné, reflexní, periodické, atd.). z rovnice (C.70) vyplývá, že pokud A_ ~ A+ ~ A, potom (A_A+) ~ A a pokud A_ A+, potom (A_A+) ~ min(A_,A+). To zaručuje, že hodnoty derivované funkce / na hranicích výpočetní buňky lokálně „nepřestřelí" střední hodnoty funkce / v sousedních buňkách (viz obrázek C.8). Výsledkem prediktorového kroku bude veličina / (nazveme ji například interpolant), která je během prediktorového kroku advektována na rozhraní druhé sítě, tj. z původní B-sítě na A-síť a naopak. Prediktorový krok bude mít v tomto případě tvar (A,n+a a , \ 4 - ^ . - ' , J , (C71) kde u je advekční rychlost (srovnej rovnici (C.49)) a horní index n + a označuje dílčí posun v rámci časového kroku n. Následující korektorovy krok bude dán rovnicí ve tvaru <-B,n+l _ í-B,n Aí / jA,n+a A, n+a t A, n+a A,n+a\ J3 ~h - „A _ _A Vj+l U3+l ~l3 U3 )' ^-'^ X3+l X3 Po provedení korektorového kroku se tedy skalární veličina / opět vrací na B-síť, tj. uprostřed mezi polohy A(j + 1), A(j). Rovnice (C.72) je zároveň numerickou formou jednorozměrné divergence. Obdobné schéma ve dvoj a trojrozměrné verzi se nazývá metoda Príloha C. Praktické základy numerických výpočtů 203 „B, n fj~2 i-2 xB rB,n vL i-2 xA jŕ-i j Obrázek C.8a Obrázek C.8b J Obrázek C.8: Schematické vyobrazení podmínky monotónnosti van Leerovy derivace (rovnice (C.70)) je na obrázku C.8a: sklon lineární distribuce veličiny / v prostřední výpočetní buňce (čárkovaná čára) je díky van Leerově derivaci (plná čára označená jako d^L) redukován, takže hodnoty lineárně interpolované, advektované skalární veličiny / na rozhraní buňky musí po celé šířce této buňky „padnout" mezi hodnoty této veličiny, zprůměrované přes objemy sousedních výpočetních buněk. Obrázek C.8b znázorňuje prediktorový krok advekce skalární veličiny / (rovnice (C.71)). Veličina je lineárně interpolovaná (plná červená čára, označená d^L, znázorňuje sklon van Leerovy derivace) a advektována na rozhraní buňky v polovičním časovém kroku t + At/2. Hranice buňky, vyznačené plnou čarou, symbolizují objem látky, advektovaný v čase, zatímco „čárkovaná" buňka je pevně fixována v prostoru. Poloha lineárního interpolantu / je označena jA'n+a_ Následující korektorovy krok (rovnice (C.72)) advektuje veličinu na střed B-sítě v čase t + At. konečných objemů (hnite volume method - viz např. LeVeque (2002)). Vícerozměrná podoba rovnice (C.72) (počítaná v souřadnicovém směru j, index k zde symbolizuje všechny ostatní souřadnicové směry, v závislosti na dimenzi výpočetní sítě) by tedy vypadala: /■b, n+l _ í-b,n / t A, n+a A, n+a qA j-A,n+a A,n+íiQA \ ((~\ yqN Jj,k - Jj,k ~ 77b" \2j+l,k Uj+l,k ňj+l,k ~ 1j,k Uj,k ňj,k) > (U.Mj j,k kde veličina V^k znamená objem jedné tzv. buňky výpočetní sítě (grid cell), středovaný na síti B, veličina S^k znamená potom plochu této buňky (nacházející se na síti A), přes niž prochází tok veličiny / ve směru j (viz odstavce A.1.2, A.2.2, A.3.2, A.7.2, popisující vztahy mezi těmito veličinami v různých souřadnicových soustavách - viz také obrázek C.8). Pokud bychom modelovali transportní rovnici (C.40) pro vektorovou veličinu, bude postup zcela obdobný, pouze namísto ze sítě B budeme vycházet ze sítě A, prediktorový krok transportuje tuto veličinu na síť B a následný korektorovy krok opět na síť A. • Stejná rovnice (C.49), modelovaná uvedenou metodou prediktor-korektor je uvedená na obrázku C.9. Courantovo číslo cfl = 0.5. • Numerické schéma, uvedené v tomto odstavci, není zdaleka jediné možné, představuje pouze ukázku tzv. po částech lineární metody (Piecewise Linear Method), kdy numerické diference jsou prokládány úsečkami. Je možné použít i přesnější tzv. po částech Príloha C. Praktické základy numerických výpočtů 204 0.5 - 100 Obrázek C.9: Časový snímek postupné hustotní vlny, popsané Burgersovou rovnici (C.40), modelované metodou prediktor-korektor (pomocí rovnic (C.71) a (C.72)). Křivka hustoty je na rozdíl od předchozích schématu stabilní a ostrá, malý sklon čela vlny je dán hustotou výpočetní sítě (vzdáleností sousedních prostorových bodů). Tvar vlny lze korigovat přidáním střihové tzv. Navier-Stokesovy viskozity nebo objemové, tj. v praktických výpočtech používané tzv. numerické viskozity (viz například, LeVeque, 2002, a další). parabolickou metodu (Piecewise Parabolic Method - PPM, viz např. Colella & Woodward (1984)), nevýhodou je ovšem zcela zákonitě vyšší výpočetní náročnost, tj. nároky na výkon počítačů, atd. Kromě toho existuje celá řada jiných metod, založená na jiných principech numerického derivování, jiných typech prostorových sítí (například tzv. adaptivní sítě, které se v průběhu času samy mění), nebo k výpočtům vůbec prostorové sítě nevyužívají - např. tzv. SPH metoda (Smooth Particle Hydrodynamics), atd. C.4.9 Příklady modelování reálných fyzikálních procesů Riemannova-Sodova rázová trubice: Základní testovací úloha pro většinu numerických kódů se snadno ověřitelnými výsledky. Jedná se o uzavřenou trubici, respektive box, rozdělený na dvě části pevnou přepážkou, nazývanou též diafragma ( latinský název pro bránici), kde obě oddělení jsou naplněné plynem s rozdílnými hustotami a tlaky. Náhle přepážka zmizí což vyvolá pohyb plynu předcházený rázovou vlnou šířící se kolmo k rovině původní přepážky ve směru řidšího plynu. Obrázek (C.10) ukazuje snímek průběhu hustoty, kdy počáteční stav plynu (kde index L označuje levou stranu trubice s vyšší počáteční hustotou a tlakem ,index R označuje pravou stranu trubice s nižší počáteční hustotou a tlakem) je zvolen následovně: pi = 1.0, pr = 0.125, Pl = 1.0, Pr = 0.1, 7 = 5/3 kde p je hustota, P je tlak a 7 je adiabatická konstanta. Obrázek C.ll ukazuje obdobnou testovací úlohu s počátečními průběhy veličin proměnnými v obou směrech x, y, s následujícími 2 2 2 2 parametry: pí = e~y , pr = 0.125 e~y , Pl = e~y , Pr = 0.1 e~y , 7 = 5/3. Profily hustoty a tlaku v příčném směru y jsou tedy „Gaussovské". V tomto modelu je ještě přidána „porucha", způsobená malou počáteční složkou rychlosti Vv = 0.05. Príloha C. Praktické základy numerických výpočtů 205 Obrázek C.10: Výsledek simulace hustoty p v Riemannově-Sodově rázové trubici v případě neviskózního toku v čase t = 0.28 (v jednotkách odpovídajících popisu v odstavci C.4.9). Počáteční stav plynu je statický a je hxován pevnou přepážkou (nazývanou také diafragma), situovanou v 1/3 délky trubice. Hodnoty hustoty p a tlaku P na levé straně přepážky jsou pi, = 1.0, Pl = 1.0, hodnoty na pravé straně přepážky jsou pp> = 0.125, Pr = 0.1. Celková délka x šířka trubice (boxu) je 4.0 x 2.0 v libovolných jednotkách a je zde použita výpočetní síť s počtem 300 x 100 zón, okrajové podmínky jsou „pevné stěny". Tři charakteristické „schody" v hustotě jsou (zprava doleva) vlastní rázová vlna (jejíž rychlost šíření může až čtyřikrát převyšovat skutečnou rychlost pohybujícího se plynu), dále tzv. kontaktní nespojitost, což je místo původní přepážky, šířící se vlastní rychlostí pohybujícího se plynu a konečně tzv. zřeďující vlna, šířící se opačným směrem (viz grafy stejné testovací úlohy například v Stone & Norman, 1992). f = 5.32 0.5 5», 0 -0.5 0.8 0.6 0.4 0.2 1.5 2.5 3.5 4.5 Obrázek C.ll: Barevný graf průběhu hustoty ve stejné Riemannově-Sodově rázové trubici v čase t = 5.32, s přidanou malou počáteční y-ovou složkou rychlosti, Vy = 0.05. Tato „porucha" způsobí určitou příčnou deformaci toku kde jsou rovněž viditelné Kelvinovy-Helmholtzovy a Rayleigh-Taylorovy nestability. Príloha C. Praktické základy numerických výpočtů 206 f = 18.0 0 0.2 0.4 0.6 0.8 1 x Obrázek C.12: Barevný graf průběhu hustoty v Kelvinově-Helmholtzově nestabilitě (viz odstavec C.4.9). Snímek ukazuje tok v pokročilém čase, kdy je nestabilita již zcela nelineární, tj. s plně rozvinutými turbulencemi. Kelvinova-Helmholtzova nestabilita Dalším oblíbeným testovacím problémem je modelování Kelvinovy-Helmholtzovy nestability (viz např. Chandrasekhar, 1961, viz také obrázek C.12). Pravoúhlá oblast (box) je naplněná plynem se dvěma opačně směřujícími toky, oddělenými lineární pomyslnou diskontinuitou. Okrajové podmínky jsou periodické na čelních okrajích toků, tj. v obrázku C.12 na stranách se souřadnicemi x = 0 a x = 1, zatímco na zbývajících dvou stranách jsou zvoleny opět jako „pevné stěny". Počáteční podmínky k úloze jsou převzaty z parametrů, uvedených v instrukcích ke kódu ATHÉNA (Stone et ak, 2008; Springel, 2013): pro y > 0.5 je podélná rychlost toku VX)\ = 0.3 a hustota plynu p\ = 1, pro y < 0.5 je podélná rychlost toku VX)2 = —0.3 a hustota plynu p2 = 2. Počáteční tlak P = 1.0 v celé výpočetní oblasti a adiabatický exponent 7 = 5/3. Abychom se vyhnuli naprosto ostrému rozhraní mezi oběma toky, definujeme přechodovou oblast která propojí oba toky, popsanou rovnicemi (Springel, 2013): p(x,y) =pi + (p2-pi) (l+e1^) \ (C.74) která charakterizuje počáteční poruchu hustoty ve směru y, podobně Vx(x,y) = Vx,1 + (Vx,2-Vx,1) (l+e^y1, (C.75) která charakterizuje počáteční poruchu x-ové složky rychlostního pole ve směru y, kde střední kvadratická odchylka rychlosti o = 0.01. Do těchto počátečních podmínek vložíme periodickou poruchu y-ové složky rychlosti ve tvaru Vy(x,y) = Acos(kx)e-k\y-°-5\, (C.76) s vlnovým číslem k = 2 x (2tt/L) a amplitudou poruchy A = 0.05. Význam tohoto testu spočívá také ve snadném ověření linearity nárůstu poruchy v rané fázi průběhu úlohy, zatímco Príloha C. Praktické základy numerických výpočtů 207 později je průběh vývoje poruchy zjevně nelineární, což vylučuje provedení analytických kvantitativních výpočtů. Navíc, „ostrost" rozhraní mezi oběma protisměrnými toky může sloužit jako indikátor tzv. numerické difúzivity (tj. stabilizace algoritmu advekčního schématu pomocí druhých derivací toku) výpočetního schématu (Stone et al., 2008). C.5 Paralelizace výpočetních algoritmů Pro urychlení a často dokonce i pro samotné umožnění výpočtu velmi rozsáhlých (jednorozměrných nebo vícerozměrných) algoritmů (kódů) je nezbytné tyto algoritmy paralelizovat, tj. rozdělit je na více oddílů (procesů) souběžně (paralelně) počítatelných na odpovídajícím počtu strojových procesorů. Principem paralelizace je tedy rozdělit celkovou prostorovou výpočetní oblast (viz například obrázek C.7) na množství separátních výpočetních oblastí, ranků (ranks). Tyto ranky lze, v závislosti na povaze problému, počítat buď zcela samostatně, nebo, pokud je nutná vzájemná „komunikace" na styku těchto ranků (například při hydrodynamických výpočtech, kde je nutná návaznost na okrajové podmínky na hranicích celé výpočetní oblasti, jsou na hranicích ranků předávány informace o hodnotách výpočtů v sousedním ranku). Tato „meziranková komunikace" přitom nepůsobí žádné zásadní zpomalení výpočtu. Existuje řada specializovaných knihoven pro tvorbu paralelních algoritmů, asi nej rozšířenější z nich je knihovna MPI (Message-Passing Interface), včetně několika podtypů, vytvořená skupinou výzkumných a vývojových pracovníků z akademické a průmyslové sféry pro široké využití na paralelně řazených počítačích. Oficiální zdroj knihovny včetně programovacích manuálů je na webové stránce http://www.mpi-forum.org/, pro úvodní seznámení se s knihovnou i s technikami paralelního programování doporučuji skripta Lísal (2007), pro podrobnější studium manuál Pacheco (1998). Knihovna je naprogramována pro přesun dat z jednoho procesu do jiného procesu pomocí kooperativních operací v každém procesu (tzv. point-1 o-point komunikace mezi dvěma procesy). Hlavním smyslem používání metod paralelního programování je významné urychlení výpočtů jak v případě zcela samostatně pracujících ranků, tak v případech, kdy je nutná vzájemná hraniční „send and receive" komunikace. Často je výpočet na jednom procesoru dokonce neproveditelný, v případě, že binární soubor indikuje neúměrně rozsáhlý výpočetní proces, nelze zdrojový soubor vůbec zkompilovat. Knihovna MPI je vyvinutá pro různé programovací jazyky, jako jsou Fortran, C, C++, Python a Java, mohou zde být ovšem velké dílčí rozdíly v organizaci výpočtu (například rozdílné pořadí zahrnování prostorových buněk při dvourozměrném paralelním výpočtu v případě jazyka Fortran, kdy výpočet „běží" v rámci každého ranku nejprve ve „vertikálním" směru, zatímco v případě jazyka C výpočet „běží" vždy nejprve „horizontálně"). Protože se v současnosti jedná již o velmi rozsáhlou a specializovanou disciplínu, nebudeme zde detailněji popisovat techniky paralelního programování. V rámci počítačových volně vázaných seskupení (počítačových clusterů), pracujících v České republice, lze standardně docílit současné zapojení až několik stovek procesů. Dostupnými a výkonnými počítačovými clustery například jsou: • METACENTRUM, což je virtuální organizace, která řídí a distribuuje výpočetní infrastrukturu spolupracujících akademických a univerzitních center. Výpočetní a paměťová zařízení jsou spravována v rámci projektu „Czech National Grid Infrastructure", který je součástí projektu „Projects of Large Infrastructure for Research, Development, and Innovations" (LM2010005). Součástí počítačového clusteru METACENTRUM jsou: výpočetní centrum Masarykovy univerzity v Brně (centrum CERIT-SC, Loschmidt Laboratories -pracoviště Ústavu experimentální biologie PřF MU a NCBR - Národní centrum pro výzkum biomolekul, PřF MU), výpočetní centrum Západočeské univerzity v Plzni (KIV Príloha C. Praktické základy numerických výpočtů 208 - Katedra informatiky a výpočetní techniky FAV ZČU, KM A a KKY - Katedra matematiky a Katedra kybernetiky FAV ZČU), výpočetní centrum Jihočeské univerzity v Českých Budějovicích (Přírodovědecká fakulta JU), výpočetní centrum Akademie věd ČR, výpočetní centrum Katedry telekomunikační techniky FEL ČVUT v Praze, atd., zastřešující organizací je e-infrastruktura pro vědu, výzkum a vzdělávání CESNET z.s.p.o. Celkové parametry a výkon clusteru převyšují 10 000 CPU počítačových jader (desítky TB operační paměti RAM) a s paměťovou kapacitou cca 1 PB (1063 TB) pro operační data a cca 19 PB (19 000 TB) prostoru pro ukládání dat. Oficiální webovou stránkou je http://metavo.metacentrum.cz/. • Počítačový cluster ANSELM (národní superpočítačové centrum, VSB - Technická univerzita Ostrava), který sestává z celkem 3 344 počítačových jader CPU (15 TB operační paměti RAM). Oficiální webovou stránkou je http://www.it4i.cz/ • V současnosti je již k dispozici uživatelům nový počítačový cluster SALOMON (národní superpočítačové centrum, VSB - Technická univerzita Ostrava), který je dle žebříčku TOP 500 oficiálně 40. nej výkonnějším superpočítačem na světě! Současné parametry: 24192 jader CPU Intel Xeon (Haswell-EP), 129 TB operační paměti RAM, 52 704 jader akceleračních koprocesorů Intel Xeon Phi s 13,8 TB RAM, 2 PFLOP/s maximální výpočetní výkon, 2 PB diskové kapacity a 3 PB zálohovací páskové kapacity. Oficiální webovou stránkou je http: //www. it4i. cz/. Reference Abramowitz, M., & Stegun, I. A. 1972, Handbook of Mathematical Functions Anderson, E., Bai, Z., Bischof, C, et al. 1999, LAPACK Users' Guide, 3rd edn. (Philadelphia, PA: Society for Industrial and Applied Mathematics) Arfken, G. B., & Weber, H. J. 2005, Mathematical methods for physicists 6th ed. Arsenin, V. J. 1977, Matematická fyzika, https://vufind.lib.cas.cz/Record/000065882 Bartsch, H.-J. 2008, Matematické vzorce, http://www.academia.cz/matematicke-vzorce. html Germák, L., & Hlavička, R. 2006, Numerické metody Chandrasekhar, S. 1961, Hydrodynamic and hydromagnetic stability Colella, P., & Woodward, P. R. 1984, Journal of Computational Physics, 54, 174 Franců, J. 2011, Parciální diferenciální rovnice, skriptum, FSI VUT Brno Huber, P. J., & Ronchetti, E. M. 2009, Robust Statistics, 2nd ed., http://eu.wiley.com/ WileyCDA/WileyTitle/productCd-0470129905.html Humlíček, J. 2009, Základní metody numerické matematiky, skriptum, MU Brno Kvasnica, J. 2004, Matematický aparát fyziky, Academia, AV CR, Praha, 2nd edn. Lísal, M. 2007, Paralelní programování s aplikacemi, skriptum, UJEP Ústí nad Labem, http: //physics.uj ep.cz/~mlisal/par_progrm/pprg_esf-web.pdf Lenc, M. 2001, Poznámky k přednášce Elektrodynamika a teorie relativity, skriptum MU Brno LeVeque, R. J. 2002, Finite Volume Methods for Hyperbolic Problems, 1st edn. Musilová, J., & Musilová, P. 2006, MATEMATIKA I pro porozumění i praxi Pánek, P. 2001, Úvod do fyzikálních měření, skriptum, MU Brno Pacheco, P. S. 1998, A User's Guide to MPI, http://www.sdsc.edu/~allans/cs260/docs/ MPIusersguide.pdf Plch, R. 2002, Příklady z matematické analýzy, diferenciální rovnice, skriptum, MU Brno Pospíšil, Z. 2006, Rovnice matematické fyziky, skriptum, MU Brno Přikryl, P. 1985, Numerické metody matematické analýzy 209 Reference 210 Rektorys, K. a. k. 2009, Přehled užité matematiky I.,11., http://knihy.abz.cz/prodej/ prehled-uzite-matematiky-i Springel, V. 2013, Lectures on high-performance computing and numerical modeling, http://obswww.unige.ch/lastro / conferences / sf2013/hands-on-2.pdf Stone, J. M., Gardiner, T. A., Teuben, P., Hawley, J. F., k Simon, J. B. 2008, ApJS, 178, 137 Stone, J. M., & Norman, M. L. 1992, ApJS, 80, 753 Thompson, M. J. 2006, An introduction to astrophysical fluid dynamics van Leer, B. 1977, Journal of Computational Physics, 23, 276 van Leer, B. 1982, in Lecture Notes in Physics, Berlin Springer Verlag, Vol. 170, Numerical Methods in Fluid Dynamics, ed. E. Krause, 507-512 Vitásek, E. 1987, Numerické metody Young, E. C. 1993, Vector and tensor analysis, 2nd ed., http://searchworks.stanford.edu/ view/2470130 http://physics.muni.cz/~czudkova/PRIKLADYZ.pdf http://physics.muni.cz/~chm/priklady.pdf http://physics.muni.cz/~mikulas/zvc.html http://www.sagemath.org/ http://www.salford.ac.uk/ http://www.wolframalpha.com/